You are on page 1of 121

Chapter 1

Structure and
Properties of Materials
chapter highlights

☞☞ Structure of Materials ☞☞ Cyniding or Liquid Carbon Nitriding


☞☞ Crystallography ☞☞ Nitriding
☞☞ Bravais Lattices ☞☞ Flame Hardening
☞☞ Metallic Structure ☞☞ Spherodizing (Spherodize Annealing)
☞☞ Imperfection in Crystal Structure of Metals ☞☞ Process Annealing or Recrystallisation
(defects) Annealing
☞☞ Line Defects ☞☞ Iso Thermal Annealing
☞☞ Allotropic Forms of Materials ☞☞ Composition and Application of Some
☞☞ Properties of Engineering Materials Non-ferrous Alloys
☞☞ Stress–strain Diagram for Engineering Materials ☞☞ Organic Materials
☞☞ Effect of Carbon Percentage on Stress-strain ☞☞ Brinell Hardness Test
Diagram of Steel ☞☞ Rockwell Test
☞☞ Carburising ☞☞ Vicker’s and Knoop Hardness Test

Structure of materials 3. Hexagonal


4. Orthorhombic
Crystallography 5. Rhombohedral
Arrangement and disposition of atoms with in a crystal is 6. Monoclinic
called structure 7. Triclinic
Atoms of a crystalline material is arranged in a definite
geometrical order. When these materials solidify from other Bravais Lattices
state, the atoms arrange themselves into specific positions
According to Bravais there are 14 possible types of space
in 3 dimensional space to form crystals. If the centres of
lattices in the 7 crystal systems mentioned above. They are
these atoms in a crystal are joined, the lines joining form a
3 dimensional geometrical shape called space lattice. The 1. Simple cubic lattice
smallest volume of this space lattice which properly repre- 2. Body centred cubic lattice
sents the positions of atoms with respect to each other is 3. Face centred cubic lattice
known as a unit cell. Unit cells are the building blocks of a 4. Simple tetragonal lattice
crystal. This is the simplest volume which completely fills 5. Body centred tetragonal lattice
the space and has the characteristics of the whole cell. 6. Simple hexagonal lattice
If the atoms in a crystal is replaced by smaller circles or 7. Simple orthorhombic lattice
points, a group of small circles or points is obtained. Each 8. End centred orthorhombic lattice
point has its surrounding identical to that of every other 9. Body centred orthorhombic lattice
point. These points are called lattice points. Thus lattice 10. Face centred orthorhombic lattice
points are corner points of unit cells. 11. Simple rhombohedral lattice
There are seven types of basic crystal systems. They are 12. Simple mono clinic lattice
1. Cubic 13. End centred monoclinic lattice
2. Tetragonal 14. Simple triclinic lattice
3.764  |  Part III  •  Unit 7  •  Manufacturing Technology

Metallic Structure 5. Schottky defect


We have seen that there are 14 valid lattices and a crystal When a pair of positive and negative ions is missing
belong to any one of these. But it is found that most of the from ionic crystal, the defect is known as Schottky
common metals have cubic or hexagonal structure only defect. Due to this defect there is no change in the
Three most common crystal lattice structures are neutrality of the crystal.

1. Face Centered Cubic (FCC) Line Defects


2. Body Centered Cubic (BCC) The type of defects are due to dislocation or distortion of
3. Hexagonal Close-Packed (HCP) atoms along a line. Examples are
In FCC structure, unit cell is a cube with one atom at each 1. Edge dislocation
corner of the cube and one each face. When a half plane of atoms is inserted between the
Examples are – Ca, Ni, Cu, Ag, Pt, Au, Pb, Al and alloys planes of atoms, the defect is called edge dislocation.
of these. They have good ductility and good electrical 2. Screw dislocation
conductivity. This occurs when a part of the crystal displaces angu-
In BCC structure, atoms are present at all eight corners larly. Magnitude and direction of edge dislocation is
of the cube and another one at the body centre. given by a vector called Burger vector.
Examples are – Fe, Na, K, V, Mo, Ta, W
They have more tendency to deform plastically. Surface Defects
In HCP structure, atoms are at each corner of a hexago- Surface defects or plane defects occur on the s­ urface of a
nal prism and one atom each at the centre of top and bottom material. If happens due to imperfect ­packing during crys-
faces. And three atoms in the mid plane. tallization or defective orientation of the surface.
Examples are – Be, Mg, Zn, Cd, Co, Ti
They have good ductility and can be easily deformed. Grain Boundary Defect
Imperfection in Crystal Grains of different orientations separate the general patterns
of the atoms exhibiting a boundary. This defect is called
Structure of Metals (Defects)
grain boundary defect.
Strength of a metal is always less than the value obtained Twinning: In twinning or twin boundary defect, atomic
by theoretical calculations. This is due to imperfections arrangement on one side of the boundary is a mirror image
and defects in crystalline structure. The following are three of arrangement of atoms on the other side.
types of important defects.
1. Point defects Volume Imperfection
2. Line defects Volume imperfections are 3-dimensional imperfections
3. Surface defects or planar imperfections inside the solids. The reasons are foreign particle inclusions
region of non-crystallizing, pores etc.
Point Defects
Point defects take place due to imperfect packing of atoms Allotropic Forms of Materials
during crystallization. Vibration of atoms at high tempera- Some materials exist in more than one form depending up
ture also cause point defects. on temperature. They are called allotropic forms.
1. Vacancy defect For example,
When one or more atoms are missing from a normally Cobalt
occupied position, the defect is known as vacancy. Below 420°C - HCP
470°C to 1495°C = FCC
2. Interstitial defects Chromium
When an extra atom occupies interstitial positions Below 20°C - HCP
(voids) in the parent crystal, the defect is called 20°C to 1799°C - BCC
interstitial defect. Fe
3. Frenkel defect Below 912°C - BCC
It is a combination of vacancy and interstitial defect. 912°C to 139°C – FCC
A missing atom responsible for vacancy occupies an In carbon steels, temperature ranges vary depending upon
interstitial position. the carbon content. Carbon steel with 0.8% carbon has a
4. Substitutional defect BCC structure below 723°C.
When a foreign atom displace a parent atom, the Usually non-crystalline materials do not have lattice
defect is known as substitutional defect. arrangement of their atoms. Such materials are known as
Chapter 1  •  Structure and Properties of Materials  |  3.765

amorphous substances. Some of the common examples of Similarly shearing stress


amorphous substances are rubber, transparent polymers, ΔQ
metallic glasses thermoset plastics etc. τ = lim
ΔA→ 0 ΔA
dQ
Properties of Engineering =
dA
materials where Q is the shearing resistance.
Engineering materials can be metalic or non-metallic. When the body is subjected to forces, it undergoes
Mettalic materials may be ferrous or non-ferrous. Ferrous changes in shape and size also.
materials consist mainly of iron with comparatively small It is seen that bars extend under tensile force and shorten
addition of other materials. Cast iron, wrought iron and car- under compressive force. The change in length per unit
bon steels are examples. length is known as linear strain.
Non-ferrous materials contain little or no iron. Examples ∆L
Thus linear strain e =
are Aluminium, copper, zinc, tin, nickel, titanium etc. L
Non-metallic materials include plastics, rubber, leather, Experiments has shown that for a body under the action
wood, glass etc. of forces, the stress and strain has got certain relationship

Types of Materials Hooke’s Law


Robert Hooke, from his experiments concluded that stress
Materials can be classified into three groups.
is proportional to strain up to elastic limit. This is known as
1. Isotropic material Hooke’s law.
2. Anisotropic material Thus s ∝ e
3. Orthotropic material or s = Ee
A material that displays same elastic properties in all where E = constant of proportionality and is known as
­directions of loading is called isotropic material. The equa- Young’s modulus or modulus of elasticity.
tion of strength and elasticity are based on this a­ ssumption. Stress–strain Diagram for Mild Steel
In these materials all planes are planes of symmetry. All met-
als are isotropic. Number of elastic constants are two only.
In anisotropic materials different property values are x
D
exhibited in different directions with respect to a set of ref-
B
erence axes. For these materials there are no planes of sym- x E
metry. Total number of elastic constants are 36 in which 21 A’ x x
A x C
numbers are independent.
Orthotropic material is a type of anisotropic material.
It has got 3 orthogonal planes of symmetry. There are 12
­elastic constants in which 9 are independent.
Stress

Stress–strain Diagram for


Engineering Materials
When a body is subjected to forces it develops internal
resisting forces. The resisting forces may be split into nor- O Strain
mal and parallel to a section. Resisting force parallel to the
section is called shearing resistance. Intensity of resisting When a mild steel specimen is subjected to tensile load and
force normal to the section is called intensity of normal the stress-strain relationship is plotted, the following salient
stress or simply the normal stress. points are observed.
Normal stress is mathematically expressed as
Limit of Proportionality (A)
ΔR
σ = lim Upto this point stress is proportional to strain.
ΔA→ 0 ΔA
dR Elastic Limit (A′)
=
dA This is slightly above A and up to this point the elastic-
ity is maintained or the strain disappears completely and
where R is the normal resisting force.
the original length is regained, when it is unloaded below
this point.
3.766  |  Part III  •  Unit 7  •  Manufacturing Technology

Upper Yield Point (B) In these materials there is no clear cut yield point. At ulti-
When this point is reached stress starts reducing and mate stress necking takes place and breaking occurs at a
extension increases or yielding of the material takes lower stress as in the case of mild steel.
place. The stress at which 0.2% permanent set is there is treated
as yield stress.
Lower Yield Point (C) (The percentage of permanent strain left in the material
when the material is unloaded after loading above elastic
At this point stress and strain increases again. point is called permanent set.)
Ultimate Stress (D) Effect of Carbon Percentage on Stress-strain
This is the maximum stress the material can resist. After Diagram of Steel
this point cross sectional area reduces at a section (­necking).
Stress again reduces and strain increases.
Increasing %
Breaking Point (E) of carbon × High speed steel
Stress at this point is called fracture stress. This is the point × High carbon steel
at which the specimen breaks. × Medium carbon steel
The region C to D is called work hardening or strain σ × Mild steel
hardening region. × Wrought iron

Stress-strain Relationship in Brittle Materials


(example-cast iron)
Stress-strain relationship for a brittle material like cast iron →ε
is shown in the figure.
Characteristic Changes in Properties of
Materials Showing Different Stress-strain
Relationships

Stress

Hard Strong

weak
Stress → Soft
σ
In this case there is no appreciable change in rate of strain.
There is no yield point. There is no necking and ultimate Brittle Ductile
point and breaking point are same. Rigid
Elastic

Stress-strain Relationship in Aluminium and →ε


High Strength Steel
Properties

σv
F 1. Strength
Resistance offered by a material on application of external
force is called strength. Or it is the ability to sustain loads
without distortion. It can be tensile, compressive, shear,
bending or torsional.

2. Hardness
Stress

It is the resistance of the material to mechanical indentation.


It is the general indication of the strength as well as resist-
ance to wear and scratching.
F’ Diamond is the hardest known material.
0.2 Stress →
Chapter 1  •  Structure and Properties of Materials  |  3.767

3. Toughness Anisotropy
It is the ability of a material to with stand suddenly applied load If elastic propertries are different in different directions it is
and thus absorb a certain amount of energy without failure. anisotropic.
It depends both on strength and ductility. Alloy steels
with good toughness are used for manufacturing of cutting
tools like milling cutters gears subjected to impact loads etc.
Equilibrium Diagrams
Pure metals have clearly defined freezing/melting point,
4. Ductility solidification/melting lakes place at constant temperature.
Ductility is the property of a material which permits perma- But in the case of an alloy solidification/melting takes place
nent deformation before fracture by stress in tension. It is the over a range of temperature.
extend to which a material can sustain plastic deformations
under tension, before rupture gold is most ductile of all metals.
Cooling of liquid
5. Brittleness
Brittleness is the inability to undergo plastic deformation. Freezing begins
Materials that under go very little plastic deformation is Freezing ends
called brittle. If the percentage of elongation is less than 5%
the material is treated as brittle. If it is more than 5% it is duc- Cooling of solid
tile. Cold shortness is the brittleness that exist below recrys-
Temp
talisation temperature. Red shortness is the brittleness of steel
at high temperature caused by the formation of iron sulphide. Time
Cooling curve of a pure metal
6. Malleability
Malleability of a material is its ability to be flattened in to 1455°C Liquid (L)
thin sheets without cracking in the cold state under com-
pressive loading (pressing, rolling, hammering, etc.)
L +S
7. Resilience
It is the capacity of a material to absorb energy elastically.
Energy stored is released on removal of the load. It is impor- Temp Solid (S) 1083°C
tant property desirable for shock absorbers and springs.
8. Stiffness
It is the resistance of a material to elastic deformation or Ni = 100% Ni = 0
deflection. Cu = 0 composition Cu = 100%
9. Fatigue Cooling curve of an alloy (Ni + Cu)
Under fluctuating or repeated loads, a material tend to
develop a characteristic behavior which is different from Allotropic Forms of Steel
that under steady loads and then the material is said to be For metals with different allotropic forms the curves are
another fatigue. Due to fatigue, material will fail at loads not as simple as shown above. The transformation from one
less than loads at normal conditions. The maximum stress at allotropic form to another takes place at a constant tempera-
which even a billion reversal of stresses cannot cause failure ture just the same way as in the case of liquid – solid trans-
is called endurance limit. formation. Iron exist in different allotropic forms depending
10. Creep up on temperature. Cooling curve for pure iron is shown in
Creep is the slow plastic deformation of a material under this figure.
constant stresses usually at high temperature.
1600 Liquid iron
11. Elasticity 1537
It is the ability of a material to return to original shape from Delta iron (BCC)
1392
the deformed shape caused by an applied load, up on the Ar4
removal of the load.
Temp°C Gamma iron(FCC)
12. Plasticity
Plasticity is the ability of the materialto undergo some 910
Ar3 Beta iron(BCC)
degree of permanent deformation without rupture.
768 Curie temp
Ar2
Isotropy Ferro
magnetic
Alpha iron (BCC)

If a material has same elastic properties in all direction, it is


an isotropic material. Time
Cooling curve for pure iron
3.768  |  Part III  •  Unit 7  •  Manufacturing Technology

The first horizontal step at 1537°C represents liquid to a – Alpha iron (Ferrite)
solid transformation with evolution of heat. The mixture P – Pearlite
consist of liquid plus delta iron solid solution. At tempera- Led – Lederburite
ture 1392°C delta iron transforms to a new allotropic form
Iron-carbon equilibrium diagram is obtained on plotting
gamma iron The next step is at 910°C where Gamma iron
the critical points of different iron-carbon combinations.
transform to Beta iron. At the last (4th) step at 768°C Beta
Carbon in equilibrium does not appear as free carbon
iron is transformed to Alpha iron. Gamma iron is of a FCC
(graphite) but appear in the form of iron carbide (Fe3C) con-
structure and other forms are of BCC structure. Alpha iron
taining 6.67% carbon on the right. Cementite is a chemical
acquires pronounced ferromagnetic properties Beta and
compound of iron and carbon and may form up on rapid
other forms are para magnetic.
cooling of iron-carbon melt from high temperature. It is
Since space lattice does not change in the Alpha to Beta
brittle, weak in tension, strong in compression and is the
transformation beta iron is regarded as paramagnetic state
hardest of any material in the equilibrium diagram. It will
of alpha iron. Thus there are only two main allotropic form
be observed from the figure that carbon is soluble in µ - iron
– Alpha iron and Gamma – iron.
to a maximum of 0.025% at 723°C (point P). At room tem-
Critical Points perature solubility is only 0.008%. Alpha iron is commonly
The points where structural changes occur are known as called ferrite or more accurately Alpha–ferrite. It is the
critical points. On a cooling curve it is represented by Ar4, softest.
Ar3 and Ar2. at temperature 1392°C, 910°C and 768°C The temperature 723°C is called curie temperature.
respectively. On a heating curve these points are represented Below curie temperature steel acquires ferromagnetic
by Ac4, Ac3 and Ac2 properties.
Absolutely pure iron is very difficult to obtain. In this
Critical Temperatures
state it is a soft and very plastic material of not much use
in engineering application. But it can be alloyed with many A1 is the temperature at which Pearlite changes to Austenite.
elements. Alloys of iron and carbon are most widely used This transformation occurs at a fixed temperature of 723°C
in engineering. irrespective of the composition of alloy. A1 represents the
lower critical temperature.
Iron–Carbon Equilibrium Diagram or A3 is the temperature at which last traces of ferrite
Phase Diagram changes to Austenite and the alloy becomes 100% austenite.
A3 vary from 723°C to 912°C. Depending up on the carbon
A δ+L content. This represents the upper critical temperature.
1539 B 1492° Acm is the temperature at which last traces of cementite
δ K Liquidus
Liquid (L) D changes to austenite and the alloy become completely aus-
1400 H tenite. Depending up on Carbon content this temperature
solidus varies from 723°C to 1130­°C. This also is upper critical
r+L
Austenite (r) temperature. During cooling Acm denotes the temperature
1130° L+Ct+
at which iron carbide is first rejected from austenite instead
J C E of ferrite.
912 G ACM r+ct+Led
A3 Referring to the iron-carbon phase diagram, the solidifi-
r+Ct cation, of the liquid iron and carbon melt begins along the
∝+r 723°
723
P
liquidus ABCD. Along AB the crystals of solid solutions of
S A1
Cementite (Ct)

carbon in delta iron are separated from the liquid. Along the
Lederbinte (Led)

Ferrite P+Ct+ line BC crystal of austenite are separated. Complete solidi-


Temp°C

(∝) P+Ct Led fication of these alloys takes place along the solidus line
Pearlite (P)

∝+P HKJCE. All the liquid is completely solidified at 1130°C.


At the same time crystals of austenite containing 2% car-
O 0.025 bon and cementite containing 6.67% carbon get separated
0.80 2 4.3 5 6.67
steel from it. The solidification of alloys containing 4.3% to
Hypo Hyper Cast iron 6.67% carbon begins along the line CD with the separation
eutectoid eutectoid of ­primary cementite from the melt.
Carbon %
Eutectoid Point
Iron-carbon phase diagram
Point S is called eutectoid point. At the temperature of
L – Liquid (Molten alloy) 723°C eutectoid reaction occurs. i.e, formation of two solids
d – Delta iron occur from a single solid. Simultaneous formation of ferrite
g – Austenite (gamma iron) and cementite occurs from austenite at the temperature of
Ct – Cementite (Fe3c)
Chapter 1  •  Structure and Properties of Materials  |  3.769

723°C and composition of 0.80% carbon. Nearly 12% of TTT Curves


iron carbide and slightly more than 88% ferrite are formed
Iron-carbon equilibrium diagram, provides a complete pic-
at this condition. The intimately mixed, lamellar mixture is
ture of phase changes, corresponding microstructure and
composed of alternate layers of ferrite and cementite. This
temperature under equilibrium conditions. The effects of
microstructure is called Pearlite.
different cooling rates are not revealed in this diagram. In
The eutectoid reaction can be written as
different heat treatment process, transformation of austenite
Colling
⎯⎯⎯⎯
solid A ← ⎯⎯⎯ ⎯→ solid B + solid C depends upon the cooling rate. A time based equilibrium
Heating
diagram explains the transformation with respect to cooling
or rates. The basis of heart treatment of steel is time, tempera-
Cooling ture and transformation. Time temperature and transforma-
⎯⎯⎯⎯
Austenite ← ⎯⎯⎯ ⎯→ Ferrite + Cementite
Heating tion diagram (TTT diagram) depicts these variables.
(γ 0.8%C) (0.02%C) (6.67%C) To construct these curves a number of small specimens
of steel are heated above critical points and these are then
The alloy containing 0.8% carbon is called eutectoid steel. rapidly cooled to a number of temperatures and held at these
If carbon content is greater than 0.8% austenite transforms temperatures for different periods of time until austenite is
into Pearlite and cementite. completely decomposed. At any given time, the percent-
Maximum solubility of carbon in gamma iron is 2% age of austenite and pearlite are also noted by studying the
(point J). structure. The transformation of austenite at a constant tem-
perature starts only after an ‘incubation period’.
Eutectic Point First, curves for constant temperature are drawn with
Point C is the eutectic point at which the two constituents time as x-axis and percentage composition of pearlite and
solidifies simultaneously. It has the lowest liquidus tempera- austenites on y-axis.
ture (1130°C) and carbon content is 4.3%. In the eutectic
E3 E2 E1 O
reaction the molten alloy solidifies into a mixture of two 100
solids (ausentite and cementite), known as Lederburite. At
eutectic point only lederburite is formed. (Lederburite is g –
Fe3C eutectic matrix) At higher carbon content greater than

Austenite 0%
Pearlite %

4.3% C, Fe3C is embedded in the eutectic matrix. 50


400°C 600°C 700°C 50
The eutectic reaction can be written as
Liquid L → solid A + solid C Time 100
⎯⎯⎯⎯
or Liquid ←
43%C
⎯⎯⎯ ⎯→ Austenite (γ ) + Cementite [ Fe3C ]
1130 C O B3 B2 B1
At eutectic point liquid phase coexist with two solid phases
Iron-carbon alloys containing more than 2% carbon is Time
known as Cast iron.
Less than 4.3% carbon is hypoeutectic and more than From the above curves, the TTT curves are constructed.
4.3% and up to 6.67 they are hyper eutectic cast irons.
800 Austenite (723°C)
B1 Pearlite
Peritectic Point 700 E1 Sorbite
B2 E2
Point K is called the peritectic point. 600
Troosite
Peritectic reaction can be generally written as
500

→ solid A
Solid B + Liquid L ←
 E3
400
B3
Peritectic reactions takes place along the horizontal line at Acicular
300 Austenite
1492°C. Liquid + d iron transforms to g – iron at point K. MS (240°C) Troostite
Maximum solubility of carbon in d – iron is 0.09% but in 200 (bianite)
fcc r-iron the solubility of carbon is much greater.
100
Martensite and austenite
(20°C)
Peritectoid Point 0
MF(50°c)
Martensite
In peritectoid reaction two solid phases change to one solid −100
0.5 1 10 102 103 104 105
phase and the corresponding point is called peritectoid Time (sec)
point. TTT Diagram
3.770  |  Part III  •  Unit 7  •  Manufacturing Technology

Austenite decomposes to pearlite at near about 700°C. Types of Cast Iron


Troosite is formed at 500 to 550°C. Bianite is formed at Cast iron is the cheapest cast material. In molten state, it
550 to 200°C. Martensite transformation starts (point Ms) flows readily. Plain cast iron is strong in compression and
at 240°C. and finishes (point MF) at 50°C. These points are weak in tension and shear. It has a very good damping
not affected by cooling rate. But their positions depend upon capacity. Properties of cast iron can be varied by the addi-
chemical composition of steel. TTT diagram is also known tion of alloying elements and heat treatment. Main types of
as isothermal (IT) diagram or C-curve. or S-curve based on cast iron are
its shape.
1. Gray cast iron
2. White cast iron
Manufacturing of Cast Iron, 3. Malleable cast iron
Wrought Iron and Steel 4. Ductile cast iron
Cast iron, steel and wrought iron are manufactured from 5. Chilled cast iron
pig iron which contains 4.5 to 6.67% carbon. Pig iron is 6. Alloy cast iron
obtained from chemical reduction of iron ore in blast fur- 7. Meehanite cast iron
nace. The process is known as smelting. 8. Mottled cast iron
Cast iron which contains 2 to 6.67% carbon is produced
1.  Gray cast iron
in cupola by remelting pig iron.
It is the most widely used cast iron. It contains 2.5 to 3.75%
Commercial cast iron contains 2.5 to 4% carbon. Cast
carbon and up to 2.5% silicon.
iron is very brittle and has low ductility.
In gray cast iron, carbon is in the form of graphite flakes.
Wrought iron is a mixture of pure iron and 1-3% slag. It
Gray cast iron is soft and only moderately brittle. The
contains traces of carbon (less than 0.05%), silicon, manga-
free graphite in its structure acts as a good lubricant and
nese, sulphur and phosphorus. For production of wrought
increases the machinability.
iron pudding furnace or Aston or Byers process is used.
Wrought iron is ductile and soft and is very suitable for 2.  White cast iron
forging and forge welding. Strength of wrought iron can be In white cast iron, carbon is in the form of cementite (Fe3C)
increased by cold working or alloying with constituents like It contains carbon up to 2.5% and silicon below 1.5%.
nickel (1.5 to 3.5%) It is very hard and brittle and has very limited applications.
Steel contains iron and less than 2% carbon. Steels can
be classified as plain carbon steels, alloy steels, special alloy 3.  Malleable cast iron
steels and cast steels. Plain carbon steels contain iron and Malleable cast iron is produced by heating white cast iron
carbon and impurities like silicon, manganese, sulphur and for a prolonged period of time to a temperature of 850 to
phosphorus. 1000°C and after that by cooling at a slow rate. In this pro-
Depending upon the carbon content, plain carbon steels cess cementite decomposes to ferrite. Silicon content is kept
are classified as less than 1% to avoid graphitization. Method of production
used is called Blackheart method and the cast iron obtained
1. Low carbon or mild steel is ferritic malleable cast iron.
(0.05 to 0.3% C) In another method called white heart method pearlitic
2. Medium carbon steel malleable cast iron is produced.
(0.3 to 0.6% C)
3. High carbon steel 4.  Ductile cast iron or spheroidal cast iron
(0.6 to 1.5% C) In ductile cast iron carbon is precipitated as spherical nod-
ules of graphite. It is also called nodular cast iron. It is of
Sulphur and phosphorus are harmful impurities in steel high grade in comparison to malleable cast iron. To produce
causing brittleness. Silicon and manganese are deoxidizers ductile cast iron, small amounts of special alloys containing
and purify the steel. magnesium (Mg) or cerium (Ce) is added to desulpherised
Alloy steels contain specially introduced alloying ele- molten cast iron.
ments which modifies or enhances the properties of plain Ductile iron possesses good fluidity, high strength and
carbon steel. ductility. It is tougher and less porous than gray cast iron.
Most frequently used alloying elements are chro-
mium (Cr), Nickel(Ni), Manganese (Mn), silicon (Si), 5.  Chilled cast iron
Molybdenum (Mo), Vanadium (Va), Tungsten (W), Chilled cast iron is made by using ‘chills’ in the moulds
copper(Cu) and aluminium(Al). which promotes rapid cooling of the cast iron. On rapid cool-
Steel is produced using Bessemer process, open hearth ing of the molten metal in the mould, a hard, wear resistant
process or electric furnace. surface of 1 mm to 2 mm thickness consisting of white cast
iron is produced. The material inside is only gray cast iron.
Chapter 1  •  Structure and Properties of Materials  |  3.771

6.  Alloy cast iron Normalising


Cast iron added with alloying elements is called alloy cast In the normalized process, metal is heated to a point 40° to
iron. Commonly used alloying elements are nickel, copper, 50° above its upper critical temperature, held at that tem-
chromium, molybdenum, vanadium and boron. Alloying perature for a short duration and cooled in still air at room
­elements are added to control graphitization and to improve temperature. The purpose of normalizing is
mechanical properties such as improved resistance to
heat, corrosion, wear and brittle fracture. Castability and 1. to elimimate coarse grain structure produced during
machinability are also improved by the addition of alloying forging, rolling etc
elements. 2. to remove internal stresses
3. to improve certain mechanical and electrical prop-
7.  Meehanite cast iron erties (improves corrosion resistance since it is
It is a patended product produced by the addition of cal- cooled in still air sufficient oxidization would have
cium-silicon alloy called calcium silicide. It acts as a gra- taken placed. Further it resists corrosion during
phitizer and produces fine graphite structure with excellent application.)
mechanical properties. Gray cast iron with low silicon and
moderately low carbon content is used for the production of Spherodizing and Hardening
meehanite cast iron.
In this process steel is heated to a temperature above, criti-
8.  Mottled cast iron cal point, held at this temperature for a considerable time,
Mottled cast iron is produced by heating cast iron to red hot depending up on its thickness and then quenched (rapidly
with powdered red hematite. The product has outer layers of cooled) in water, oil or molten salt baths. For hypo euctoid
white cast iron with core of gray cast iron. Toughness of cast steels heating is done 30°C to 50°C above lower critical
iron is increased by this process. temperature.

Heat Treatment of Metals Spherodisation


Heat treatment means heating the metal to various definite It happens generally in steels with carbon percentage
temperatures, holding these for various time durations and between 0.6 and 1.2
cooling at various rates. In these controlled heating and The process of obtaining spherical grains of F3C in steel
cooling, of nature and distribution of the micro constituents, in the process of annealing is known as Spherodisation.
grain size etc changes. The main purpose of heat treatment Spherical grains offer low Coefficient of friction against the
operation is the control of properties through the alteration tool bit used to perform machining. Therefore spherodising
of structure of the metal or alloy. improves machinability.
Spherical
Various heat treatment processes Fe3C

Annealing Just below Furnace


Anealing means heating the metal to a temperature slightly 723°C Cooling
above the upper critical temperature and then cooling
slowly, usually in the furnace itself. Purpose of annealing is:
1. To refine grain structure
Purpose of hardening is to increase wear resistance and to
2. To relieve internal stresses and
make it suitable for cutting tools etc.
3. To reduce hardness. After annealing ductility, machina-
Hardening of steel is accompanied by development of
bility etc are improved.
high internal stresses and brittleness. So hardening is fol-
Generally annealing is a stress relieving and softening lowed by the process tempering. After hardening and tem-
process. pering steel acquires higher strength hardness and ductility.

Full Annealing Sorbite, Troostite, Bainite and Martensite


It removes all structural imperfections by complete recrys- Depending upon the rate of cooling austenite transforms
talisation, In this process steel is heated 50° to 60°C above various structures such as sorbite, Troostite, Bainite and
the upper critical temperature for hypo-eutectoid steel and martensite.
by the same temperature above the lower critical tempera- At very slow rate of cooling austenite transform to a
ture for hyper–eutectoid steel. Metal is held at this tempera- mixture of ferrite and cementite called Pearlite.
ture for a sufficient time so that proper structure changes On increasing the rate of cooling Sorbite is formed at
takes place. The rate of cooling varies from 30 to 300°C. about 600°C. Sorbite is fine pearlite which is harder.
3.772  |  Part III  •  Unit 7  •  Manufacturing Technology

If steel is cooled at a faster rate Troostile is formed at 3. Stepped quenching


about 500–550°C. Troostite is extremely fine Pearlite which 4. Interrupted quenching or martempering
is still harder. 5. Isothermal quenching or austempering and
If temperature is lowered to 220°C acicular (needle like) 6. Surface hardening
troostite is formed. This is known as Bainite.
In conventional hardening, quenching is done in a
If steel is cooled at a very high rate so that austenite
­single medium (Rapid cooling by immersing in a l­iquid
does not have time to decompose in to sorbite or troost-
medium is called quenching) This is the most com-
ile it is transformed in to a new acicular form called
monly used method. Other methods are used for m ­ aterials
Martensite. It is very hard and brittle (BHN 650 to 750) and
of intricate shape and when it is necessary to reduce
ferromagnetic.
distortion.
Critical rate of cooling is the minimum rate of cool-
When quenching in two media is used, first liquid is usu-
ing at which the austenite is transformed In to marten
ally water. Cooling is done in the first quench up to tempera-
site alone.
ture of 300°C to 400°C and then quickly transferred to the
Tempering second liquid(for example oil) and allowed to cool to room
temperature.
Fully hardened steel is unable to with stand impact loads. Stepped quenching is also done in two stages. The
Therefore some of the hardness and strength is sacrificed first medium has a temperature of 150°C to 300°C above
in a compromise to gain sufficient ductility and toughness. the point of formation of martensite. The part is held in
This is accomplished by tempering. this medium until the part attain the temperature of the
Tempering consists of medium through out its cross section. Usually the first
1. Heating the hardened steel to a temperature below the medium is molten salt. Then it is transferred to the sec-
critical temperature ond medium usually an oil bath, where it is cooled to room
2. Holding it at the temperation for a specified time and temperature.
3. Slow cooling
Interrupted Quenching or Martempering
As tempering is done at a lower temperature all the hardness It is a modification of a stepped quenching method. Here
is not lost. second medium is still air and allowed to cool there to
Tempering is carried out to make steel tough and to resist room temperature. Here the martensite formation takes
shock and fatigue, to remove internal stresses, to stabilize place at a lower cooling rate. Therefore internal stresses are
the structure and to decrease hardness. minimized.
Tempering is carried out in liquid such as oil, salt or lead.
Bath is heated to an adequate predetermined temperature Isothermal Quenching or Austempering
and metal is immersed in this bath for a predetermined This process also is similar to martempering but the holding
length of time. After that metal is removed and allowed to time in the salt bath above the martensitic temperature has
cool to room temperature. a longer duration. The bath temperature is 300°C to 550°C.
Longer holding time leads to isothermal transformation of
austenite to bainite. Then the steel is quenched in water to
215°C Troosite
Stand (Low temp room temperature. Here the residual stresses are substan-
free air compiring) tially reduced.
The difference between martempering and austempering
is illustrated in the figures given below.
550°C
Sorbite
(High temp Austenite
tempering) 600°C
Salt bath
(Hardness)M > (Hardness)T > (Hardness)S
(Grain size)M < (Grain size)T < Grain size)S 300°C
Martensite formation
starts
Hardening Methods Air cooling
Hardening methods consist of:
1. Conventional or continuous or quench hardening
2. Quenching in two media Time →
Chapter 1  •  Structure and Properties of Materials  |  3.773

Cyniding or Liquid Carbon Nitriding


700°C Constant temperature In this process carbon and nitrogen are added to cases of
Salt bath metals. The piece of low carbon steel immersed in a bath of
Bainite formation cynide salt (sodium cynide, sodium chloride and sodium car-
500°C
bonate in equal proportions.) of temperature 850°C to 950°C
Quenching for about 15 to 20 minutes. Then it is quenched in water or
in water
oil. Carbon monoxide and nitrogen from the cynide bath
behave as active diffusing agents in hardening the surface.

25°C Nitriding
In this method nitrogen gas is used for obtaining hard sur-
Time → face of steel. This process is commonly used for steels
Austempering
which are alloyed with chromium, molybdenum, aluminium
manganese etc. Steel articles well machined and finished
Surface hardening: It is a selective heat treatment pro- are placed in a container through which ammonia in circu-
cess in which high hardness is imparted to thin surface lated. Nilriding process is generally done in electric furnace
layer of the work. This becomes necessary for parts, sub- where temperature is maintained in the range 450 to 550°C.
jected to heavy impact and fatigue loads such as worms, Ammonia gas when passing through the steel articles
gears, spindles, Cam shafts etc. where hard and wear gets dissociated and nascent nitrogen is formed. Nascent
resistant surface and a soft tough core is necessary. Low ­nitrogen reacts with the surface of the articles forming
carbon steels having up to 0.2% carbon are subjected to nitrides which is very hard. Surface hardness up to depth of
chemical treatment process (case hardening ) such as car- 0.8 mm is obtained.
burizing, Nitriding, Cyaniding etc. Steels with higher
percentages of carbon are subjected to surface hardening Induction Hardening
methods in which only the surface layers of the work are This method is similar to flame hardening. But heating of
heated and then quenched in some medium. Two methods the surface is achieved by electromagnetic induction. The
used are work piece is enclosed in the magnetic field of an alternat-
ing current of high frequency (10 kHz to 2 MHz) current
1. Electric induction surface hardening process
conductor (Induction block) Heating effect is due to the
2. Flame hardening
induced eddy currents and hyrterisis loses in the surface of
the material. The induction block has water connections and
Carburising numerous small holes on its inside surface. As soon as sur-
A low carbon steel cannot be hardened appreciably by the face reaches proper temperature, it is automatically spray
hardening process. - quenched under pressure.
Surface of such steels are enriched by carbon before This process is very rapid and takes only few seconds to
hardening and quenching. This is achieved by carburizing heat steel to a depth of 3 mm. The time varies depending
process. upon frequency, power input, depth of hardening required etc.
The specimen along with a carbonaceous material such
as charcoal is packed in a sealed container and kept in a Flame Hardening
furnace. The pack is heated to a temperature between 800 Some times full case hardening of an article will not be
to 900°C and held for a period of 4 to 20 hours. Carbon required. Only the areas subjected to wear, abrasion or
monoxide is formed with the small amount of oxygen pre- shocks are to be hardened. Such local hardening can be
sent. Carbon of carbon monoxide is diffused in to the metal achieved by flame hardening. These areas are heated by a
surface releasing oxygen. This oxygen again reacts with oxy - acetylene fame above its critical temperature. Heated
carbon and carbon monoxide is formed and the process is portions is immediately quenched by means of spray of
repeated. As time progress metal surface gets enriched with water. Since heating is localized stresses are not developed.
carbon. This process is known as pack carburizing. The So disortiuons and cracking are reduced.
amount of carbon diffused into steel depends on the carbur-
izing temperature and time. The process of carburizing is by Spherodizing (Spherodize Annealing)
atomic diffusion. High carbon hardened steel contains grains having sharp
Another type of carburizing is gas carburizing. Gases corners, which make the steel brittle. Spherodizing is the
such as natural gas propane, or methane is used instead heat treatment process that produces a rounded or globular
of charcoal. Gas carburizing has better control of surface form of Carbide. Low carbon steels may be speroidized to
­carbon and case depth than in pack carburizing.
3.774  |  Part III  •  Unit 7  •  Manufacturing Technology

improve strength before subsequent heat treatment. High Diffusion Annealing or Homogenising
carbon steels may be spherodized to improve machinabil- In-homogeneity due to development of dentritic segrega-
ity and toughness. The process involves heating the steel tions in castings and ingots can be eliminated by diffusion
slightly below the critical temperature, holding at this tem- annealing.
perature for a period of time and then letting them cool in In this process the article is heated to a very high tem-
the furnace. Holding time depends up on the thickness of perature and held there for a prolonged period, cooled with
metal and carbon content. More the percengage of carbon, the furnace to 800°C–850°C. and finally cooled in still
longer will be the time required to complete sperodization. air. A  coarse grain structure is obtained and composition
Homogenizing becomes more uniformly. After homogenizing a full anneal-
Normalising
ing treatment is required.
Annealing
Effect of Alloying Elements in Steel
Temperature

1. Chromium (Cr)
A3
Improves resistance to corrosion and oxidation
A1
spherodizing 2. Nickel (Ni)
Improves tensile strength and toughness.
3. Manganese (Mn)
Stress releiving
Recrystallisation/ process annealing
Imparts wear resistance
4. Silicon (Si)
Time
Added as a deoxidizing agent.
Full Annealing 5. Molybdenum (Mo)
Imparts hardenability.
The process of annealing in which the metal is heated above
the upper critical temperature, is held there until the tem- 6. Vanadium (V)
perature of work piece is uniform through out, and finally Improves grain distribution.
cooling at a slowly controlled rate so that temperature of the 7. Tungsten (W)
surface and that of the centre of the work piece is approxi- Imparts red hardness
mately same is called full annealing. 8. Cobalt (Co)
The process wiped out all traces of previous struc- Increases hot hardness
ture, refines the crystalline structure, soften the metal and 9. Copper (Cu)
relieves the previously set up internal stresses. Lowers critical temperatures
Process Annealing or Recrystallisation 10. Aluminium (Al)
Annealing Deoxidises efficiently
Involves heating the steel to recrystallisation temperature, 11. Titanium (Ti)
holding at this temperature for a prolonged period and slow Increases abrasion resistance.
cooling. 12. Phosphorous (P)
(Recrystallisation temperature = 0.4 × melting point of Enhances machinability
the metal)
Work hardened steels can be softened by this method. Composition and Applications of
Some Alloy Steels
Iso Thermal Annealing Alloy steel Composition percentage Application
When ordinary annealing process is incapable of reducing High carbon C 0.8 to 1.3 Milling cutters,
hardness of an alloy steel to a sufficient degree, isothermal tool steel Si 0.1 to 0.4 turning tools
annealing is employed. Here heating and cooling is done in Mn 0.15 to 0.4
two steps. First the steel is heated above critical tempera- rest Fe
ture and held for sometime at the temperature. Austenite is Low alloy C 1.2, Mn 0.3 to 0.7 For high
carbon steel W 2.75, Mo 0.4 temperature
formed at this stage. Then steel is rapidly cooled to 50° to
S 0.3, rest Fe operation
100°C below point A1 and held at this temperature until
18-4-1 W 18, Cr 4, High speed
austenite is completely decomposed to form pearlite. Then High speed V1 milling, drilling
steel is cooled in still air. Decomposition of austenite takes steel (HSS) C 0.6 to 0.8 and turning
place at constant temperature. So it is called isothermal Rest Fe tools
annealing.
Chapter 1  •  Structure and Properties of Materials  |  3.775

Composition and Application of Some Metals are elements atoms of which give up electrons and
Non-ferrous Alloys form metallic bonds. Metals conduct electricity. When two
or more metals are melted together they form an alloy.
The property of an alloy is quite different from that of the
Alloys Composition percentage Application
constituent metals.
Copper Metals may be classified as ferrous metals and non-­
Alloys ferrous metals. Iron and alloys which contain iron are called
Yellow brass Cu 60, Zn 40 For hot work- ferrous metals. Cast iron, wrought iron, carbon steels, alloy
(Muntz metal) ing by rolling, steels are examples of ferrous metals.
extrusion and
stamping
Metals and alloys which does not contain iron(Fe) are
called non ferrous metals. Copper, aluminium, zinc, lead,
Leaded brass Cu 62.5, Zn36 For plates,
Pb 1.5 tubes etc tin and alloys brass, bronze, duralumin, g­ unmetal, solder etc
and bearing metals (zinc base alloys and nickel base alloys)
Phosphor Cu 87 - 90, For bearings,
bronze Sn 9 - 10, worm wheel, are examples of non-ferrous metals.
P 0.1 - 0.3 gears etc
Ceramics
Gun metal Cu 88 For boiler fit-
Sn 10, Zn 2 tings bushes, Ceramic materials are products which are made from inor-
bearings etc ganic materials and have non-metallic properties. Most of
Aluminium them are oxides. Examples are silica, Al2­O3, MgO, silicon
alloys carbide, boron nitride, ferrites etc. Traditional ceramic
Duralumin Cu 3 - 4.5 Automobile materials are stone, brick, concrete, clay, glass, vitreous
Mn 0.4 - 0.7 and air craft enamel and refractories. Ceramics may be classified as
Mg 0.4 - 0.7 components
Rest Al 1. Amorphous ceramics
Y-alloy Cu 3.5 - 4.5 Aircraft engines
2. Crystalline ceramics
(Copper Mn 1.2 - 1.7 for cylinder 3. Bonded ceramics
Aluminium Ni 1.8 - 2.3 heads and 4. Cements
alloy) Si, Mg, Fe 0.6 each piston
rest Al Amorphous ceramics include glasses (soda-glass, Lead
Magnessium Al 7.5, Zn 2, Mn 0.3, For air craft
glass and borosilicate glass). Glasses are non-crystalline
alloy Rest Mg and automobile silicates containing oxides such as CaO, Na2O, K2O and
parts Al2O3.
Common non-ferrous alloys Glasses are used for electrical insulations, optical lenses
Invar Ni 36, Fe 64 Precision and bond metals (hermetic steels).
measuring Crystalline ceramics are used for abrasive and cutting
instruments tool materials as they are very hard compared to other
Nichrom Ni 80, Cr 20 Thermocouple, materials.
strain guage In bonded ceramics individual crystals are bonded
Maganin Cu 87, Mn 13 Resistor strain together by vitreous matrix. Examples are clay products and
guage refractory materials. Their application is in refractory tiles,
Constantan Cu 60, Ni 40 Electrically furnace linings, electrical insulations, sanitary equipment
heated etc.
appliances Cement is an agglomerated material. It is mainly used for
Bearing materials construction works. It is manufactured from clay, limestone,
Tin babbit Sn 80, Sb 12, Cu 8 High speed oxides of silicon, aluminium, iron and often magnesium.
bearing bushes
Lead babbit Pb 85, Sb 10, Sn 5 Railway wagon Organic Materials
bearing
Organic materials are derived directly from carbon. Carbon
is chemically combined with hydrogen, oxygen or any
Classification of Engineering Materials other non-metallic substance. Organic materials may be
Engineering materials can be classified into the following natural or synthetic. Natural organic materials consists of
wood, cotton, natural rubber, coal, petroleum, food prod-
1. Metals ucts etc. Synthetic organic materials consists of synthetic
2. Ceramics rubber, plastics, lubricants, soap oils, synthetic fibres etc.
3. Organic materials It is observed that organic materials are made of very large
molecules.
3.776  |  Part III  •  Unit 7  •  Manufacturing Technology

Organic compounds consists of molecules formed by The Seleroscope work on the principle that rebound
carbon atoms along with other non-metal atoms such as height a ball made to fall on the specimen depends on the
hydrogen, nitrogen, oxygen, chlorides etc. Some of the hardness of the specimen.
important organic compounds are
1. Saturated organic compounds
Moh’s Scale of Hardness
2. Unsaturated organic compounds In Moh’s scale of hardness different hardness numbers
3. Aromatic organic compounds are assigned to ten different minerals of varying hardness.
Starting from Talc–1 to diamond–10

Measurement of Hardness
Hardness is generally measured by the intendation made by Solved Examples
a harder material. The intendation depends upon the applied Example 1:  Compare lattice parameters of FCC and BCC
load, sharpness of the intended and the time for which structure.
applied load is maintained.
Solution:  Lattice parameter is the side length of a unit cell:
This can be expressed in terms of radius of the atom.
Brinell Hardness Test For FCC structure
In brinnel hardness test, a sphere (usually of 10+ 0.01 mm
diameter.) made of steel or tungsten carbide is used as the r
intender. Brinnel Hardness number is given by
2P
BHN = 2r
π D D - D2 - d 2 ⎤
⎡ • a
⎣ ⎦

Where P = Applied load in kg


r
D = diameter of ball in mm
d = diameter of intendation in mm. a
Brinnet Hardness of steel is roughly related to ­tensile
a2 + a2 = (4r)2
strength of the material. The empirical relationship is
given as 2a2 = 16 r2
Tensile strength (MPa) = 3.55 × BHN
4r
When BHN ≤ 175 a=
Tensile strength (MPa) = 3.38 × BHN > 175 2
For BCC structure
Rockwell Test G H
In the Rockwell test, a spheroconical diamond cone of 120°
D C
angle and a spherical apex 0.2 mm radius is used.
Rockwell number (R) =100 – 500 t.
Where t = depth of intendation
a
Depending upon load a number of scales A, B, C etc are
available.
E
Vicker’s and Knoop F
a
A
Hardness Test a B
In this case a square box pyramid diamond intender having ( AH )2 = ( AE )2 + (HE)2
136° between opposite forces is used.
Vickers Hardness number (VHN) (4r)2 = 2a2 +a2
= 3a2
1.854 L
= 3
D2 r= a
4
where L = Applied load in Kg
D = measured average diagonal of the intendation in 4r
or a =
mm. 3
Chapter 1  •  Structure and Properties of Materials  |  3.777

Example 2:  Calculate atomic packing factor for FCC and 2P


BCC. BHN =
π D( D - D 2 - d 2 )
Solution:  Atomic Packing Factor
Volume of atoms per unit cell 2 × 500
APF =
volume of unit cell ⎡
π × 10 10 - 10 2 - 2.52 ⎤
⎣ ⎦
No of atom/unit cell for FCC
= 100.24.
8 6
= + =4 Example 4:  0.6% Carbon steel was cooled from a temper-
8 2 ature of 1500°C. to room temperature. Discuss the cooling
each corner atoms are shared bu 8 unit cells and each face process.
centered atoms are shared by 2 unit cells.
Solution:  Steel containing 0.6% carbon is in Liquid +Aus-
4
4 × π r3 tenite faustenite. Below 72.3°C, a mixture of ferrite phase
APF for FCC = 3 at 1500°C. Due to cooling it changes to ferrite and pearlite
93 is obtained.
4 Example 5:  You are given two pieces of steel samples.
4 × π r3
= 3 (i)  0.2%Carbon steel.
3 (ii)  0.8% Carbon steel.
⎛ 4r ⎞
⎜⎝ ⎟ Both are heated to austenite region and then quenched
2⎠
in water at room temperature. Describe the changes in
42 π each case.
= ×2 2
3 × 43 Solution:
π 2 (i)  0.2%Carbon steel
= = 0.74
6 If 0.2% Carbon steel is quenched in water from austenite
region. We get the martensite consisting of domains of
4 lath of different but limited orientations through a whole
2 × π r3
APF for BCC = 3 domain. This structure is slightly distorted consisting of
3
⎛ 4r ⎞ regions with high densities of dislocation tangles, called
⎜⎝ ⎟⎠ lath martensite.
3
(ii)  0.8% Carbon steel
3
= π = 0.68. When 0.8% carbon steel is quenched in water we get
8 mixture of both lath and plate type martensites.
Example 3:  Hardness of a steel sample is measured taking Example 6:  Discuss the cooling process of 0.6% carbon
Brinnel identation. 10 mm Tungsten carbide ball at 500 Kg steel from 1500°C to room temperature.
f load was used. If the average diameter of identation is Solution:  During cooling, Liquid + Austenite at 1500°C
2.5 mm, Brinnel Hardness number is -------- is converted to complete austenite at 1440°C. After 750°C
Solution:  D = 10 mm it changes into ferrite + austenite which after 723°C. after
D = 2.5 mm 723°C temperature becomes mixture of pearlite and ferrite.
P = 500 Kgf.

Exercises
Practice Problems 1
1. A body which is permanently deformed is said to have 2. Hardness is the property of material due to which it
under gone (A) Can be drawn into wires
(A) Elastic deformation (B) Will break with little permanent distortion
(B) Plastic deformation (C) Can cut another metal
(C) Limit of elastic deformation. (D) Can be rolled or hammered into thin
(D) Uniform deformation sheets
3.778  |  Part III  •  Unit 7  •  Manufacturing Technology

3. Failure of a material due to fatigue occurs 11. In sperodising process, the steel is
(A) At elastic limit (A)  Heated above upper critical temperature and
(B) Below the elastic limit cooled in still air
(C) At the yield point (B) Heated up to lower critical temperature and cooled
(D) Below the yield point. in still air
4. For the materials of tools and machines, the important (C) Heated slightly above the lower critical tempera-
material property is ture and cooled in the furnace.
(A) Elasticity (D)  Heated below lower critical temperature and
(B) Plasticity quenched in water.
(C) Ductility 12. The process in which steel is heated to a temperature
(D) Malleability above upper critical temperature and then suddenly
5. No of atoms per unit cell of a face centered cubic (FCC) cooled by quenching in a salt bath maintained at 250° –
lattice is 525°C, is called
(A) 14 (A) Normalizing (B) Tempering
(B) 7 (C) Austempering (D) Martempering
(C) 4 13. In the austempering heat treatment process austenite
(D) 10 changes to
(A) Martensite (B) Troosite
6. Carbon steel containing 0.8% carbon is known as
(C) Sorbite (D) Bainite
(A) Eutectic steel
(B) Hypoeutectic steel 14. The heat treatment process for softening hardened steel
(C) Eutectoid steel is
(D) Hypereutotoid steel (A) Carburizing (B) Normalizing
(C) Annealing (D) Tempering
7. According to IS specification a plain carbon streel
designated by 40C8 means that 15. The hardening process in which carbon and nitrogen
(A)  Carbon content 0.4 percent and Manganese are absorbed by the surface of the metal is called
content 0.8 percent. (A) Carburising (B) Nitriding
(B)  Carbon content 0.4 percent and Manganese (C) Cyaniding (D) Flame hardening
content 0.8 percent. 16. Delta iron occurs in this temperature range
(C) Carbon content 0.35 to 0.45 percent and Manga- (A) 1400°C to 1530°C (B) 723 to 910°C
nese content 0.6 to 0.9 percent. (C) 910 to 1400°C (D) 400 to 723°C
(D) Carbon content 0.8 percent and Manganese con- 17. An allotropic material has
tent 0.4 percent. (A) Different crystal structure at different temperature
8. Percentage of carbon content in wrought Iron is (B) Same structure at all temperature
about (C) Atoms distributed in random pattern.
(A) 0.02 (D) More than three types of crystal structures.
(B) 0.01 18. The most ductile maternal of the following is
(C) 0.2 (A) FCC (B) BCC
(D) 0.4 (C) HCP (D) All of the above.
9. Mallealbe Cast Iron is made 19. “Killed steels” are those steels which
(A) By adding magnesium to molten cast iron (A) Destroyed by burning
(B) By quick cooling of molten cast iron (B) Contains burnt carbon
(C) From white cast iron by annealing (C) After their destruction are recycled to produce
process fresh steel
(D) None of the above (D) Are deoxidised in the Laddle with silicon and alu-
10. When austenite steel is air cooled, the structure minium.
produced will be 20. Carbon in Iron is an example of
(A) Martensite (A) Interstitial solution
(B) Fine pearlite (B) Substitutional solution
(C) Coarse pealite (C) Intermetallic compound
(D) Troostite (D) All of the above
Chapter 1  •  Structure and Properties of Materials  |  3.779

Practice Problems 2 11. The material with good ductility is


(A) Mild steel (B) Aluminium
1. Lattice parameter of metal of FCC lattice having atomic
(C) Cast iron (D) Bronze
radius. 0.144 nm is
(A) 0.14 nm (B) 0.407 nm 12. Hardest of the following is
(C) 0.333 nm (D) 0.567 nm (A) Coarse pearlite (B) Troostite
(C) Bainite (D) Sorbite
2. Metal with HCP structure is
(A) Silver (B) Iron 13. Carbon percentage in 20 Cr 18 Ni2 steel is
(C) Magnesium (D) Aluminium (A) 0.02% (B) 2%
3. The temperature at which ferromagnetic material (C) 0.2% (D) Nil
becomes paramagnetic is called 14. Crystal structure of brass is
(A) Curie temperature (A) BCC (B) FCC
(B) Critical temperature (C) HCP (D) Orthorombic
(C) Inversion temperature 15. Stainless steel contain
(D) Magnetic temperature (A) Iron, chromium and nickel
4. The higher packing of atoms structure is (B) Chromium and nickel
(A) HCP (B) FCC (C) Chromium, nickel iron and carbon
(C) BCC (D) Simple cubic (D) Tungsten and Chromium
5. Basic atoms for HCP space lattice are 16. 18-4-1 High speed steel contain
(A) 2 (B) 3 (A) Vanadium 4%, Chromium 18% and tungsten 1%
(C) 4 (D) 6 (B) Vanadium 1% Chromium 4% and tungsten 18%
6. The percentage of chromium in 18 – 4 – 1 HSS is (C) Vanadium 18%, Chromium 1% and
(A) 1% (B) 4% tungsten 4%
(C) 18% (D) 0.4% (D) none of the above
7. Hardest of the following is 17. In full annealing, the hypo-euctoid steel is heated from
(A) Spherodite (B) Martensite 30°C to 50°C above upper critical temperature and then
(C) Sorbite (D) Troostite cooled
8. Hardness of steel improves in the process (A) In still air
(A) Annealing (B) Cyniding (B) Slowly in the furnace
(C) Normalising (D) Tempering (C) Suddenly in a cooling medium
(D) Any one other above
9. Choose the most appropriate set of heat treatment pro-
cess and corresponding process characteristics. 18. Which of the following is a case hardening process
P – Tempering (A) Carburizing (B) Cyniding
Q – Austempering (C) Nitriding (D) All of these
R – Martempering 19. In a material if elastic properties are different in differ-
1. Austenite → Bianite ent direction s. The material is
2. Austenite → Martensite (A) Anisotropic (B) Heterogeneous
3. Cementite → Globular structure (C) Isentropic (D) Isotropic
4. Hardness and brittleness reduced. 20. The property of material in which slow extension takes
(A) P -4, Q-2, R-1 place with time at constant load is known as
(B) P-4, Q-3, R-1 (A) Elasticity (B) Creep
(C) P-4, Q-1, R-2 (C) Ductility (D) Plasticity
(D) P-2, Q-3, R-4
21. For parts subjected to shock and impact loads, the most
10. For two specimens A & B of identical size, Young’s desirable property is
modulus of specimen A is greater than that of speci- (A) Elasticity (B) Plasticity
men B. (C) Toughness (D) Malleability
This means
(A) A is stiffer than B 22. The property of a material essential for spring material
(B) B is stiffer than A is
(C) A is harden than B (A) Stiffness (B) Ductility
(D) B is harden than A (C) Resilience (D) Plasticity
3.780  |  Part III  •  Unit 7  •  Manufacturing Technology

23. The property of a material essential for spring 27. During normalizing process of steel, the specimen is
material is heated
(A) Stiffness (A) Above the upper critical temperature and cooled in
(B) Ductility still air.
(C) Malleability (B) Below the upper critical temperature and cooled in
(C) Plasticity still air
24. Endurance limit is the maximum value of stress a spec- (C) Below the lower critical temperasture and cooled
imen can withstand without failure, for infinite number in still air.
of cycles where subjected to (D) Above the upper critical temperature and cooled in
(A) Dynamic load the furnace.
(B) Completely reversed, repeated loads 28. Percentage of Carbon in grey cast iron is
(C) Static load (A) 0.25 – 0.75
(D) Static as well as dynamic load (B) 1.25 – 1.75
25. Metal with 60% copper and 40% Zinc is called (C) 3 – 4
(A) Gun metal (D) 8 – 10
(B) Muntz metal 29. High carbon steels contains carbon percentage of
(C) Monel (A) 0.1 to 0.3
(D) Admiralty brass (B) 0.3 to 0.6
26. Main purpose of spheroidizing treatment is to (C) 0.6 to 0.8
improve (D) 0.8 to 1.5
(A) Hardenability of low carbon steels 30. Tap, dies and drills contain carbon of the order of
(B) Machinability of low carbon steels (A) Below 0.5%
(C) Hardenality of high carbon steels (B) Below 1%
(D) Machinability of high carbon steels (C) Above 1%
(D) Above 2%

Previous Years Qeustions


1. The percentage of carbon in gray cast iron is in range 3. When the temperature of a solid metal increases.
of [2004]  [2005]
(A) 0.25 to 0.75 percent (A)  Strength of the metal decreases but ductility
(B) 1.25 to 1.75 percent increases
(C) 3 to 4 percent (B) Both strength and ductility of the metal decrease
(D) 8 to 10 percent (C) Both strength and ductility of the metal increase
2. From the lists given below, choose the most appropri- (D)  Strength of the metal increases but ductility
ate set of heat treatment process and the correspond- decreases
ing process characteristics:  [2004] 4. The main purpose of spheroidising treatment is
Process Characteristics improve [2006]
P. Tempering 1. Austenite is
(A) Hardenability of low carbon steels
converted into (B) Machinability of low carbon steels
bainite (C) Hardenability of high carbon steels
Q. Austempering 2. Austenite is (D) Machinability of high carbon steels
converted into
martensite
5. Match the items in Columns I and II. [2006]
R. Martempering 3. Cementite Column I Column II
is converted (P) Charpy test (1) Fluidity
into globular (Q) Knoop test (2) Microhardness
structure
(R) Spiral test (3) Formability
4. Both hardness
and Brittleness (S) Cupping test (4) Toughness
are reduced (5) Permeability
5. Carbon is
absorbed into (A) P–4Q–5R–3S–2
the metal (B) P–3Q–5R–1S–4
(A) P-3 Q-1 R-5 (B) P-4 Q-3 R-2 (C) P–2Q–4R–3S–5
(C) P-4 Q-1 R-2 (D) P-1 Q-5 R-4 (D) P–4Q–2R–1S–3
Chapter 1  •  Structure and Properties of Materials  |  3.781

6. If a particular Fe-C alloy contains less than 0.83% 14. The relationship between true strain (eT) and engi-
carbon, it is called [2007] neering strain (eE) in a uniaxial tension test is given as
(A) High speed steel  [2014]
(B) Hypoeutectoid steel (A) eE = In (1+ eT)
(C) Hypereutectoid steel (B) eE = In (1 – eT)
(D) Cast iron (C) eT = In (1+ eE)
7. The effective number of lattice points in the unit cell (D) eT = In (1 – eE)
of simple cubic, body centered cubic, and face cen- 15. The stress-strain curve for mild steel is shown in the
tered cubic space lattices, respectively, are [2009] figure given below. Choose the correct option refer-
(A) 1, 2, 2 ring to both figure and table. [2014]
(B) 1, 2, 4
(C) 2, 3, 4 T
(D) 2, 4, 4
U

Stress δ(N/mnz)
8. The material property which depends only on the R
basic crystal structure is [2010] Q
(A) Fatigue strength
P S
(B) Work hardening
(C) Fracture strength
Strain e (%)
(D) Elastic constant
9. The crystal structure of austenite is [2011] Point on the Graph Description of the Point
(A) Body centered cubic P 1. Upper Yield Point
(B) Face centered cubic Q 2. Ultimate Tensile Strength
(C) Hexagonal closely packed
R 3. Proportionality Limit
(D) Body centered tetragonal
S 4. Elastic Limit
10. During normalizing process of steel, the specimen is
T 5. Lower Yield Point
heated [2012]
U 6. Failure
(A) Between the upper and lower critical temperature
and cooled in still air. (A) P-1,Q-2,R-3,S-4,T-5,U-6
(B) Above the upper critical temperature and cooled (B) P-3,Q-1,R-4,S-2,T-6,U-5
in furnace. (C) P-3,Q-4,R-1,S-5,T-2,U-6
(C) Above the upper critical temperature and cooled (D) P-4,Q-1,R-5,S-2,T-3,U-6
in furnace.
(D) Between the upper and lower critical temperature 16. Match the heat treatment processes (Group A) and
and cooled in furnace. their associated effects on properties (Group B) of
medium carbon steel. [2014]
11. For a ductile material, toughness is a measure of
 [2013] Group A Group B
(A) Resistance to scratching P: Tempering I: Strengthening and grain
(B) Ability to absorb energy up to fracture refinement
(C) Ability to absorb energy till elastic limit Q: Quenching II:  Inducing toughness
(D) Resistance to indentation
R: Annealing III: Hardening
12. The process of reheating the martensitic steel to S: Normalizing IV: Softening
reduce its brittleness without any significant loss in
its hardness is [2014] (A) P - III, Q - IV, R - II, S-I
(A) Normalising (B) Annealing (B) P - II, Q - III, R - IV, S-I
(C) Quenching (D) Tempering (C) P - III, Q - II, R - IV, S-I
13. A metal of initial length L0 is subjected to drawing (D) P - II, Q - III, R - I, S-IV
process. The length of the rod at any instant is given 1 7. Which one of the following types of stress-strain rela-
by the expression, L(t) = L0 (1 + t2), where t is the time tionship best described the behavior of brittle materials,
in minutes. The true strain rate (in min –1) at the end such as ceramics and thermosetting plastics, (σ = stress
of one minute is ____ [2014] and ε = strain)? [2015]
3.782  |  Part III  •  Unit 7  •  Manufacturing Technology

The toughness of the material (in MJ/m3) is ______.


 [2016]
(A)  σ (B)  σ 22. In the phase diagram shown in the figure, four sam-
ples of the same composition are heated to tempera-
ε    ε tures marked by a, b, c and d.

700

650 L
(C)  (D)  d

Temperature (Arbitrary)
σ σ
α+L
600 cα
ε    ε

550 b
18. The atomic packing factor for a material with body
α+β
centered cubic structure is ______. [2015]
500 a
19. The flow stress (in MPa) of a material is given by
σ = 500 ∈0.1, 450
where ε is true strain. The Young’s modulus of elastic- 0 5 10 15
ity of the material is 200 GPa. A block of thickness Composition (Arbitrary)
100 mm made of this material is compressed to 95
mm thickness and then the load is removed. The final At which temperature will a sample get solutionized
dimension of the block (in mm) is ______. [2015] the fastest? [2016]
20. The ‘Jominy test’ is used to find [2016] (A) a (B) b
(A) Young’s modulus (C) c (D) d
(B) hardenability 23. The value of true strain produced in compressing a
(C) yield strength cylinder to half its original length is: [2016]
(D) thermal conductivity (A) 0.69 (B) –0.69
21. A hypothetical engineering stress-strain curve shown (C) 0.5 (D) –0.5
in the figure has three straight lines PQ, QR, RS with 24. In a binary system of A and B, a liquid of 20% A (80%
coordinates P(0, 0), Q(0.2, 100), R(0.6, 100), R(0.6, B) is coexisting with a solid of 70% A (30% B). For
140) and S(0.8, 130). ‘Q’ is the yield point, ‘R’ is the an overall composition having 40% A, the fraction of
UTS point and ‘S’ the fracture point. solid is: [2016]
(A) 0.40 (B) 0.50
160
(0.6, 140) (C) 0.60 (D) 0.75
140 (0.8, 130)
R 25. Engineering strain of a mild steel sample is recorded
Engg. Stress (MPa)

120 S as 0.100%. The true strain is: [2016]


(0.2, 100)
100 (A) 0.010% (B) 0.055%
Q
80 (C) 0.099% (D) 0.101%
60
40
P
20
(0, 0)
0
0 0.2 0.4 0.6 0.8 1
Engg. Strain (%)
Chapter 1  •  Structure and Properties of Materials  |  3.783

Answer Keys
Exercises
Practice Problems 1
1. B 2. C 3. D 4. A 5. C 6. C 7. C 8. A 9. C 10. B
11. C 12. C 13. D 14. D 15. C 16. A 17. A 18. A 19. D 20. A

Practice Problems 1I
1. B 2. C 3. A 4. B 5. D 6. B 7. B 8. B 9. C 10. A
11. A 12. C 13. C 14. B 15. C 16. B 17. B 18. D 19. A 20. B
21. C 22. C 23. A 24. B 25. B 26. D 27. A 28. A 29. D 30. C

Previous Year’s Questions


1. C 2. C 3. A 4. D 5. D 6. B 7. B 8. D 9. B 10. C
11. B 12. A 13.  0.9 to 1.1 14. C 15. C 16. B 17. D 18.  0.66 to 0.69
19.  95.14 to 95.20 20. B 21. 0.85 22. C 23. B 24. A 25. C
Chapter 2
Metal Casting and Forming
chapter highlights

☞☞ Metal Casting ☞☞ Melting and Pouring


☞☞ Pattern Making ☞☞ Runner Extension
☞☞ Pattern Allowances ☞☞ Riser
☞☞ Shrinkage Allowance ☞☞ Gating System Design
☞☞ Machining or Finishing Allowance ☞☞ Chvorinov’s Rule
☞☞ Draft or Taper Allowance ☞☞ Modulus Method
☞☞ Shaking or Rapping Allowance ☞☞ Cooling and Solidification
☞☞ Classification of Casting Process ☞☞ Moulding Material Defects
☞☞ Coreprints and Coreseats ☞☞ Metallurgical Defects
☞☞ Chaplets ☞☞ Roll Passes
☞☞ Moulding Materials ☞☞ Forging
☞☞ Properties of Moulding Sand ☞☞ Extrusion
☞☞ Moulding Processes

Metal Casting Each material has its own advantages and limitations. The
required accuracy, strength and life of a pattern depend on
Metal casting is the process of pouring a molten metal in to a
the quantity of casting to be produced.
mould and allowing it to solidify to produce a desired object.
Generally for short run production wood is used as pat-
Most commonly used method of metal casting is sand
tern material. For mass production metal patterns are used.
casting in which a sand mould is used. It is a very old
Plastics may be used for batch production.
method. Improved methods such as shell moulding, invest-
ment casting, die casting, centrifugal casting etc. now find Pattern Allowances
wider application. Various steps in casting are
A pattern is generally slightly larger in size compared to the
1. Pattern making casting due to the various allowances to be provided.
2. Mould and core making Various allowances are
3. Melting and pouring
4. Fettling after solidification 1. Shrinkage or contraction allowance
5. Inspection and testing 2. Machining or finishing allowance
3. Draft or taper allowance
Pattern Making 4. Shaking or rapping allowance
5. Distortion or camber allowance
Pattern is the replica or full size model of the casting to be
made. It is used to give shape to the mould cavity where the
poured molten metal solidifies to the desired form and size. Shrinkage Allowance
Pattern is generally made using the following materials. Almost all metals shrink or contract on solidification. So
1. Wood to compensate the reduction in size of the casting, size of
2. Metals pattern is to be increased. The allowance provided for this is
3. Plastics called shrinkage allowance. Following is the typical values
4. Plasters of shrinkage allowances for some of the commonly used
5. Wax metals
Chapter 2  •  Metal Casting and Forming  |  3.785

Cart iron/ 6. Gated pattern


Malleable iron : 10 mm/m 7. Sweep pattern
Brass, Cu, Al : 15 mm/m 8. Segmental pattern
Steel : 20 mm/m 9. Skelton pattern
Zn, Lead : 25 mm/m 10. Built up pattern
11. Follow board pattern
A ‘shrink scale’ is a pattern maker’s rule using which pat-
tern dimensions, are laid out. Shrink scale is longer than a Solid or single piece pattern is used either in the drag or in
standard scale by the shrinkage value for the appropriate the cope. It is generally used for large castings of simple
metal. shape.
All castings, for example spherical shaped, cannot be
Machining or Finishing Allowance mould using single piece patterns. So split patterns are
To remove surface roughness and other imperfections and used. Split patterns are made in two parts. So that one part
to achieve exact casting dimensions, a casting has to be will come in the dag and the other part will come in the
machined. Therefore to account for this the pattern is to be cope. This facilitates easily removal of the pattern from the
made sufficiently larger. The allowance provided for this is mould without damage to the mould.
called machining or finishing allowance. In loose piece patterns projections or overhanging parts
are made as loose pieces which are attached to the main part
Draft or Taper Allowance using wooden dowel pins. After making the mould, the main
part of pattern is withdrawn first and then the loose pieces.
Draft is the taper provided on the vertical surfaces of pattern
Cope and drag pattern is another type of split pattern.
for the easy removal of the pattern from the mould with out
The split halves of the pattern are mounted on different
damaging the edges of the mould around the pattern. Draft
plates. The two halves are made separately and then assem-
is expressed in mm per meter on a side or in degrees. The
bled. These are used for very large castings.
amount of taper depends on method of moulding, shape and
Match plate patterns are mostly used in machine mould-
size of pattern and the moulding material.
ing and for producing large number of small castings by
Shaking or Rapping Allowance hand moulding. The pattern halves with gating and run-
ner systems are mounted on opposite sides of one single
The pattern is to be shaked or rapped for easy withdrawal wooden or metal plate called match plate.
from the mould. The cavity of the mould is slightly increased Gated pattern is one or more loose patterns having
by the shaking action. To compensate this the pattern is attached gates and runners. Time required to cut gating sys-
made slightly smaller than the required size. tem by hand is eliminated. These are used for mass produc-
tion of small castings.
Distortion or Camber Allowance
Sweep patterns are used for symmetrically shaped large
A casting will distort or warp during cooling due to uneven castings. A section or wooden board of proper contour is
shrinkages due to irregular shape, uneven metal thickness rotated about one edge so that a cavity is made in the mould-
or difference in exposures of the metal surfaces. To allow ing material such as loam sand with the desired contour.
for this, the shape of the pattern is modified in such a way it Segmental patterns are also known as part patterns.
bends in the opposite direction of distortion. These are used for producing large circular castings such as
rings, gears etc. The part pattern is moved about the centre.
Types of Patterns After ramming one section it is moved to the adjacent sec-
The type of a pattern to be used depends up on the follow- tion and this is continued to complete the mould.
ing points Skeleton patterns are used for very large castings of sim-
1. Quantity of casting to be produced ple geometrical shapes. If no. of castings required is small
2. Size and complexity of the shape of casting to be it is not economical to construct solid patterns. Pattern is
produced made of wooden frame and ribs. The frame work is filled
3. Moulding method used with loam sand and rammed. A strike off board known as
4. Surface finish and accuracy required etc strickle board is used to give the pattern the desired shape.
Built up patterns are composed of two or more segments
The most common type of patterns are made by cutting strips of wood. Pattern for special pulleys
1. Solid or single piece pattern are built up type.
2. Split pattern Follow board pattern is a wooden board and is used for
3. Loose piece pattern supporting a pattern which is very thin and fragile. It also
4. Cope and drag pattern helps to establish a parting plane with ease in a pattern that
5. Match plate pattern has an irregular shape.
3.786  |  Part III  •  Unit 7  •  Manufacturing Technology

Stop Off = 20 mm/m


Stop offs are wooden pieces used to reinforce some por- = 0.02 mm/mm
tion of the pattern which are structurally weak. The cavities Pattern dimension/mm of casting  = 1.02 mm
formed after moulding due to the stop offs are refilled with ( for holes = 0.98 mm)
sand before pouring molten metal.
Casting dimension Pattern dimension

Pattern Colour Codes 100 mm 100 × 1.02 = 102 mm


200 mm 200 × 1.02 = 204 mm
Colour codes are used in patterns for indicating different
surfaces. There is no universally accepted standard colour 350 mm 350 × 1.02 = 357 mm
code. The colour code adopted by most of the foundries are 100 mm hole 100 × 0.98 = 98 mm
as follows.
Part of the pattern Colour 98
102 mm
1. Surface to be left unmachined Balck
2. Surface to be machined Red
3. Coreprints and seats for loose core Yellow 204 mm
prints
4.  Seats for loose pieces Red strips on yellow 357 mm
background
5.  Stop offs or supports Black strips on Pattern dimension Table
yellow Metals/Alloys Shrinkage allowance (mm/m)
Gray cast iron 10.4
Pattern Design Considerations
White cast iron 20
A pattern not properly designed will produce only bad cast-
Plain carbon steel 20.8
ings. The following are some of the factors that must be
taken into account while designing a pattern. Chromium steel 20
Manganese steel 25–38
1. The pattern should be dimensionally accurate and
should have very good surface finish Al. alloys 12.5–15
2. Metallic pattern give good surface finish but are cost Brass 15.3
compared to wood castings Bronze 15.5–20.8
3. All the pattern allowances should be taken into account Copper 16
4. In the case of split pattern the parting surface should Zinc 24
be such that the larger part of the pattern should be in
Magnesium 17
the drag
5. All sharp corners and edges should be rounded Example 2:  Provide draft allowance for the wood pattern
6. Change in section thickness should be smooth, gradual in the example 1.
and uniform
98 mm
7. Match plate patterns are preferred for machine moulding
8. Jointed cores should be avoided
102 mm

Solved Examples 357 mm


Example 1: Calculate dimensions of the pattern for the
casting shown in figure. (Material white cast iron) Refer to the tables
Solution:  Assume 1° taper for external and 3° taper for in-
100
100 mm ternal Draft allowance is provided for the vertical surfaces
only
External 102 × tan 1°= 1.78 mm
200 mm ~ 2 mm
Internal = 102 × tan 3°= 5.35 ~ 6 mm
350 mm Outer dimension = 357 + 2 + 2
    = 361 mm
Solution:  Shrinkage allowance for white cast iron Inner dimension  = 98–12 = 86 mm
(from tables)
Chapter 2  •  Metal Casting and Forming  |  3.787

361 mm In expendable mould casting a refractory material is used


86 mm as mould material. The mould is to be destroyed for taking
out the solidified casting. Sand mould casting is an exam-
ple. The main draw back of sand mould casting is the poor
surface finish of the casting.

98
Sand Mould Casting
357 mm About 80% of the cast products are made using sand mould
Table casting method. Sand moulds are single casting moulds and
Draft Values for patterns are completely destroyed for taking out the casting. The
moulding material is sand mixed with small quantities of
Pattern Draft (degrees)
binding materials and additives and water to improve the
materials External surface Internal surface cohesive strength and mouldability of sand.
Wood 0.25 to 3.00 0.5 to 3.0 Sand moulds are prepared in wooden or metallic boxes
Metal 0.35 to 1.50 0.5 to 3.0 called flasks. Depending up on the type of casting there can be
Plastic 0.25 to 1.0 0.35 to 2.25
one flask or more than one flasks. Two flask system is the most
Machining allowances for patterns commonly used. In the assembled position, the upper flask is
called ‘cope’ and bottom flask is called ‘Drag’. In three flask
Material/alloy Machining allowances (mm) system, the intermediate or central flask is called the ‘Cheak’
( on surface)
One flask dsign is used in ‘full mould process’ or in pit
Cast Iron moulding where it is used as cope, the pit acting as ‘drag’
Medium castings 3
Large castings 10
Cores
Cast steel
Medium castings 4.5 Cores are the materials used for making cavities and hollow
Large castings 12 projections, which cannot normally be produced by pattern
Non-ferrous alone. Cores are generally made of sand. During casting,
(brass, bronze, Al) cores are surrounded by molten metal and are subjected to
Medium castings 1.5 severe thermal and mechanical conditions. There fore core
Large castings 5 sand should be of higher strength than the moulding sand.
Core is set in the mould cavity of the prepared mould before
Example 3: Steel castings are produced from a brass closing and pouring. Through holes, recesses, projections,
pattern which is to be made from a wooden pattern. If the undercuts and internal cavities can be formed using cores.
dimensions of a component part is 80 mm, the corresponding
dimension of the wooden pattern considering shrinkage Types of Cores
only will be Cores can be classified according to the state of the mate-
15.3 rial as
Solution:  Allowance for brass = 80 ×
1000
1. Green sand core
= 1.224 mm 2. Dry sand core
20.8
Allowance for steel = 80 ×
1000 Green sand core is formed by the patterns it self when it is
= 1.664 mm being rammed while preparing a green sand mould (Green
Total shrinkage allowance sand means sand containing moisture) The core is made of
= 1.224 + 1.664 the same sand as the moulding sand. Green sand core is
= 2.888 mm weak and can be used only for light castings.
= 2.9 mm (approx) Dry sand core is made separately by core making process
Dimension of wooden pattern of moulding a green core, drying, baking, finishing, coat-
= 80 + 2.9 = 82.9 mm ing etc. They are positioned in the mould, after the pattern
is taken out before the mould is closed. These are the most
Classification of Casting Process commonly used cores.
According to the position of the core in the mould, core
The casting process may be classified in to
can be classified as
1. Expendable mould casting
1. Horizontal core
2. Permanent mould casting
2. Vertical core
3. Semi-permanent mould casting
3. Balanced core
3.788  |  Part III  •  Unit 7  •  Manufacturing Technology

4. Hanging core sand has proper amount of water added to it to give a high
5. Drop core etc strength with sufficient plasticity it is said to be ‘tempered’.
According to the amount of clay present, the moulding
Coreprints and Coreseats sand may be classified as
Coreprints are extra projections provided on the pattern for 1. Silica sand: up to 2% clay
making depressions in the mould for the placement of the core. 2. Lean or weak sand: 2 to 10% clay
The depression made by the coreprint in the mould is called 3. Moderately strong sand: 10 to 20% clay
core seat. Cores are placed in the core seats. Coreseats sup- 4. Strong sand: up to 30% clay
port the core against buoyancy of molten metal. Core prints, 5. Extra strong or loam sand: up to 50% clay
though a part of the pattern do not appear on the cast part.
According to the initial conditions and use moulding sand
may be classified as
1. Green sand
Core print Core print 2. Dry sand
3. Loam sand
Pattern 4. Facing sand
Mould cavity 5. Parting sand
6. Backing sand
Foundry sand containing moisture is known as green sand.
Chaplet

Green sand may contain 20 to 30% clay and 6 to 8% water.


Core seat

Another classification of moulding sand is


1. Natural sand
2. Synthetic sand
3. Chemically coated sand

Mould Core Natural sand is available from natural deposits. It contains


sufficient or slightly more clay content that is required
for moulding. So only additives and water need be
added.
Relatively clay free sand having specified grain size is
mixed with suitable type of clay additives and water to form
Section of the cast part
synthetic sand.
Clean silica grain are coated with a non thermosetting
Chaplets hydrocarbon resin, which acts as a binder to form chemi-
Chaplets are metallic supports often kept inside the mould cally coated sand.
cavity to support the cores. They are made of the same
material of the pouring metal. They melt and fuse with the
Binders
pouring metal during solidification. There are various types Binders used in a foundry may be organic or inorganic. The
of chaptlets such as radiater chaplets, rivaled chaplets, dou- most common is inorganic binders. Fire clay, Kaolinite, Illite
ble head chaplets, cast chaplets, sheet metal chaplets etc. and Bentonite are the clays used in moulding. The basic
constituent which gives refractoriness to a clay is aluminia,
Moulding Materials Al2O3. Bentonite is the most commonly used clay. As it
Moulding materials may be classified in to two groups– needs only smaller quantity of water to get sufficient plas-
basic and auxiliary ticity. Therefore steam generation during casting is less and
Basic materials include silica sands and binders moulding sand of low porosity or permeability can be used.
Auxiliary materials include various additives which impart Other inorganic binders are Portland cement, sodium silicate
desired properties to the moulding and core sands. etc.
A moulding sand consists silica sand grains, binder, Organic binders are most commonly used in core
additives and water. Silica sand grains form the base of the ­making. Examples are
moulding sand. Binder is the bonding agent in a green sand. 1. Cereal binders obtained from wheat, corn etc
Most commonly used binder is clay. Clay imparts cohe- 2. Drying oils such as linseed oil, fish oil, soyabean oils
siveness and plasticity to the moulding sand in the moist and some mineral oils
state and increase its strength after drying. When moulding 3. Pitch and molasses
Chapter 2  •  Metal Casting and Forming  |  3.789

Additives 7. Fineness
Additives are small quantities of special materials added to Finer mould sand resists metal penetration and pro-
a moulding sand in order to enhance its existing properties duces smooth surface for the casting. But permeability
and to impart some special properties. is reduced by fineness
Sea coal, cereals, sand dust, wood flour, silica flour, fuel 8. Coefficient of expansion of the moulding sands should
oil, Iron oxide, dextrin etc are some of the additives used. be less

Properties of Moulding Sand Forces Acting on the Moulds


The choice of moulding maerials is based on their process- When the mould cavity is filled with molten metal it experi-
ing properties. The properties that are required in moulding ence high metellelo static pressures and buoyant forces As
materials are a result moulds may be distorted or cores may be displaced.
1. Permeability or porosity The effects of these forces can be eliminated by various
The ability of the sand to allow gases from molten methods such as placing weights on cope, clamping cope
metal and steam and water vapours generated in the and drag properly by reinforcing sand mass etc.
mould to pass through it is called permiability. It The metellostatic force is due to the head acting by the
depends on the size and shape of grains, moisture con- molten metal in the mould cavity
tent, degree of ramming etc Metallostatic force is given by

2. Plasticity or flowability Fm = Awh


It is the property of the moulding sand by which it is Where A = projected area.
able to flow around and over a pattern during ram- w = sp. weight of molten metal
ming and to uniformly fill the flask. Plasticity can be h = head of the molten metal
increased by adding clay and water
upward force on core = displaced volume × difference in
3. Adhesiveness sp.wt
The property to adhere with other materials is called Example 4:  Find the weights that is required to be kept to
adhesiveness. Adhesiveness between moulding sand compensate the force during pouring in casting of a cast
and the moulding flasks are required during handling iron pipe of 12.5 cm OD and 10 cm ID with a length of
of the rammed boxes 200 cm. The metal head is about 25 cm. While moulding,
4. Cohesiveness or strength flask size used for the purpose is 220 × 25 × 20 cm in size.
The ability of the sand particles to stick to each other is Weight density of core sand is 0.0165 N/cm3 and that of
called cohesiveness. In the absence of this property the liquid metal is 0.0771 N/cm3
mould will break when molten metal is poured. This Solution:  Displaced volume
depends up on grain size of sand and clay content V = Volume of core in the mould cavity
Green strength is the strength of the sand in green or p
moist state. = (10) 2 × 200 = 15708 cm3
4
Dry strength is the strength of moulding sand in dry
Upward force on the core
condition.
= V(wm–wc )
Hot strength is the strength required to hold the shape
of the mould cavity in hot condition. It is above 100°C = 15708 (0.0771–0.0165)
and the molten metal in the mould is still in liquid state. = 951.9 N
Projected area
5. Refractoriness A = 12.5 × 200 = 2500 cm2
The capability of the moulding sand to with stand high
Metallostatic force
temperatures of the molten metal with out fusing is
Fm = Awmh
known as refractoriness
= 2500 × 0.0771 × 25
6. Collapsibility = 4818.75 N
It is the ability of the moulding sand to decrease in Total upward force = 4818.75 + 951.9 = 5770.65 N
volume under compressive forces developed by the wt of cope
shrinkage of metal during freezing and subsequent
⎡ p (12.5) 2 ⎤
cooling. This is very important for cores. It permits = ⎢ 220 × 25 × 20 - × 200 ⎥ × 0.0165
the moulding sand to break easily during its knock out ⎣ 4 2 ⎦
from casting = 1612.5 N
3.790  |  Part III  •  Unit 7  •  Manufacturing Technology

et upward force = 5770.65–1612.5


N in to the mould at a pressure of about 1.5 Kgf/cm2 for 10 to
= 4158.15 N. 30 seconds.
So the weight that is required to compensate upward force Sodium silicate reacts with CO2 to form silica jel
is approximately 4200 N. To account for the dynamic SiO2 Na2O + CO2 → Na2CO3+H2O + SiO2
forces due to change in momentum of the molten liquid an Carbon dioxide is expected to form a weak acid, which
additional 50% wt may be placed. hydrolises the sodium silicate resulting in amorphous silica,
Total weight = 4200 × 1.5 which form the bond. The introduction of CO2 gas starts
= 6300 N. the reaction by forming hydrated sodium carbonate. This
gelling action increases the viscosity of the binder till
it becomes solid. The compressive strength of the bond
Moulding processes increases due to dehydration.
According to the method used moulding processes can be
classified as Melting and Pouring
1. Bench moulding After the preparation of moulds, molten metal is poured
2. Floor moulding in to the mould to get the casting. Various type of melting
3. Machine moulding furnaces are available for melting metals and alloys. Most
commonly used furnaces are,
Bench moulding is used for preparing small moulds. It is
carried out on a bench of convenient height. 1. Cupola furnace–for melting cast iron
In floor moulding, the mould is made in the foundry 2. Crucible furnace–for melting non-ferrous metals
floor. It is used for making medium and large size castings. 3. Electric furnace–for melting steel and special alloy
Machine moulding is used in batch and mass ­production. steels
The moulding production becomes faster and labour is min- Molten metal is poured into the mould through the gating
imised. Moulding machines are classified according to system.
1. Method of compacting moulding sand and
2. The method of removing the pattern Gating System
Squeezer machines, jolt machine and sand slinger comes in All the passages through which molten metal passes before
the first group. entering the mould cavity is referred as gating system.
Straight draw moulding machine and turn over moulding Various elements of a gating system are
machine are example of second group.
1. Pouring basin
According to the mould materials used moulding pro-
2. Sprue
cess can be classified as
3. Sprue base or well
1. Green sand moulding 4. Runner
2. Dry sand moulding 5. Runner extension
3. Loam sand moulding 6. Choke
4. Metallic moulding or permanent moulding 7. Skim bob
5. Carbondioxide moulding 8. Gates and/or ingates
9. Riser
Green sand moulding is used for small and medium cast-
ings. It is simplest and least expensive and requires less time Pouring Basin
to prepare.
A pouring basin or cup is a reservoir at the top of the ver-
In dry sand moulding the green sand mould is dried
tical passage (sprue) in the cope, where the molten metal
before pouring molten metal. Drying or baking is carried
is poured, It is used for maintaining the required flow rate,
out in ovens. Time for baking depends upon the binders
minimize turbulence and to aid in separating dross and slag
used.
from the molten metal before it enter the runner system.
Loam sand moulding is used for large castings.
Approximate contour of the casting is made by bricks and
Sprue
other materials and a thick coating of loam sand is given
inside. Using a sweep pattern, correct contour of the casting It is a vertical passage through the cope and connects the
is produced. pouring basin to the runner or gate. A straight sprue with
In carbondioxide moulding, sand is thoroughly mixed sharp corners will cause severe aspiration (air sucking)
with 3 to 5% sodium silicate liquid base binder (water glass thereby causing turbulence in the molten metal. If a tapered
SiO2­ Na2O) in a muller. Mould is prepared using the sand sprue with round corners and dam type pouring basin are
mixture by machine or hand moulding. Then CO2 is forced used aspiration and turbulence are negligible.
Chapter 2  •  Metal Casting and Forming  |  3.791

The molten metal when moving from the top of the cope Runners
to parting plane gains in velocity and as a consequence A runner is a horizontal channel which connects the sprue
requires smaller area of cross section for the same amount with the gates. It is generally located in the horizontal, part-
of metal to flow. If straight sprue is used metal flow would ing plane. Generally these are constructed in a trapezoidal
not be full at the bottom leading to aspiration. cross section. For ferrous metals, generally runner is pro-
Equations of continuity can be used to find the exact vided in the cope portion and in gates in the drag portion.
tapering required. Denoting top and choke by ‘t’ and ‘c’. This helps to trap the lighter slag and dross flowing with the
AtVt = AcVc molten metal.

or At = Ac
Vc Runner Extension
Vt The runner is extended a little further after it meets the
2gh ingate. The slag flowing with the molten metal is trapped at
But V =
the runner extension with out going to the ingates.
Or Va h
Skimbob
hc Skimbob is an enlargement provided in the runner. It’s func-
\ At = Ac
ht tion is to trap impurities such as dross or eroded sand from
going in to the mould cavity.
From the above equation it can be seen that the profile of an
ideal sprue is parabolic. But in practice for convenience a Gates
straight tapered sprue is used. Gate is a channel which connects runner with the mould
Riser cavity. Ingate is the end of the gate where it joins the
mould cavity and through which the molten metal is intro-
duced to the mould cavity.

Pouring Choke
cup Choke is that part of the gating system which has the small-
Casting
est cross sectional area. It helps to lower the flow velocity
Sprue
in the runner, to hold back slag and foreign materials in the
runner also minimise the sand erosion.
Gate
Gating Ratio
It describes the relative cross sectional area of sprue: total
Runner
Sprue extension
runner area: total gate area.
Runner Depending up on the position of choke, the gating sys-
base
tem may be described as
Top riser
Strainer Pouring basin 1. Pressurised or choked system
2. Unpressurised or free system
In pressurised system, ingates serve as choke. The total gate
area is smaller than the sprue area. So a back pressure is
Gate
Sprue maintained in the system. A gating ratio 2:2:3 indicates a
unpressurised gating system.
Sprue
base Mould
Splash Skim cavity Types of Gates
core bob Side Gates may be classified as
riser
1. Parting line gate
Sprue Base or Well 2. Bottom gate
Where the sprue joins the runner an enlargement called 3. Top gate
sprue base or sprue well is provided. The mollen metal pool In parting line gate, metal enters the mould cavity at the
formed at the sprue base p­ revents excessive sand erosion parting line. The gate may contain skimbob, skimming
where the molten metal impinges. gate, shrink bob and whirl gate. Skimming gate is a vertical
3.792  |  Part III  •  Unit 7  •  Manufacturing Technology

passage through the cope: Lighter foreign material is As the metal enters the pouring basin, it has the highest
trapped in skimming gate. Shrink bob may be provided if potential energy with no kinetic or pressure energies. But as
there is a tendency for shrinkage defect near the ingate. the flow proceeds there is frictional head loss and heat loss
Whirlgate employs centrifugal force to aid the slag come (heat loss is not represented in the Bernoulli’s equation).
to the centre where it rises up in the skimming gate. A bot- The continuity equation which states that
tom gate is provided in the drag portion of the mould. Metal Q = AV = constant can also be applied
fills in the bottom first and then rises up steadly in the mould. (Q = rate of flow m3/s
In top gate, metal is poured down directly, into the mould
A = area of cross section, m2
cavity. In this, molten metal at the top of the casting is
always hot. V = velocity, m/s)

Riser Pouring Time


Due to the shrinkage of metal during solidification voids are The duration of time required for complete filling of a mould
likely to form in the casting unless additional molten metal with molten metal is termed as pouring time. It depends on
is fed in to these places.A reservoir of molten metal is to be the casting material, complexity of casting, section thick-
maintained from which the metal can flow readily in to the ness and casting size.
casting when the need arises. These reservoirs are called ris- High pouring rate means turbulent flow in the mould and
ers. Riser is a hole cut in cope to prevent the molten metal to mould erosion. Low pouring rate requires higher pouring
rise above the highest point in the casting. temperature. Otherwise the mould cavity may not be filled
Other functions of the risers are completely.
Empirical formulae for pouring time for, different cast-
1. The pourer can see the metal in the mould cavity. If the ing materials are given below:
metal is not seen it indicates that metal is not sufficient
1.  Grey cast iron, mass less than 450 kg
or there is some obstruction
Pouring time,
2. It gives passage of steam, gas, air etc
 T 
t = k 1.41 + w seconds
Directional Solidification  14.59 
In a casting all parts do not cool at same rate. Some parts
solidify more quickly than others. Due to this cavities are Fluidity of iron in inches
Where k =
formed at certain regions. This cavities should be filled with 40
the molten metal from the risers. For this to happen, the T = average section thickness, mm
riser should be the last to solidify. So solidification should W = mass of casting, Kg
progress towards the risers from the filled areas. This is 2.  Grey cast iron, mass greater than 450 Kg
known as directional solidification.
Directional solidification can be achieved by the follow-  T 
t = k 1.236 +  2 w secs
ing methods.  16.65 

1. Proper designing and positioning of risers 3.  Steel castings
2. Use of padding to increase thickness of certain sections t = (2.4335–0.3953 Log w) w secs
3. Use of exothermic materials in risers
4. Use of chills in moulds Choke Area
It is the area at sprue exit. Choke area can be calculated
Gating System Design using Bernoullis equation as
Bernoulli’s theorem can be applied to the molten metal that W
folws through the gating system. Ignoring frictional loss, A=
dtc 2 gH
law states that
p v2 Where A = choke area, mm2
h+ + = constant
w 2g W = casting mass, Kg
t = pouring time, s
Where h = potential head, m
d = density of molten metal Kg/mm3
p = pressure, Pa
v = liquid velocity, m/s g = acceleration due to gravity, mm/s2
w = specific weight, N/m3 H = effective metal head (sprue height)
g = acceleration due to gravity, 9.81 m/s2 C = efficiency factor–a function of gating system used.
Chapter 2  •  Metal Casting and Forming  |  3.793

H depends on the casting dimensions and the type of gat- draw back is the formation vacuum due to the formation of
ing used. The following relations can be used. metal skin on the walls. This hinders the metal feeding. To
Top gate, H = h avoid this a permeable dry sand core is used, connecting it
c to the mould sand layers.
Bottom gate, H = h -
2
Shape and Size of Riser
p2 To help directional solidification the risers should loss heat
Parting gate, H = h -
2c at a slower rate. Amount of heat content is proportional to
Where h = height of sprue the volume and rate of heat dissipation depends up on the
p = height of mould cavity in cope surface area. Therefore a riser should be designed for a high
c = total height of mould cavity V/A (Volume/Surface area) ratio for a given size. From this
point of view a spherical shape is ideal one as it is hav-
Risering Design ing the lowest surface area a for same volume. But risers of
Riser is used to feed the casting during solidification so that spherical shape is difficult to mould. Therefore, a cylindri-
no shrinkage cavities are formed. The requirement of risers cal shape is preferred. Height of a cylindrical riser is gener-
depends to a great extend up on the type of metal poured ally taken as 1.5 × diameter of riser.
and the complexity of casting. Various metals have various
volumetric shrinkages. Riser Size
Grey cast iron some times may have a negative shrink- The solidification time of a casting depends up on the heat
age. This is because with higher carbon and silicon con- in the casting and its dissipation. So it is directly propor-
tents, graphitisation occurs which increases the volume and tional to the volume. And indirectly proportional to the sur-
counter acts the metal shrinkage. Risering is not very criti- face area. Based on these facts many relations are available
cal in these situations. But for metals such as aluminium for determining the riser size.
and steel volumetric contraction is very high and risering is
very important. Chvorinov’s Rule
Riser is also called feed heads. Molten metal rises in the Solidification or Freezing Time
riser after the mould cavity is filled up. They are used in
heavy section casting or for high shrinkage alloys. ⎛ V ⎞2
During casting metal shrinkage occur in three stages, T =C⎜ ⎟
⎝ SA ⎠
i) Liquid contraction or shrinkage
Where V = volume of casting
ii) Solidification shrinkage
SA = surface area of casting
iii) Solid shrinkage
C = constant of proportionality
First two are considered for risering purposes. To achieve directional solidification,
⎛V ⎞ ⎛V ⎞
Riser Location ⎜⎝ ⎟⎠ >⎜ ⎟
SA Riser ⎝ SA ⎠ Casting
A riser should be located close to each heavier section. It
should be located in such a manner that it is the last portion In practice
of the casting to solidify.
⎛V ⎞ ⎛V ⎞
⎜⎝ ⎟⎠ = 1.10 to 1.15 ⎜ ⎟
Types of Risers SA Riser ⎝ SA ⎠ Casting
Depending upon the location, a riser can be side riser, top ⎛ V ⎞ for the casting is known.
riser and end riser. If it is located between runner and cast- ⎜⎝ ⎟⎠
SA
ing it is also called live or hot riser as this contains the hot-
test metal. Top risers and end risers are also called dead So ⎛⎜ V ⎞⎟ for the riser can be calculated. Assuming
risers as these risers fill up with coldest metal and are likely ⎝ SA ⎠
to solidify before casting. A riser can also be an open riser height to diameter ratio for a cylindrical riser, its size can
or blind riser. be determined

Blind Risers Caine’s Formula


A riser which does not break to the top of the cope and Caine’s method of determining the size of a riser in based
is entirely surrounded by moulding sand is known as blind on the experimentally determined hyperbolic relationship
riser. As the rate of cooling is slower it helps directional between relative freezing times and relative volumes of the
solidification. Also only a smaller size is required. It’s main casting and the riser.
3.794  |  Part III  •  Unit 7  •  Manufacturing Technology

Relative freezing time or freezing ratio, It is generally preferable to choose a riser with height to
diameter ratio = 1
Cooling characteristics of casting p D3
X = In such cases, Volume =
Colling characteristics of riiser 4
p D2
(SA/V )Casting Surface area = + p D2
4
=
(SA/V )Riser 5
= p D2
Volume ratio, 4
p D 3 5p D 2
Volume of riser Modulus of riser, MR = = 0.2 D
Y = 4 4
Volume of casting Since MR = 1.2Mc
D = 6 Mc
Caine’s formula is given as,
Thus in this method, calculation of riser size is very
a much simplified.
X = +c
Y -b Example 5:  Optimum pouring time for a casting of cast
Where a = freezing characteristic constant iron of mass 60 Kg and a section thickness 50 mm is (take
b = Liquid–solid solidification contraction constant fluidity = 22 inches).
c = Relative freezing rate of riser and casting
Typical values a, b, c for commonly used cast metals are (
Solution:  Pouring time t = k 1.41 + T 14.59 W)
given below.
22 ⎛ 50 ⎞
= ⎜ 1.41 + ⎟ 60
Cast metals a b c 40 ⎝ 14.59 ⎠
Steel 0.12 0.05 1.00
= 20.6 sec.
Aluminium 0.10 0.06 1.08
Graycast Iron 0.33 0.03 1.00
Cast iron, Brass 0.04 0.017 1.00
Example 6:  Determine diameter of the sprue at the exit to
Aluminium, Bronze 0.24 0.017 1.00 fill the mould of a CI casting neglecting flow losses.
Silicon Bronze 0.24 0.017 1.00 Casting weight = 35 Kg
Pouring time = 22 seconds
A typical Caine’s hyperbolic curve is as shown in figure Density of melt = 7 gm/mm3
Y Height of sprue = 60 mm
And top gate.
Sound casting
Solution:  Choke area
W
CA =
cdt 2 gH
Unsound casting
Where W = casting weight = 35 Kg
X
C = efficiency factor = 0.85 for taper sprue
Caine’s curves for different cast metals are available in hand d = density of melt = 7 gm/cm3 = 7 × 10-6 kg/mm3
books. To find the riser size for a given casting, the riser t = pouring time = 22 seconds
diameter and height are assumed. Knowing values of a, b, H = effective metal head
and c, the values of X and Y are calculated. Values of x and = height of sprue for top gate
Y are plotted on the hyperbolic curve. If the values meet   = 160 mm
above the curve the assumed size is satisfactory. 35
\ CA =
-
0.85 × 7 × 10 × 22 2 × 9800 × 160
6
Modulus Method
= 150.99 mm2
Modulus is the inverse of cooling characteristic
Diameter at choke area = d
V pd2
i.e, modulus = = 150.99
SA 4
It has been empirically established that if the modulus of 150.99 × 4
the riser exceeds the modulus of casting by a factor 1.2, the d2 =
π
feeding solidification would be satisfactory.
d = 13.865 mm
Chapter 2  •  Metal Casting and Forming  |  3.795

Example 7:  A casting of 200×100×70 mm3 size solidifies 0.12


in 10 minutes. Estimate the solidification time for 200×100 0.1067D = +1
0.0000556 D 3 - 0.05
×10 mm3 casting under similar conditions.
0.12
Solution:  Solidification time 0.1067 D - 1 =
0.0000556 D 3 - 0.05
2
⎛V ⎞ 5.93 × 10-6D4 – 0.0000556 D3
T = k⎜ ⎟
⎝ A⎠ – 5.335 × 10-3 D + 0.05 = 0.12
Where k = mould constant D4–9.376 D3 – 900D = 11804
V = volume of casting D = ~ 16 cm (By trial).
A = surface area of casting
V1 = 200 × 100 × 70 mm3 Cooling and Solidification
V2 = 200 × 100 × 10 mm3 When a molten metal starts to cool and solidify crystals
A1 = [(20 × 100) + (100 × 70) +(70 × 200)]2 will begin to form independent of one another, at differ-
A2 = [(200 × 100) + (100 ×10) + (10 × 200)]2 ent locations in different orientations within molten mass.
The formation of the first tiny particle is called nucleation.
2 2 2 2 With passage of time these tiny particles (crystals) grow in
t1 ⎛ V1 ⎞ ⎛ V2 ⎞ ⎛ V1 ⎞ ⎛ A2 ⎞
= ⎜⎝ A ⎟⎠ = ⎜⎝ V ⎟⎠ × ⎜⎝ A ⎟⎠ size or grain growth takes place. When the cooling is rapid
t 2 ⎜⎝ A1 ⎟⎠ 2 2 1 the pattern of grain/crystal growth is in the shape of radial
2
⎡ ( 200 × 100) + (100 × 10) + (10 × 200 ⎤
2 arms which begin to develop from the various nuclii. As
⎛ 70 ⎞
=⎜ ⎟ ×⎢ ⎥ time progresses secondary arms develop at right angles to
⎝ 10 ⎠ ⎣ ( 200 × 100) + (100 × 70) + (70 × 200) ⎦ the radial arm and the process continues. This type of tree
⎛ 23000 ⎞
2 like crystals are called ‘dentrites’. When cooling is slow
= 49 × ⎜ = 15.42
⎝ 4100 ⎟⎠
equiaxed crystals grow uniformly in all directions. Crystal
grows until it comes into contact with adjacent crystal of
t1 10 proper geometrical form and different orientations. The
t2 = = = 0.648 minutes
15.42 15.42 boundary formed between two adjacent crystal line growth
Example 8: Calculate the size of a cylindrical riser because of different orientations of the grains is known as
necessary to feed a steel casting block of dimension grain boundary
30 × 30 × 5 cm with a side riser, casting poured horizontally
Solidification in a Casting Mould
into the mould. Assume height = diameter for the riser.
When molten metal is poured in to a mould solidification
Solution:  Volume of casting = 30 × 30 × 5 = 4500 cm3 will be rapid along the mould walls. In this chill zone, a
Surface area = 2 [30 × 30] +4 [30 × 5) layer of fine, polly crystalline, equiaxed grains will be pro-
= 2400 cm2. duced. As the solidification progress the grains growth will
π D3 be inwards, towards the centre, their lateral growth reduced
Volume of riser = due to the early contact made with the adjacent crystals.
4
Here columnar, dentrite crystals are formed leading to the
p D 2 5p D 2 formation of a mushy zone. The width of the mushy zone
Surface area of riser = p D2 + =
4 4 depends upon the mould material, cooling rate and tempera-
ture gradient. As the heat extraction continue through out
2400 4500 the mass, simultaneous freezing of the metal at the centre
Freezing ratio X =
5p D 2 p D 3 of the mould takes place. Here equiaxed coarse grains are
4 4 formed. This central zone of equiaxed coarse grain can be
extended throughout, without formation of dentrites, by
= 0.1067 D
the addition of inoculants (nucleating agents ) to the liquid
0.25p D 3 alloy. Ferro-silicon, Ferro-manganese, Ferro-Chromium,
Volume ratio Y = = 0.0000556 D 3 Ferro-silicon-chromium etc are the inoculants used.
4500
a Types of Solidification
Caine’s formula X = +C
Y -b i) Skin forming
For steel a = 0.12 Skin forming takesplace when molten metal is solidi-
b = 0.05 fied in moulds, in the case of pure metals or alloys hav-
c=1 ing eutectic compositions. Since the mould walls are
Substituting values in the Caine’s formula at room temperature, solidification first starts near the
3.796  |  Part III  •  Unit 7  •  Manufacturing Technology

mould walls. The solidification front will move towards Pinhole Porosity
centre, and layer by layer solidification takes place This is due to the presence of hydrogen in molten metal.
ii) Dantritic growth During solidification of the molten metal hydrogen leaves
Dantritic growth will happen if mushy zone appear the metal causing small diameter and long pinholes.
during solidifications. Initially solid is nucleated
near the boundary and the solidification front moves Shrinkage Cavities
towards the centre. Solid is also nucleated in the liquid Shrinkage cavities are formed during solidification of the
region and the solidification front movement in this molten metal, if feeding by risers are not proper or if the
case is towards the centre and to the primary dantrite. casting design is not proper.
In this case unidirectional solidification does not takes
place. Micro voids are formed at the meeting point of Moulding Material Defects
primary and secondary dantrites. These voids cannot Lack of the required properties of moulding materials
be compensated by the riser. This results in defective improper ramming etc will lead to defects such as scabs,
casting swell, runout and drop.

Segregation Scab
When a liquid alloy metal cools and solidifies the separating Scabs are projection formed on the casting. This occurs
out of the constituent elements of different freezing points is when a portion of the mould face lifts and metal flows
called segregation. Various types of segregation are, micro underneath in a thin layer. If it happens due to the expansion
segregation, macro segregation, normal segregation and of surface layer of sand it is called expansion scab. Scabs
gravity segregation. occur due to uneven ramming, excess moisture in sand etc,

Casting Defects Run Out


In a casting various defects may occure due to various rea- Runout is the escape of molten metal from mould cavity.
sons. It may be due to improper pattern design, improper Faulty moulding flask or faulty mould making may lead to
mould construction, improper melting practice, improper this.
pouring etc. The defects may be broadly classified as
Drop
1. Gas defects
2. Shrinkage cavities Drop occurs when cope surface cracks and breaks.Pieces
3. Moulding material defects of sand fall into mollen metal. Low green strength of sand,
4. Pouring metal defects improper ramming of cope etc are the causes.
5. Metallurgical defects
6. Moulding and corebox defects
Cuts or Washes
Cuts or washes appear as low projections and areas of
excess metal casused by erosion of moulding sand by the
Gas Defects flowing molten metal. It usually occurs with bottom gat-
These are due to lower permeability of the mould. Blow ing castings, when too much metal is made to flow and the
holes and open blows, air inclusions and pinhole porosity moulding sand has insufficient hot strength.
etc. come under gas defects.
Blow holes and open blows are spherical, flattened or Pouring Metal Defects
elongated cavities present inside the casting or on the sur- Misrun, cold sheet, poured short etc are pouring metal
face. If they are on the surface they are called open blows. defects.
Moisture in the mould will be converted into steam due to
the heat of the molten metal. A part of the steam may be Misrun
entrapped in the casting. This leads to the formation of blow
This happens when the metal is unable to fill the mould
holes. Lower venting and lower permeability of mould also
­cavity completely leaving unfilled cavities. This may h­ appen
causes formation of blow holes and open blows.
due to lack of fluidity or due to obstruction in flow.
Air Inclusion Cold Shut
Atmospheric and other gases are absorbed by the molten
This is caused when two metal streams coming from
metal at high temperatures in the furnace, during pouring or
­different directions while meeting in the mould cavity do
in the mould. If they are not allowed to escape, they will be
not fuse together properly. Due to this, discontinuity or
trapped inside the casting.
weak spot in the casting is formed.
Chapter 2  •  Metal Casting and Forming  |  3.797

Too thin sections, improper gating system, slow and Design of Castings
intermittant pouring, poor fluidity of metal etc may cause The design of a cast part should be such that the design
misrun or cold shut. should ensure high level of its working characteristics such
as strength, rigidity, stiffness, lightness and corrosion resist-
Poured Short ance. Proper attention to design aspects will minimize cast-
If the mould cavity is not completely filled due to insuffi- ing problems and lower the costs.
cient metal, this happens. The design of the casting should be such that it allows
directional solidification. Product design should be studied
under the following categories.
Metallurgical Defects
1. Design for economical moulding
Hot Tears 2. Design for elimination of defects
Occur due to hindered contractrion of casting parts imme- 3. Design for features to help handling of castings
diately after the metal solidification. Main reason is poor The following factors should be taken into account while
casting design. designing a casting.
1. Function of the casting
Hardspots 2. Selection and optimum use of the casting alloy
This happens in metals such as gray cast iron with in suf- 3. Strength of the casting
ficient silicon. Such metals may become hardened by the 4. Simplification of foundry practices
chilling effect of moulding sand. Hardspots make machin- 5. Consideration of safety aspects
ing of the metal difficult. 6. Economy of production
A few of the rules in design are
Mismatch or Mould Shift
1. Stress concentration should be avoided. For thin,
This is due to mismatch of cope and drag flasks at the part-
sharp corners and frequent use of fillets etc should be
ing line. The worn out or loose dowels in the pattern halves,
avoided
inexpert assembly of mould halves etc are reasons. Core
2. All members should have uniform sections as far
shift occurs due to the misalignment of core halves during
as possible. Abrupt changes in sections should be
assembly.
avoided
3. Large flat surfaces should be avoided because it is
Flashes or Fins difficult to get true surfaces on large castings
Flashes or fins appear at the mould joints where gaps are 4. Pattern allowances should be provided
formed This may be due to wear or warping or improper 5. By providing curved shapes, contraction stresses
fastening of the cope and drag. should be minimized (example- arm of pulleys and
wheels)
6. Stiffening members such as webs and ribs should be
Buckles and Rat Tails minimum as these give rise to defects like hot tears
Buckle is a broad vee shaped depression appearing on the and shrinkage
face of casting. It extends in a fairly straight line across the 7. Deep and narrow pockets in the casting should be
entire flat surface. It results from the sand expansion car- avoided to minimize cleaning costs
ried by the high temperature molten metal, when sand has 8. Vertical walls should be as smooth as possible for easy
insufficient hot deformation. It also happens in the design withdrawals of patterns (markings such as names or
with too large flat surfaces in the mould cavity. Rat tail numbers etc on vertical walls should be avoided)
is a defect similar to buckle, happening due to same rea- 9. Tolerance should be provided depending up on the
sons, but the shape of the defect is different. Rat tail is not dimensional accuracy desired
shaped as a broad vee. Buckles are the rat tails which are
severe.
Special Casting Processes
Sand casting processes give fairly good results at lowest
Swell
cost. Main draw back of sand casting is that the moulds are
Swell is a slight, smooth bulge appearing on vertical walls single use types. If in its place a permanent mould could
of a casting. This is caused by liquid metal pressure at areas be used there is considerable saving in time and labour
of low mould strength, due to improper ramming or too cost. Also surface finish and accuracy is poor in the case of
high water content. sand casting. To overcome this a number of special casting
3.798  |  Part III  •  Unit 7  •  Manufacturing Technology

processes are developed. Some of the widely used casting the mould is placed upside down and heated to melt out the
processes are, wax. The shell produced is used as mould for casting. The
final moulds can be made as solid type or shell type. In solid
i) Shell moulding
type, the shell is placed in a flask and a hard setting mould-
ii) Precision investment casting
ing material is poured. For shell type moulding dipping in
iii) Permanent mould casting
slurry is continued till sufficient thickness is obtained.
iv) Die casting
v) Centrifugal casting Permanent Mould Casting
vi) Squeeze casting
In permanent mould casting the mould can be used repeat-
Shell Moulding edly. Permanent moulds are also called dies. In this method
gravity feeding of molten metal is used. So this method is
It is a modification of sand moulding process. In this pro-
also known as gravity diecasting. This method is generally
cess thin, shell type half moulds are made using a mixture
used for non-ferrous metals and alloys such as Al, Mg, Zn,
of dry silica sand and phenolic resin.
Cu–alloys, Sn, Pb etc. A permanent mould is made up of
The sand is mixed with either urea or phenol formal-
two parts–one stationary and the other moving. Two com-
dehyde in a muller and transferred to a dump lox. Pattern
mon designs are
made of metal is heated to 205 to 230°C and placed over
the dump box and clamped. When the dump box is inverted i) Hinged type or book type
the sand resin mixture falls over the hot patterns. The resin ii) Straight line retractable type
melts and acts as bond between sand grains at the surface
For making hollow portions, cores are to be used. Cores can
of the pattern. After 30 seconds a hard and thin layer sand is
be made of metal or sand. Metal cores can be used repeat-
formed over the pattern. Then the dunp box is brought to the
edly. But complex shapes cannot be used. When sand cores
original position. The excess sand falls back. Pattern with
are used, the process is called semi–permanent moulding.
thin shells is cured at 315°C for 2 minutes. The shell is then
ejected by ejector pins and removed. Similarly shell is made Die Casting
for the other half also. These shells are assembled in a flask
with backing sand to form the mould. Die casting involves the preparation of components by
High dimensional accuracy and good surface finish are injecting molten metal at high pressure into metallic die.
the main advantages. The shells can be stored for a long time. Unlike in permanent mould casting or gravity die casting,
High cost and requirement of specialized equipments are metal is fed under pressure. So this process is also called
the main disadvantages. pressure die casting. Narrow sections complex shapes and
fine surface details can easily be produced as high pres-
Precision Investment Casting sure is used in this method. The stationary part of the die is
called the cover die and moving part is called the ejector die.
(Lost Wax Method)
Lubricant is sprayed on the diecavity manually or by auto
The term ‘investment’ refers to the layer of refractory mate- lubrication system to avoid sticking of the casting in the die.
rial with which the pattern in covered to make the mould. Die casting machines are of two types.
Like sand casting method mould is destroyed every time a
casting is made. 1. Hot chamber die casting
Shell moulding also is a precision investment casting 2. Cold chamber die casting
method, though lost wax method is generally refered as pre- In hot chamber die casting machine the melting unit of
cision investment casting. metal forms an integral part of the machine.
In lost wax method a wax pattern is used. Wax pattern is In cold chamber machine metal is melted in a furnace
melted from the mould, leaving the cavity. outside and transferred to the cylinder of the machine from
In this method a master pattern is prepared using steel where it is forced into the mould by means of the plunger.
or brass. Using this pattern split mould is made with bis- Advantages of die casting are,
muth alloy or lead alloy. This mould is used for making wax
pattern. Several wax patterns are assembled with necessary 1. Very high production rates are possible
gates and risers. This assembly is dipped in a refractory 2. Thin an complex section can be cast
slurry and refractory fine sand is sprinkled over it to ensure 3. Close dimensional control can be maintained
smooth surface for the casting. After the primary casting
Vacuum Die Casting
has dried sufficiently a final investment layer consisting of
coarser and less expensive slurry is applied. The common Air trapped in the dies when they are closed is a major
refractory used is silica. The binder is of gypsum or water problem die casting. This also causes a back pressure when
based sodium silicate. After the investment material is set moulten metal is injected in to the mould. This problem is
Chapter 2  •  Metal Casting and Forming  |  3.799

solved in vacuum die casting. As air is evacuated after clos- melt rapidly and withdrawing the solidified product in a
ing the die, metal enters in to the die much faster. As a result continuous length from the bottom at a rate consistent with
fill time is reduced and blisters are avoided. the pouring rate. Using continuous casting process, slabs,
billets and blooms can be directly cast with going through
Low Pressure Die Casting the rolling of ingot in various stages to obtain the products.
In low pressure die casting molten metal in a crucible rises The skin formed in the water cooled mould is further solidi-
to the mould through a riser tube dipped in the molten metal fied by intensive cooling with water sprays as the casting
when low pressure in the range of 0.3 to 1.5 bar is applied to moves forward.
the molten metal. Since metal enters the mould slowly com-
pared to die casting with less turbulence the casting quality Squeeze Casting
is improved eliminating the defects. In squeeze casting the casting is solidified under high pres-
sure. The product quality is greatly improved. Shrinkage
Centrifugal Casting cavities, dissolved gases etc are eliminated at high pressures.
In centrifugal casting process, the mould is rotated and the It is a combination of casting and forging. Molten metal is
molten metal in the mould is acted up on by the centrigal poured in to a die. Pressure is applied on this metal with a
force. Due to this molten metal is distributed to the periph- punch which is having the inner contour of the product.
ery of the mould cavity. Centrifugal casting process is clas-
sified as Metal Forming Processes
Metal forming processes or mechanical working processes
1. True centrifugal casting
are based on permanent changes in the shape of body or
2. Semi–centrifugal casting
plastic deformation under the action of external forces.
3. Centrifuging process
Mechanical working processes include rolling, forging,
In true centrifugal casting process, the axis of rotation of extrusion, drawing and sheet metal working. Sheet metal
the mould coincides with the axis of casting and due to the working is also known as press working. The stresses
centrifugal force molten metal is thrown out uniformly to induced in the part are greater than the yield strength but
the periphery of the mould cavity. In this case no core is less than the fracture strength except in sheet metal opera-
required for making a concentic hole. Centrifugal casting tions. In sheet metal operations such as shearing, piercing,
can be used for the production of cast iron pipes. The fin- blanking etc stress induced are greater than or equal to the
ished flask rammed with sand inside, with the required con- fracture strength.
tour and end details, is rotated on rollers. The amount of Machining or metal cutting operations are not included
molten metal poured determines the thickness of the pipe in the forming processes. In machining operations mate-
to be cast. rial is removed from the part in the form of chips. But in
The mechanical properties of centrifugally cast objects mechanical forming processes there is no material loss.
are better compared to the other processes because impu-
rities such as slag and oxides get segregated towards the Recrystallisation Temperature
centre. This can be removed easily by machining. During deformation, metal flows plastically and the shapes
In semi–centrifugal casting the mould is rotated about of grains are changed. If the deformation is carried out at
the vertical axis and metal is poured in to central sprue high temperature, new grains are formed in the metal. The
where it first enters the hub and then moves outwards to the process of formation of new grains is known as recrystalli-
periphery by centrifugal force. If central hole is required, sation. The temperature at which formation of new grains is
a core is to be provided. Rotating speed is lesser than that complete is known as recrystallisation temperature. Plastic
is true centrifugal casting. For high rates of production, deformations of a metal above the recrystallisation temper-
moulds can be stacked one over the other and molten metal ature but below the melting temperature is known as hot
can be fed through a common central sprue. working. And plastic deformation below the recrystallisa-
Centrifuging process is similar to semi­–centrifugal cast- tion temperature is known as cold working.
ing process. Metal is fed through a central sprue and the Under the action of heat and force, atoms reach high
mould is rotated. The main difference is that in the case energy level, new crystals starts forming during the recrys-
of centrifuging the axis of the mould is not same as axis tallisation. Recrystallisation destroys old grain structure
of rotation. Here identical small moulds are arranged in a deformed by mechanical working and entirely new crystals,
circle which are connected through radial gates from the which are strain free are formed. Recrystallisation tempera-
central sprue. ture is defined as the approximate minimum temperature
at which complete recrystallisation of a cold worked metal
Continuous Casting occurs with in a specified time.
Continuous casting process consists of pouring into a short Recrystallisation temperature generally varies between
vertical metal die or mould at a controlled rate, cooling the one third and half the melting point for most of the metals.
3.800  |  Part III  •  Unit 7  •  Manufacturing Technology

For lead and tin minimum recrystallisation is below the Rolling is done in both hot and cold. The starting mate-
room temperature. For Cadmium and Zinc it is at the room rial is cast ingots. These are rolled to blooms, billets and
temperature For iron it is 450°C. slabs initially and then hot rolled to plates, sheets, rods etc.
In the rolling process the metal is passed through two
Cold Working rolls rotating in opposite directions at a uniform peripheral
Advantages speed. The space between the rolls can be adjusted to obtain
the desired thickness of the rolled product. The work piece
1. Strength and hardness of the metal is increased due to comes out of the rolls with reduced thickness, but its width
strain hardening. and length is increased.
2. As no oxides are formed on the surface, good surface In the rolling process, because of the squeezing, the
finish is obtained. grains are elongated in the direction of rolling and the
3. Dimensional accuracy achieved is better. velocity of metal is higher than that at the entry. After the
4. It is easier to handle cold parts and is economical for stress zone, the elongated grains start refining in the case of
smaller sizes. hot rolling. The elongated grains remains as it is the case of
cold rolling.
Disadvantages
Roll
1. The amount of deformation that can be given is A
limited due to the higher yield strength and also due α B
to strain hardening.
2. Brittle materials cannot be cold worked.
3. Possibility of crack formation and propagation is Refined grains
great.
Original grains Elongated grains
Hot Working Roll
Advantages
1. Since there is no strain hardening any amount of O
working can be imparted.
2. Even brittle materials can be hot worked. A α
B
3. Only lesser force is required for hot working. P
4. A favourable grain size can be attained, at controlled µP
working conditions. So better mechanical properties The metal contacts each of the rolls along the arc AB. The
can be achieved. angle (a) made by the arc at the centre of the roll is called
5. Blow holes and porosities are eliminated by welding angle of bite or angle of contact. At the moment of bite two
action at high temperature and pressure. forces act on the metal, normal force P and tangential force
mP, where m is the coefft of friction. If the horizontal compo-
Disadvantages
nent of the resultant normal force P and frictional force mP
1. Some metals cannot be hot worked because of their is directed in the direction of rotation, the work piece will be
brittleness at high temperature. dragged in the same direction. In the limiting case,
2. Poor surface finish due to scaling. Psina = mP cosa
3. Because of thermal expansion dimensional accuracy Or m = tan a
is hard to achieve.
4. Handling and maintaining of hot metal is difficult and Or a = tan-1 m
troublesome. If a is greater than tan-1m, the work piece will not be
5. Surface decarbonisation in steels reduces strength passing through the rolls, without aid of external forces.
and hardness of the surface. In hot rolling the primary purpose is to reduce the sec-
tion. So the value a and hence m will be maximum. The roll
Rolling surface can be roughened to increase the value of m. This
The process of shaping metals and alloys into finished or process is called ‘ ragging.’ In the case of cold rolling, roll-
semi finished conditions passing between circular or con- ing loads are high and value of m will be smaller.
toured rotating cylinders (rolls) is called rolling. The metal During the plastic deformation of metal it can be
is drawn in to the opening between rolls by frictional forces assumed that the volume passing per unit time constant. So
between metal and roll surface. The work piece is subjected it can be stated that
to high compressive forces and is plastically deformed. V0 h0 b0 = V1 h 1and b1
Chapter 2  •  Metal Casting and Forming  |  3.801

where V, h and b are velocity, height and breadth Based on no. of rolls in the roll stand it can be two high,
respectively. three high, four high, multi roll universal rolling mill and
But b0 ~ b1 planetary rolling mills.
Based on products, it can be blooming and slabbing
So V1 = V0 h0 mills, billet mills, rail and structural mills, rod mills, plate
h1 and sheet mills, seamless tube mills etc.
As h0 > h1, V1 > V0 Based on arrangement of rolling stands, it can be looping
mills cross country mills, continuous mills etc.
If Vr is the velocity of the roll V1 > Vr > V0
V1 - Vr Roll Passes
× 100 is called forward ship
Vr Bars, rods and special purpose sections (I beam, channels,
rails etc) are rolled between grooved rolls. The grooves cut
Vr − V0
and × 100 is called backward ship on mating rolls will form passes through which the metal
Vr is passed to get the required cross section. Before getting
In the deforming area between the rolls velocity of metal is the final stage the work passes through many passes. Roll
changing from V0 to V1. At a particular section this equal to passes are classified as
Vr, the velocity of roll. This section is called neutral or no 1. Break down or roll down or roughing passes
slip section. 2. Leader passes
Other parameters are 3. Finishing passes
Absolute draught Dh = h0–h1 mm
Break down passes are intended to reduce the cross sec-
∆h tional area.
Relative draught × 100
h0 Leader passes gradually brings the cross section near to
Absolute elongation Dl = L1–L0 mm the final shape.
Finishing pass is for the final or the required cross
L1 section.
Coefficient of elongation =
L0
Absolute spread = b1 –b0 mm Rolling Defects
A0 − A1 Defects found in rolling process are
% of cold work = × 100 1. Wavy edges
A0
2. Spread
where A = area of cross section. 3. Crocodile neck
h0 − h1 4. Surface defects
5. Edge cracks
It can be shown that cos a = 1 − 2
R Wavy edges result from roll bending. The strip becomes
∆h thinner along edges corresponding to the centre thickness.
or cos a = 1 −
D As free expansion is restrained, buckling occurs. If the
where a = angle of contact width to thickness ratio of plates and sheets are smaller,
D = diameter of the roll width increases considerably in the roll gap. This is called
Maximum possible draught D hmax = m2R spreading.
If the plate is weak at the centre, sheet bifurcates into
D
Where R = two causing neck formation. This is called crocodile neck.
2 Presence of impurities such as scale, rust, dirt in hot bloom,
billets or slabs causes surface defects in the rolled prod-
Rolling Mills ucts. Limited ductility of the metal or uneven deformation
A set of rolls assembled in a housing is called a rolling at edges will cause edge cracks.
stand. A rolling mill may contain one or more rolling stands. Example 9:  A stock of thickness 30 mm is to be rolled to
A rolling mill is a place where metal rolling is done on rolls 10mm in a single stage. Calculate the minimum diameter of
and other auxiliary operations are performed. the rolls, if the maximum angle of bite is 40°C. Find also
the required coefficient of friction.
Classification of Rolling Mills
Solution:  Dh = h1–h2
1. Classification based on number of rolls in the stand
2. Based on products rolled 30–10 = 20 mm
3. Based on arrangement of rolling stands a = 40°
3.802  |  Part III  •  Unit 7  •  Manufacturing Technology

Basic Forging Operations


Cos a = 1 − ∆h
D 1. Upsetting: Upsetting is the process of increasing the
20 cross sectional area of the stock at the expense of
Cos 40 = 1 – its length by application force in the direction of its
D
length axis.
20
= 1 − cos 40 2. Drawing down: Drawing down is the operation of
D
reducing thickness of the stock to increase its length.
= 0.234
Force is applied in a direction perpendicular to the
20 length axis. Thickness is reduced at the end of the
D= = 85.49 mm
0.234 stock. Also known as cogging.
Example 10: A sheet of 4 mm thickness is rolled with 3. Fullering is the operation reducing thickness of a
300 mm diameter rolls to reduce the thickness without any stock between ends, at a central place. This is done
change in its width. The friction coefficient at the work roll with the help of tools called fullers.
interface is 0.1. Calculate the minimum possible thickness
4. Edging or rolling operation is to distribute the metal
of the sheet that can be produced in a single pass.
longitudinaly from a thick portion to a portion where
Solution:  Given, material is deficient.
hi = 4 mm 5. Heading is an upsetting operation where thickness is
300 increased only at on end.
R= = 150 m
2 6. Blocking It is a forging operation in which the work
m = 0.1 piece is obtained its general shape, prior to its final
hi–hmin = maximum draft = m2 R shape.
=(0.1)2 × 150 mm = 1.5 7. Piercing is the operation done with the help of punch
hmin = hi–1.5 to obtain blind or through holes in the metal.
= 4–1.5 = 2.5 mm. 8. Punching is the operation of shearing out a slug in a
forging to produce a hole.
Forging
9. Swaging is the operation of reducing the cross
Forging is the process of shaping a material under compres-
sectional area of a circular piece. by rotating it and
sive forces in dies by plastic deformation normally after
giving fast impact blows. Bending, flattening, cutoff
heating. The material is heated to a temperature at which
coining etc are other forging operations.
its elastic properties entirely d­ isappear and obeys the laws
of plastic flow, following the directions of least resistance Forging Processes
when deformed under pressure.
Forging processes can be classified in to two groups.
Forging enhances mechanical properties of metals and
improves the grain flow due to which strength and tough- 1. Open die forging
ness of the forged component is increased. 2. Closed die forging
Forging temperature of some common metals are given
below. Open Die Forging
In open die forging the work piece is struck or pressed
Material Forging starting Temperature finishing
between two flat surfaces. This process is used when the
Mild steel 1300 800 components to be forged are only a few. Flat–die forging is
Wrought iron 1275 900 another name of open die forging. Open die forging can be
Medium carbon 1250 750 further classified as
steel
1. Hand forging
High carbon 1150 825 2. Power forging
steel
3. Hammer forging
Copper, brass, 950 600 4. Press forging
bronze
Hand forging or Smith forging is usually done in a black-
Al and Mn 500 350
Alloys
smith’s hearth with the help of small hammers and anvil, to
produce small number of light forgings.
Chapter 2  •  Metal Casting and Forming  |  3.803

Power forging is used for producing large components (250)­2 × 60 = (450)2 × L2


which cannot be forged by hand. Machines which work on \ L2 = 18.5 mm
forging by blow are called hammers; while those working Change in length
by pressure are called presses. Hammers can be mechanical, = L1–L2
pneumatic, steam or air hammers. = 60–18.5 = 41.5 mm
L1 − L2 41.5
Closed Die Forging Strain = = = 2.24
L2 18.5
In closed die forging cavities or impressions are cut in the
die block, where metal is forced to take the final shape. True strain = Ln (1 + e)
The dies can be single impression type or multi impression = Ln (1 + 2.24) = 1.176.
type.
Impression-die forging is used to make complex shapes Extrusion
of products with greater accuracy. After forging the product Extrusion is the process by which a block of metal is reduced
should be trimmed to remove flashes. in cross section by forcing it to flow through a die orifice
under high pressure. High pressure is applied by hydraulic
Types of Forging press or mechanical press.
The four types of forging methods generally used are Extrusion process can be hot extrusion or cold extrusion
depending up on the temperature at which the extrusion is
1. Smith forging done.
2. Drop forging Based on the direction of flow of metal and application
3. Press forging of force extrusion can be classified as
4. Machine forging
1. Direct or forward extrusion
In drop forging closed type dies and drop hammers are 2. Indirect or backward extrusion
used. Using drop hammers repeated blows are given to the 3. Impact extrusion
material in the die cavity. 4. Side extrusion
In press forging the dies are similar to that of drop forg- 5. Tube extrusion
ing. But instead of repeated blows by hammer a single con-
tinuous squeezing action is given to the material by means Direct or Forward Extrusion
of hydraulic presses. Because of the continuous action the In this case the billet is placed inside the container and
material gets uniformly distributed through the cavity. force through the die with the help of pressure applied by a
Machine forging is also known as upset forging as the hydraulic driven ram.
operation involved is upsetting. Upsetting machines are
also called upsetters. Upsetters were originally developed
for making bolt heads in a continuous fashion. Now gear
blanks, shafts, axles and similar parts are also produced Plunger(Ram)
using machine forging. Upsetting machines are generally
horizontal acting.

Defects in Forged Parts


Defects may occur in forged parts due to various fac- Billet
tors such as forging process, improper heating, incor-
rect die design, uneven cooling after forging etc. Defects Direct extrusion
are similar to those in casting. These are mismatch, scale Die
pits, cold shuts or Laps, unfilled section, cracks, fins and
rags etc.

Example 11:  In an open die forging a disc of 250 mm


diameter and 60 mm height is compressed without any Indirect or Backward Extrusion
barelling effect. The final diameter of the disc is 450 mm. It is similar to the direct extrusion except that the extruded
The true strain is part is forced through the ram stress. The deformed metal
flows through the die opening in the direction opposite to
p 2 p that of ram motion.
Solution:  D1 L1 = D22 L2
4 4
3.804  |  Part III  •  Unit 7  •  Manufacturing Technology

Ram Tube Extrusion


Die Extrusion is one of the methods for producing seamless
tubes. It is a form of direct extrusion but a mandrel is used
to form the inside of the tube. After placing the billet inside
the cylinder, die containing the mandrel is pushed through
Billet it. When force is applied on the press stem it advances
extruding the metal through the die and around the mandrel
forming the seam less tube.
Billet

Hydrostatic Extrusion
Hydrostatic extrusion is a cold extrusion process, in which
Ram billet is surrounded by a working fluid, which is pressurised
by ram. To provide the extrusion force. Friction between bil-
In indirect extrusion, the billet inside the cylinder has no let and container is elimimated in this case and this makes it
relative motion with the cylinder. So there is no friction possible to extrude very long billets.
between cylinder and billet. Therefore power required for Extrusion ratio is defined as the ratio of cross sectional
indirect extrusion is less compared to direct extrusion. But area of the bullet to the cross sectional area of the product.
the long hollow ram is required and this limits the loads that
can be applied.
Extrusion Pressure
Impact Extrusion Estimation of extrusion pressure in many cases is done with
It is a cold extrusion process used for soft metals (like alu- the help of empirical relation ships. One such relation for
minium). It can be backward or forward. In backward impact calculating maximum pressure of backward extrusion of
extrusion, metal flows in reverse direction of the plunger. carbon steel is
The flowing metal is guided only initially. Afterwards it  A 
moves by its on inertia. Impact extrusion is carried out at P = T 3.45ln 0 + 1.15 kw /mm 2
 Ab 
higher speeds. The punch strikes a single blow with con-
siderable force causing the metal to squirt up against the
Where T = upper yield point, kN/mm2
punch.
A0 = cross sectional area of extruded component
Application of impact extrusion process are in the manu-
Ab = cross sectional area of billet
facture of collapsible tubes for tooth paste, ointments, shell
The above expression is valid for extrusion ratios vary-
cases, causes etc.
ing from 1.65 to 4.25 using billets with 0.6 length to diam-
Combined forward and backward extrusion can be
eter ratio.
used to produce complex shapes. In the same stroke of the
Another expression for extrusion force is
plunger backward and forward extrusion happens.
 A1 
Extrusion force = kA1 ln  
 Af 
Where k = extrusion constant
A1 = initial area of cross section
Af = final area of cross section

Drawing of  Wire, Rod and Tube


Drawing is a cold working process, used to reduce the diam-
eter of wires, rods and tubes by drawing through a tapered
hole in a die.
Backward impact extrusion Wire is made by cold drawing of hot rolled wire rod
through one or more dies. Metals blow 16 mm diameter are
Side Extrusion handled in coil form. The end of the rod or wire to be drawn
In side extrusion the movement of the material is in a direc- is made pointed by swaging or hammering so that it freely
tion perpendicular to that of the ram motion. In this case the enters the die orifice and sticks out behind the die. This end
force required is very high and therefore mostly used for is gripped by jaws of a gripper and pulled. Under the ten-
non-ferrous metals or highly plastic materials. sile force material inside die undergoes deformation and its
cross sectional area is reduced.
Chapter 2  •  Metal Casting and Forming  |  3.805

In rod drawing the product is to be straight. Maximum Bulge formation occurs infront of the die due to high die
length of the rod that can be drawn depends upon the maxi- angle and low reduction.
mum travel of the carriage which the pulls the rod through Internal cracks or central burst occur due to increase
the die. Before drawing the rod is to be cleaned to remove in die angle or increase in the amount of impurities. It
scales etc. This is done by acid pickling. After pickling the decreases with increasing drawing ratio and friction.
metal is washed well and conditioned by sulling, coppering, Surface defects occur due to inadequate lubrication and
phosphating, liming etc to have to have proper lubrication. improper selection of drawing methods.
In sulling or yellowing the blank is given a thin coating Longitudinal scratch or fold in materials is called seams.
of iron hydroxide Fe(OH)3, which combines with lime to
serve as a filler for the lubricant. Phosphating consists of Drawing Force
applying coating of phosphates of Mn, Fe or Zn. Lubricant Force required for drawing under frictionless conditions is
sticks to the phosphates coatings. In coppering the metal is given by the following expression.
dipped in a weakly acidified solution of copper sulphate. In
 Ao 
liming the metal is dipped in a boiling lime solution. Acid F = Yav Af ln
A 
remains are neutralized by liming and forms as a carrier for  f
lubricant. Before drawing the metal is dried above 100°C. where A0 = original area of cross section of the wire or
A suitable lubricant is applied on the dry surface. Drawing rod
lubricants used are mineral and vegetable oils, animal fats, Af = final area of cross section
graphite, certain emulsions etc. Considerable heat is gener- Yav = average true stress of the material in die gap
ated during drawing. Water is circulated around the die to Maximum reduction in diameter
remove heat.
2 1/ B
Tube drawing also is similar to the other drawing pro-  Df   1 
R=1-  = 1 −  
cesses. The main difference is that a mandrel is used to form  Do  1+ B
the internal hole.
Where Df = final diameter
Degree of Drawing D0 = original diameter
Degree of drawing is measured as the reduction of area µ
(RA), which is the ratio of the difference in cross sectional B=
tan α
area before and after drawing to the initial cross sectional
area. where m = coefficient of friction
2 a = semi die angle
Di2 − D02 D 
i.e. RA = = 1– 0
Di2  D  Hot Drawing and Cupping
i
Defects in Wire Drawing This is another method of producing seamless tubes and
In wire drawing operations the following defects are cylinders. A thick metal blank of circular shape is hot
observed pierced by a hydraulically operated plunger to form a cup
shaped product first. This is drawn through a series of dies
1. Bulge formation by pushing with plungers to reduce diameter and increase
2. Internal cracks the length. This method is generally used for producing
3. Surface defects thick walled cylindrical products.
4. Seams

Exercise
Practice Problems 1 2. The top diameter of a down sprue is 20 mm. Its length is
180 mm. Liquid metal in the pouring up is maintained
1. A rectangular block of dimensions 100 × 50 × 20 mm is up to 60 mm height. For ensuring flow with out aspira-
to be made from cast iron by casting process. For this a tion, the diameter of down sprue at lower end will be
wooden pattern is to be made treating 20 mm as verti- (A) 16.26 mm (B) 14.15 mm
cal. Assuming a machining allowance of 2 mm, shrink- (C) 12.38 mm (D) 15.92 mm
age allowance of 2% and a draft allowance of 1°, find
3. A 350 mm thick slab is to be cold rolled using rolls of
the bottom dimensions and height of the pattern in mm.
(A) 55.9 × 106.9, 25.3 650 mm diameter. If coefficient of friction is 0.08, the
(B) 55.9 × 106.9, 24.5 maximum possible reduction in mm thickness will be
(C) 55.1 × 106.1, 25.3 (A) 1.18 (B) 2.28
(D) 55.1 × 106.1, 24.5 (C) 1.32 (D) 2.08
3.806  |  Part III  •  Unit 7  •  Manufacturing Technology

4. Large size bolt heads are produced by thickness of 3 mm in a single pass rolling mill having rolls
(A) Swaging (B) Roll forging of 300 mm diameter. The strip is 450 mm wide. The average
(C) Upset forging (D) Tumbling coefficient of friction in the roll gap is 0.08. Assume plane
5. Seamless tubes in mass production are manufactured strain flow stress of 140 MPa for the metal and negligible
by spreading.
(A) Rolling (B) Spinning 13. The average roll gap pressure will be
(C) Welding (D) Extrusion (A) 161. 56 M Pa (B) 157. 23 M Pa
6. Needle is produced by (C) 169. 33 M Pa (D) 152. 88 M Pa
(A) Forging (B) Machining 14. The roll separating force will be
(C) Extrusion (D) Swaging (A) 836.7 kN (B) 832.5 kN
7. Frictional force between billet and die is more in the (C) 810.8 kN (D) 890.6 kN
case of 15. A grey cast iron casting is to be made using a wooden
(A) Direct extrusion pattern. The shrinkage allowance allowed on the
(B) Indirect extrusion wooden pattern should be
(C) Impact extrusion (A) 10 mm/m (B) 16 mm/m
(D) Hydrostatic extrusion (C) 26 mm/m (D) 20 mm/m
8. Out of the following the metal that cannot be forged 16. Chills are used in casting moulds to
is (A) Reduce the freezing time
(A) Wrought iron (B) Achieve directional solidification
(B) Mild steel (C) Reduce hot tear
(C) Cast iron (D) Improve surface finish
(D) High carbon steel 17. Upsetting is a process of
9. Wire cannot be hot rolled commercially/economically (A) Increasing cross section of a bar
smaller than about a diameter of (B) Reducing cross section of a bar
(A) 10 mm (B) 5 mm (C) Bending of a bar
(C) 15 mm (D) 2 mm (D) Joining two pieces of bars
10. Out of the following which one is not connected with 18. Collapsible tubes are made by
tube making (A) Cold extrusion forging
(A) Hot cupping (B) Impact extrusion
(B) Pilgering (C) Forward hot extrusion
(C) Three-roll piercing (D) Backward hot extrusion
(D) Rotary swaging 19. A two–high rolling mills consists of
11. By a 10 ton press, it is meant that (A) Two rolling stands at different heights
(A) The weight of press is 10 ton (B) Two rollers of equal diameter rotating in same di-
(B) It can handle work weighing up to 10 ton rection
(C) It can exert force up to 10 ton (C) Two rollers of equal diameter rotating in opposite
(D) Turn over per day is 10 mton directions
12. Skelton patterns are generally used for (D) One set of rolls with two back up rolls
(A) Small castings 20. Mandrel is used in tube drawing
(B) Non-ferrous castings (A) To form the internal hole
(C) Large castings (B) To give good surface finish
(D) Hollow castings (C) To aid in pulling of the tube.
Direction for question 13 and 14:  A metal strip is to (D) To provide lubrication
be rolled from initial thickness of 4 mm to a final rolled

Practice Problems 2 2. Property of sand due to which a great amount of steam


and other gases are released is called
1. A pattern maker’s shrinkage rule considers (A) Collapsibity
(A) All pattern allowances (B) Permeability
(B) Only shrinkage allowance (C) Cohesiveness
(C) All materials to be cast (D) Flowabilitry
(D) All materials of the pattern
Chapter 2  •  Metal Casting and Forming  |  3.807

3. When a hole or cavity to be cored is not in line with the to atmosphere and an acceleration due to gravity of
parting suface, the core to be used is 104 mm/s2, the area of down sprue in mm2 at its end
(A) Horizontal core (B) Vertical core (avoiding aspiration effect) should be
(C) Drop core (D) Balanced core (A) 310.3 mm2 (B) 281.2 mm2
(C) 290.7 mm 2 (D) 268.3 mm2
4. Maximum possible draft in cold rolling of a sheet
increases with 13. A cubical casting of side 45 mm undergoes 3.5%, 4.5%

(A) Increase in coefficient of friction and 5% volume shrinkage, during liquid state, phase
(B) Decrease in coefficient of friction transformation and solid state respectively, while cool-
(C) Decrease in radius of roll ing. The volume of metal compensated from the riser is
(D) Not related to radius of role (A) 9.5% (B) 8.5%
5. In a 4 high rolling mill there are four rolls out of which (C) 8% (D) 13%
(A) One is working roll and 3 are backing up rolls 14. In a rolling process a sheet of 30 mm thickness is rolled

(B) Two are working rolls and two are backing up rolls to 25 mm thickness. Roll is of diameter 700 mm. and it
(C) Three are working rolls and one is backing up roll rotates at 100 rpm. The roll strip contact length will be
(D) All four rolls are working rolls (A) 48.5 mm (B) 38.7 mm
6. Metal extrusion process is generally used for producing (C) 41.9 mm (D) 36.3 mm
(A) Uniform solid sections only. 15. A brass billet is to be extruded from its initial diam-

(B) Uniform hollow section only eter of 100 mm to final diameter 60 mm. The work-
(C) Uniform solid and hollow sections ing temperature is 700°C and the extrusion constant is
(D) Varying solid and hollow sections 250 MPa. The force required for extrusion is
7. A solid cylinder of diameter 100 mm and height 50 mm (A) 12.97 kN (B) 11.46kN
is forged between two frictionless flat dies to a height (C) 16.92 kN (D) 10.13 kN
of 25 mm. The percentage change in diameter is 16 In a wire drawing operation, diameter of a steel wire is
(A) 35.6 (B) 41.4 reduced from 12 mm to 10 mm. The mean flow stress
(C) 43.7 (D) 38.8 of the material is 400 Mpa. The ideal force, ignoring
8. Cold working of metals is carried out friction and redundant work is
(A) Below lower critical temperature (A) 12.97 kN (B) 11.46 kN
(B) Below upper critical temperature (C) 16.92 kN (D) 10.13 kN
(C) Below recrystallization temperature Direction for questions 17, 18  A cylindrical billet of 100
(D) Below 15°C mm diameter is forged from 60 mm height at 1000°C. The
9. Fullers are used material has a constant flow stress of 80 MPa
(A) For finishing flat surfaces 17. Find the work of deformation in Nm.
(B) For necking down a piece of work (A) 31400 (B) 30210
(C) For punching a hole (B) 25120 (D) 28105
(D) Bending of a bar
18. If a 10 kN drop hammer is used to complete the reduc-
10. Thick walled cylinder can be produced by tion in one blow, what will be the height of the fall?
(A) Hot drawing (A) 2.51 m (B) 2.82 m
(B) Hot extrusion (C) 3.14 m (D) 2.73 m
(C) Cold drawing
19. A mould has down sprue whose length is 25 cm and
(D) Continuous drawing
cross sectional area at the base 1.5 cm2. The down
11. In a foundry, two castings of same weight and material
sprue feeds a horizontal runner leading to mould cav-
due to be produced. One is cube shaped and the other
ity of volume 1500 cm3. The time required to fill the
is cylindrical shaped with a length to diameter ratio of
mould cavity will be
0.5. Cooling time ratio of cylindrical and cubical cast-
(A) 3.61 sec (B) 5.05 sec
ing is
(C) 4.51 sec (D) 5.81 sec
(A) 0.95 (B) 1.05
(C) 1.15 (D) 1.22 20. In a rolling operation a 25 mm thick plate of width
100 mm is reduced to 20 mm thickness. Roller radius
12. A 200 mm long down sprue has an area of cross section
is 300 mm and rotational speed is 10 rpm. The aver-
600 mm2 where pouring basin meets the sprue. A con-
age flow stress for the material is 300 MPa. The power
stant head of molten metal is maintained by the pour-
required for rolling operation in kw is approximately
ing basin. The molten metal flow rate is 6 × 105 mm3/s.
(A) 365 (B) 472
Considering the end of the down sprue to be open
(C) 343 (D) 387
3.808  |  Part III  •  Unit 7  •  Manufacturing Technology

21. A cubic casting of 60 mm side undergoes volumetric (C) Extrusion


solidification shrinkage and volumetric solid contrac- (D) Hot piercing
tion 3% and 5% respectively no riser is used. The side 26. Metal patterns are generally used for
of the cube after the solidification and contraction will (A) Small castings
be (assume uniform cooling in all direction (B) Medium castings
(A) 58.98 mm (B) 59.65 mm (C) Complicated castings
(C) 59.32 mm (D) 58.39 mm (D) Mass productions of castings
22. During hot working of metals 27. As per the general colour code, surface to be left un
(A) Porosity of metal is largely eliminated machined is marked on a pattern by
(B) Grain structure of the metal is refined (A) Red colour (B) Black colour
(C)  Mechanical properties are improved due to (C) Yellow colour (D) Blue colour
refinement of grains
28. Mass of a grey cast iron casting is 20 Kg and its aver-
(D) All of the above
age section thickness is 15 mm. Fluidity of iron is 28
23. Structurals of sections such as rails, angles, I–beam etc inches. Optimum pouring time for the casting is
are made by (A) 7.63 sec (B) 8.58 sec
(A) Hot rolling (B) Hot drawing (C) 9.32 sec (D) 6.75 sec
(C) Hot piercing (D) Hot extrusion
29. In the case of a pressurized gating system
24. Forging temperature of medium carbon steel is (A) A sprue base area is the smallest
approximately (B) Runner cross section area is the smallest.
(A) 950°C to 1300°C (C) Ingate area is the smallest.
(B) 750°C to 1250°C (D) Ingate area is the largest.
(C) 800°C to 1100°C
3 0. The term ‘shape factor’ is used in
(D) 900°C to 1250°C
(A) Riser design
25. Pressing a hot metal inside a chamber and forcing it out (B) Pouring time calculation
by high pressure through an orifice to form a product is (C) Pattern design
called (D) Core design
(A) Hot drawing
(B) Pressing

Previous Years’ Questions


1. Misrun is a casting defect which occurs due to 4. Match the following: [2004]
 [2004]
Product Process
(A) Very high pouring temperature of the metal
(B) Insufficient fluidity of the molten metal P.  Molded luggage 1.  Injection molding
(C) Absorption of gases by the liquid metal Q.  Packaging 2. Hot rolling containers
(D) Improper alignment of the mould flasks for liquid
R. Long structural 3.  Impact extrusion
2. Gray cast iron blocks 200 × 100 ×10 mm are to be cast
shapes
in sand moulds. Shrinkage allowance for pattern mak-
S.  Collapsible tubes 4.  Transfer molding
ing is 1%. The ratio of the volume of pattern to that of
the casting will be [2004] 5.  Blow molding
(A) 0.97 (B) 0.99 6.  Coining
(C) 1.01 (D) 1.03 (A) P-1 Q-4 R-6 S-3
3. In a rolling process, sheet of 25 mm thick- (B) P-4 Q-5 R-2 S-3
ness is rolled to 20 mm thickness. (C) P-1 Q-5 R-3 S-2
Roll is of diameter 600 mm and it rotates at 100 rpm. (D) P-5 Q-1 R-2 S-2
The roll strip contact length will be [2004] 5. Match the items of List I (Equipment) with the items
(A) 5 mm (B) 39 mm of List II (Process) and select the correct answer using
(C) 78 mm (D) 120 mm the given codes. [2005]
Chapter 2  •  Metal Casting and Forming  |  3.809

List (Equipment) List (Process) 11. In a wire drawing operation, diameter of a steel wire
P – Hot chamber machine 1.  Cleaning
is reduced from 10 mm to 8 mm. The mean flow stress
of the material is 400 MPa. The ideal force required
Q – Muller 2.  Core making
for drawing (ignoring friction and redundant work) is:
R – Dielectric baker 3.  Die casting  [2006]
S – Sand blaster 4.  Annealing (A) 4.48 kN (B) 8.97 kN
5.  Sand mixing (C) 20.11 kN (D) 31.41 kN
(A) P – 2 Q – 1 R – 4 S – 5 12. Which of the following engineering materials is the
(B) P – 4 Q – 2 R – 3 S – 5 most suitable candidate for hot chamber die casting?
(C) P – 4 Q – 5 R – 1 S – 2  [2007]
(D) P – 3 Q – 5 R – 2 S – 1 (A) Low carbon steel (B) Titanium
(C) Copper (D) Tin
6. A mould has a downsprue whose length is 20 cm and
the cross sectional area at the base of the downsprue is 13. Volume of a cube of side ‘I’ and volume of a sphere of
1 cm2. The down sprue feeds horizontal runner lead- radius ‘r’ are equal. Both the cube and the sphere are
ing into the mould cavity of volume 1000 cm3. The solid and of same material. They are being cast. The
time required to fill the mould cavity will be [2005] ratio of the solidification time of the cube to the same
(A) 4.05 s (B) 5.05 s of the sphere is  [2007]
3 2
(C) 6.05 s (D) 7.25 s
(A) ⎛ 4p ⎞ ⎛ r ⎞ (B) 4p
⎛ ⎞⎛ ⎞ r
⎜⎝ ⎟⎠ ⎜⎝ ⎟⎠ ⎜⎝ ⎟⎠ ⎜⎝ ⎟⎠
7. A 2 mm thick metal sheet is to be bent at an angle 6 l 6 l
of one radian with a bend radius of 100 mm. If the 2
⎛ 4p ⎞ ⎛ r ⎞ (D)
3
⎛ 4p ⎞ ⎛ r ⎞
2 4
stretch factor is 0.5, the bend allowance is [2005] (C)
⎜⎝ ⎟⎠ ⎜⎝ ⎟⎠ ⎜⎝ ⎟⎠ ⎜⎝ ⎟⎠
6 l 6 l

2 mm
14. In open-die forging, a disc of diameter 200 mm and
height 60 mm is compressed without any barreling
1 Radian effect. The final diameter of the disc is 400 mm. The
true strain is [2007]
(A) 1.986 (B) 1.686
(C) 1.386 (D) 0.602
(A) 99 mm (B) 100 mm 15. The thickness of a metallic sheet is reduced from an
(C) 101 mm (D) 102 mm initial value of 16 mm to a final value of 10 mm in one
8. An expendable pattern is used in [2006] single pass rolling with a pair of cylindrical rollers
(A) Slush casting each of diameter of 400 mm. The bite angle in degree
(B) Squeeze casting will be [2007]
(C) Centrifugal casting (A) 5.936 (B) 7.936
(D) Investment casting (C) 8.936 (D) 9.936
9. In a sand casting operation, the total liquid head is 16. A 200 mm long down sprue has an area of cross-­
maintained constant such that it is equal to the mould section of 650 mm2 where the pouring basin meets the
height. The time taken to fill the mould with a top gate down sprue (i.e., at the beginning of the down sprue).
is tA. If the same mould is filled with a bottom gate, A constant head of molten metal is maintained by the
then the time taken is tB. Ignore the time required to pouring basin. The molten metal flow rate is 6.5 × 105
fill the runner and frictional effects. Assume atmos- mm3/s. Considering the end of down sprue to be open
pheric pressure at the top molten metal surfaces. The to atmosphere and an acceleration due to gravity of
relation between tA and tB is: [2006] 104 mm/s2, the area of the down sprue in mm2 at its
end (avoiding aspiration effect) should be [2007]
(A) tB = 2 tA (B) tB = 2tA
t
(C) t B = A (D) tB = 2 2 tA Pouring
2 Basin
10. A 4 mm thick sheet is rolled with 300 mm diameter rolls
to reduce thickness without any change in its width. The 650 mm2
friction coefficient at the work–roll interface is 0.1. The
Area of down
minimum possible thickness of the sheet that can be pro- 200mm
sprue at its end
duced in a single pass is: [2006]
(A) 1.0 mm (B) 1.5 mm
(C) 2.5 mm (D) 3.7 mm
3.810  |  Part III  •  Unit 7  •  Manufacturing Technology

(A) 650.0 (B) 350.0 24. A cubic casting of 50 mm side undergoes volumet-
(C) 290.7 (D) 190.0 ric solidification shrinkage and volumetric solid con-
17. While cooling, a cubical casting of side 40 mm under- traction of 4% and 6% respectively. No riser is used.
goes 3%, 4% and 5% volume shrinkage during the liq- Assume uniform cooling in all directions. The side
uid state, phase transition and solid state, respectively. of the cube after solidification and contraction is.
The volume of metal compensated from the riser is  [2011]
 [2008] (A) 48.32 mm (B) 49.90 mm
(A) 2% (B) 7% (C) 49.94 mm (D) 49.96 mm
(C) 8% (D) 9% 25. A solid cylinder of diameter 100 mm and height 50
18. In a single pass rolling operation, a 20 mm thick plate mm is forged between two frictionless flat dies to a
with plate width of 100 mm, is reduced to 18 mm. height of 25 mm. The percentage change in diameter
The roller radius is 250 mm and rotational speed is is [2012]
10 rpm. The average flow stress for the plate material (A) 0 (B) 2.07
is 300 MPa. The power required for the rolling opera- (C) 20.7 (D) 41.4
tion in kW is closest to  [2008] 26. In a single pass rolling process using 410 mm diame-
(A) 15.2 (B) 18.2 ter steel rollers, a strip of width 140 mm and thickness
(C) 30.4 (D) 45.6 8 mm undergoes 10% reduction of thickness. The
19. Two streams of liquid metal, which are not hot enough angle of bite in radians is [2012]
to fuse properly, result into a casting defect known as (A) 0.006 (B) 0.031
 [2008] (C) 0.062 (D) 0.600
(A) Cold shut (B) Swell 27. A cube shaped casting solidifies in 5 min. The solidi-
(C) Sand wash (D) Scab fication time in min for a cube of the same material,
20. Match the items in Column I and Column II. [2009] which is 8 times heavier than the original casting, will
be [2013]
Column I Column II (A) 10 (B) 20
(P)  Metallic chills (1)  Support for the core (C) 24 (D) 40
(Q)  Metallic chaplets (2) Reservoir of the molten 28. In a rolling process, the state of stress of the material
metal undergoing deformation is [2013]
(R)  Riser (3) Control cooling of critical (A) Pure compression
sections (B) Pure shear
(S) Exothermic (4) Progressive (C) Compression and shear
padding solidification (D) Tension and shear
29. An aluminium alloy (density 2600 kg/m ) casting is
3
(A) P-1, Q-3, R-2, S-4
(B) P-1, Q-4, R-2, S-3 to be produced. A cylindrical hole of 100 mm diam-
(C) P-3, Q-4, R-2, S-1 eter and 100 mm length is made in the casting using
(D) P-4, Q-1, R-2, S-3 sand core (density 1600 Kg/m3). The net buoyancy
force (in newton) acting on the core is _____ [2014]
21. In a gating system, the ratio 1: 2: 4 represents [2010]
(A) Sprue base area: runner area: ingate area 30. With respect to metal working, match Group A with
(B) Pouring basin area: ingate area: runner area Group B: [2014]
(C) Sprue base area: ingate area: casting area Group A Group B
(D) Runner area: ingate area: casting area (P)  Defect in extrusion (I)  Alligatoring
22. The maximum possible draft in cold rolling of sheet (Q)  Defect in rolling (II)  Scab
increases with the [2011] (R)  Product of skew rolling (III)  Fish tail
(A) Increase in coefficient of friction (S) Product of rolling (IV)  Seamless tube
(B) Decrease in coefficient of friction through cluster mill
(C) Decrease in roll radius (V)  Thin sheet with tight tolerance
(D) Increase in roll velocity (VI) Semi-finished balls of ball
23. Green sand mould indicates that [2011] bearing
(A) Polymeric mould has been cured
(A) P - II, Q - III, R - VI, S – V
(B) Mould has been totally dried
(B) P - III, Q - I, R - VI, S - V
(C) Mould is green in colour
(C) P - III, Q - I, R - IV, S - VI
(D) Mould contains moisture
(D) P - I, Q - II, R - V, S - VI
Chapter 2  •  Metal Casting and Forming  |  3.811

31. A mild steel plate has to be rolled in one pass such that 37. The solidification time of a casting is proportional to
2
the final plate thickness is 2/3rd of the initial thickness, ⎛V ⎞
with the entrance speed of 10 m/min and roll diameter ⎜⎝ ⎟⎠ , where V is the volume of the casting and A is
A
of 500 mm. If the plate widens by 2% during rolling, the the total casting surface area losing heat. Two cubes
exit velocity (in m/min) is ____ [2014] of same material and size are cast using sand cast-
32. Match the casting defects (Group A) with the prob- ing process. The top face of one of the cubes is com-
able causes (Group B): [2014] pletely insulated. The ratio of the solidification time
for the cube with top face insulated to that of the other
Group A Group B
cube is: [2015]
P:  Hot tears 1. Improper fusion of two streams of liquid metal 25 36
Q:  Shinkage 2. Low permeability of the sand mould
(A) (B)
36 25
3. Volumetric contraction both in liquid and
R:  Blow holes
solid stage
6
(C) 1 (D)
S:  Could Shut 4.  Differential cooling rate 5
38. In a slab rolling operation, the maximum thickness
(A) P-1, Q-3, R-2, S-4 (B) P-4, Q-3, R-2, S-1 reduction (∆hmax) is given by ∆hmax = µ2R, where R is
(C) P-3, Q-4, R-2, S-1 (D) P-1, Q-2, R-4, S-3 the radius of the roll and µ is the coefficient of friction
3 3. The hot tearing in a metal casting is due to [2014] between the roll and the sheet. If µ = 0.1, the maxi-
(A) High fluidity mum angle subtended by the deformation zone at the
(B) High melt temperature centre of the roll (bite angle in degrees) is ______.
(C) Wide range of solidification temperature  [2015]
(D) Low coefficient of thermal expansion 39. A cube and a sphere made of cast iron (each of vol-
34. In a rolling process, the maximum possible draft, ume 1000 cm3) were cast under identical conditions.
defined as the difference between the initial and the The time taken for solidifying the cube was 4s. The
final thickness of the metal sheet, mainly depends on solidification time (in s) for the sphere is ______.
which pair of the following parameters? [2014]  [2015]
(P) Strain 40. In a two-stage wire drawing operation, the fractional
(Q) Strength of the work material reduction (ratio of change in cross-sectional area to
initial cross-sectional area) in the first stage is 0.4.
(R) Roll diameter
The fractional reduction in the second stage is 0.3.
(S) Roll velocity The overall fractional reduction is: [2015]
(T) Coefficient of friction between roll and work (A) 0.24 (B) 0.58 (C) 0.60 (D) 1.00
(A) Q, S (B) R, T 41. The strain hardening exponent n of stainless steel SS
(C) S, T (D) P, R 304 with distinct yield and UTS values undergoing
35. A cylindrical riser of 6 cm diameter and 6 cm height plastic deformation is [2015]
has to be designed for a sand casting mould for pro- (A) n < 0 (B) n = 0
ducing a steel rectangular plate casting of 7 cm × (C) 0 < n < 1 (D) n=1
10 cm × 2 cm dimensions having the total solidifica- 42. In full mould (cavity–less) casting process, the pat-
tion time of 1.36 minute. The total solidification time tern is made of: [2015]
(in minute) of the riser is _____ [2014] (A) Expanded polystyrene (B) Wax
36. Match the following products with preferred manu- (C) Epoxy (D) Plaster of Paris
facturing processes: [2015] 43. In a rolling operation using rolls of diameter 500 mm,
Product Process if a 25 mm thick plate cannot be reduced to less than
P Rails 1 Blow molding
20 mm in one pass, the coefficient of friction between
the roll and the plate is _____. [2015]
Q Engine crankshaft 2 Extrusion
44. Ratio of solidification time of a cylindrical casting
R Aluminium channels 3 Forging
(height = radius) to that of a cubic casting of side two
S PET water bottles 4 Rolling
times the height of cylindrical casting is ____.
(A) P–4, Q–3, R–1, S–2 45. The dimensions of a cylindrical side riser (height =
(B) P–4, Q–3, R–2, S–1 diameter) for a 25 cm × 15 cm × 5 cm steel casting are
(C) P–2, Q–4, R–3, S–1 to be determined. For the tabulated shape factor val-
(D) P–3, Q–4, R–2, S–1 ues given below as follows, the diameter of the riser
(in cm) is ______. [2015]
3.812  |  Part III  •  Unit 7  •  Manufacturing Technology

Shape factor 2 4 6 8 10 12 (A) 150 (B) 200


(C) 100 (D) 125
Riser Volume/
1.0 0.70 0.55 0.50 0.40 0.35 49. A 300 mm thick slab is being cold rolled using roll of
Casting volume
600 mm diameter. If the coefficient of friction is 0.08,
46. The part of a gating system which regulates the rate of the maximum possible reduction (in mm) is _______.
pouring of molten metal is: [2016]
(A) pouring basin (B) runner [2016]
(C) choke (D) in gate 50. Gray cast iron blocks of size 100 mm × 50 mm ×
47. Heat is removed from a molten metal of mass 2 kg at 10 mm with a central spherical cavity of diameter 4
a constant rate of 10 kW till it is completely solidified. mm are sand cast. The shrinkage allowance for the
The cooling curve is shown in the figure. pattern is 3%. The ratio of the volume of the patter to
volume of the casting is ______. [2016]
1100
(0s, 1023K) 51. Equal amounts of a liquid metal at the same tem-
1000
perature are poured into three moulds made of steel,
Temperature (K)

900 (20s, 873K) copper and aluminum. The shape of the cavity is a cyl-
800 (10s, 873K)
inder with 15 mm diameter. The sizes of the moulds
700 are such that the outside temperature of the moulds do
600 not increase appreciably beyond the atmospheric tem-
(30s, 600K) perature during solidification. The sequence of solidi-
500
0 10 20 30 40 fication in the mould from the fastest to slowest is
Time (s)
(Thermal conductivities of steel, copper and alu-
Assuming uniform temperature throughout the vol- minum are 60.5, 401 and 237 W/m-K, respectively
ume of the metal during solidification, the latent Specific heats of steel, copper and aluminum are 434,
heat of fusion of the metal (in kJ/kg) is _______. 385 and 903 J/kg-K, respectively. Densities of steel,
 [2016] copper and aluminum are 7854, 8933 and 2700 kg/
48. A cylindrical job with diameter of 200 mm and height m3, respectively.) [2016]
of 100 mm is to be cast using modulus method of riser (A) Copper–Steel–Aluminum
design. Assume that the bottom surface of cylindri- (B) Aluminum–Steel–Copper
cal riser does not contribute as cooling surface. If the (C) Copper–Aluminum–Steel
diameter of the riser is equal to its height, then the (D) Steel–Copper–Aluminum
height of the riser (in mm) is [2016]

Answer Keys
Exercises
Practice Problems 1
1. D 2. B 3. D 4. C 5. D 6. D 7. A 8. C 9. B 10. D
11. C 12. C 13. A 14. D 15. C 16. B 17. A 18. B 19. C 20. A

Practice Problems 2
1. B 2. B 3. C 4. A 5. B 6. C 7. B 8. C 9. B 10. A
11. B 12. D 13. C 14. C 15. B 16. B 17. A 18. C 19. C 20. A
21. D 22. D 23. A 24. B 25. C 26. D 27. B 28. A 29. C 30.  A

Previous Year’s Questions


1. B 2. A 3. B 4. B 5. D 6. B 7. C 8. D 9. B 10. C
11. B 12. D 13. D 14. C 15. D 16. C 17. B 18. None 19. A 20. D
21. A 22. A 23. D 24. A 25. D 26. C 27. B 28. C 29.  7 to 8 30. B
31.  14.6 to 14.8 32. B 33. C 34. B 35.  2.5 to 4.5 36. B 37. B
38.  5.6 to 5.8 39.  6.0 to 6.3 40. B 41. C 42. A 43.  0.10 to 0.15
44.  0.5 to 0.6 45.  10.5 to 10.7 46. C 47. 50 48. A 49.  1.9 – 1.94 mm
50.  1.08 to 1.10 51. C
Chapter 3
Sheet Metal Working
Joining Processes
chapter highlights

☞☞ Sheet Metal Working ☞☞ Shielded Metal Arc Welding (SMAW)


☞☞ Drawing ☞☞ Electric Resistance Welding
☞☞ Shearing Action ☞☞ Welding Equations
☞☞ Provision of Shear ☞☞ Heat Input and Heat Flow in Welding
☞☞ Energy in Presswork ☞☞ Gas Welding
☞☞ Powder Metallurgy ☞☞ Oxygen and Acetylene Cylinders
☞☞ Solid State Welding ☞☞ Oxy–fuel Gas Cutting (OFC)
☞☞ Electric Arc Welding ☞☞ Brazing, Braze Welding and Soldering
☞☞ AC Welding Equipment

Sheet Metal Working


Punch
Sheet metal working or press working of sheet metals is a
chipless manufacturing method producing various compo- Blank
nents using sheet metal. The operations are generally car- holder
ried out by punches and dies. These can be grouped into
Blank
two categories. before
drawing
1. Cutting or shearing operations
2. Forming operations
Die
Cutting/shearing operations involve shearing action and Drawn cup
include blanking, piercing, punching, notching, perforating,
trimming, shaving, slitting, lancing, slotting, parting etc.
Forming operations include bending, drawing, squeez- In a single drawing operation the diameter of the blank can
ing, spinning etc. be reduced 1.8 to 2 times less than that of the initial blank.
If a further reduction is required the blank is to be redrawn.
Drawing
D
Drawing is the process of shaping a flat or hollow blank into ∴ = 1.8 to 2
a three dimensional hollow component without any appreci- d
able change in sheet thickness. Considerable compressive
D-d
stresses appear in the flange portion of the blank drawn % Reduction = ×100
and this causes wrinkling if the blank thickness is small. D
To prevent wrinkling, a blank holder is used. The portion of Where D = blank diameter
blank between the die wall and the punch surfaces is under d = cup diameter
tension.
3.814  |  Part III  •  Unit 7  •  Manufacturing Technology

Blank Size Re-drawing


Blank diameter, D can be estimated using the formulae When the blank diameter and the final cut diameter is
given below. very large, the drawing operation requires more than one
Let h = height of cup stage. The drawings after the first stage is called re-drawings.
r = corner radius of cup
d = outside diameter of cup Reverse Drawing
When a cup is drawn reverse to the original direction of
d drawing it is called reverse drawing. Reverse drawing helps
If ≥ 20,
r strain softening.
D = d 2 + 4 dh Defects in Deep Drawing
d Bulging, buckling, earing and surface scratch are some
If 15 ≤ ≤ 20 defects noticed in deep drawing.
r
Bulging happens when the outer diameter of a cylindri-
D = d 2 + 4 dh − 0.5r cal shell or outer walls of a box shaped shell expands from
the straight shape.
If 10 ≤ ≤ 15, D = d 2 + 4 dh − r Excessive compressive stress causes uncontrolled defor-
mation pattern perpendicular to the surface of the sheet. As
d a result, a bend, kink or wavy condition is formed on the
If ≤ 10 surface This is called buckling.
r
Edges formed around the top of a drawn sheet is called
D= (d − 2r )2 + 4d ( h − r ) + 2π r (d − 0.7r ) earing.
Bending: It is the operation in which a flat sheet or strip
If the depth to diameter ratio (i.e, h/d ) is greater than 0.4 is uniformly strained in a linear axis, which lies in the neu-
drawing is called deep drawing otherwise it is shallow tral plane and perpendicular to the length wise direction.
drawing. In bending a straight length is transformed into a curved
length. The strain in the bent material increases when
the radius of curvature decreases. The neutral axis of the
Solved Examples
­section moves towards the inner surface. Distance of neu-
Example 1:  A cup of 10 cm height and 5 cm diameter with tral axis from inside surface is 0.3 t to 0.5 t depending
2.5 mm corner radius is to be made from a metal sheet of 3 upon the radius of curvature, where t is the thickness of
mm thickness by drawing. Determine the number of draws the part.
required.
Set back
Solution: h = 10 cm = 100 mm Set back Neutral plane
d = 5 cm = 50 mm
t = 3 mm t
r = 2.5 mm t1
d 50 R
= = 20
r 2.5 θ
d
If ≥ 20
r θ Bevel angle
Blank radius D = d 2 + 4 dh

= 50 2 + 4 × 50 × 100
= 150 mm t1
Stretch factor or k factor k =
D 150 t
= =3
d 50  = 0.33 when R < 2t
For a single draw blank diameter should not be greater than  = 0.5 when R > 2t
twice the drawn diameter. Where R = inside bend radius
∴ Number of draws required is 2. t = material thickness
t1 = distance of neutral plane from inside surface
Ironing
θπ
Ironing is the deep drawing operation in which the shell Bend allowance BA = ( R + kt )
wall thickness is reduced and surface is made smooth. 180°
Chapter 3  •  Sheet Metal Working Joining Processes  |  3.815

Where q = bend angle in degrees within a set of dies. Used for production of coins, medals
(Bend allowance is the distance along the neutral plane etc. The pressure required is 5 to 6 times the strength of the
corresponding to bend angle). material, in order to produce fine details.
Example 2:  A 3 mm thick metal sheet is to be bent at an Blanking: It is the operation of cutting a flat piece of
angle of 1.5 radian with a radius of 150 mm. If the k-factor required shape from a sheet using a punch and a die. The
is 0.5, the bend allowance is metal punched out is the required product and it is called
a blank.
Solution: t = 3 mm Piercing/punching: It is the production of a hole in a
q = 1.5 radian sheet metal by the punch and die. The operation is same as
R = 150 mm blanking. But here, the material punched out constitute the
k = 0.5 waste and the sheet with hole is the required product. In
Bend allowance punching the hole produced is circular. In piercing it can be
= (R + kt)q of any shape.
= (150 + 0.5 × 3) x 1.5
= 227.25 mm Shearing Action
Notching: It is the operation in which metal pieces are When the sheet metal is placed between two shearing blades
cut from the edge of a sheet, strip or blank. and pressed the metal is brought to plastic stage and frac-
Perforating: It means cutting very small holes which are ture is initiated at the cutting edges. The fracture on either
very close, in a sheet metal. side of the sheet further progress with the movement of the
Trimming: It is the cutting and removal of unwanted upper shear and finally separates the slug from the parent
excess material from the periphery of a previously by sheet.
formed component. Metal under the upper shear is subjected to both tensile
Shaving: It is Removal of thin strip of metal from a and compressive stresses. The upper shear pushes the metal
blanked part to make it dimensionally accurate and smooth about one third of the thickness in an ideal shearing opera-
is called shaving. tion. The area of cross section between the cutting edges
Slitting: It is the operation of cutting a sheet metal along gets reduced and facture is initiated. If the clearance is suf-
the length. ficient, further movement of the upper shear will cause the
fractures to meet and complete the shearing action.
Slotting
Cutting of elongated holes or slots is called slotting. Clearance
Clearance between the shears is an important factor which
Parting controls the shearing process. The clearance can be approxi-
Shearing of a sheet metal into two or more pieces is called mated as.
parting. C = 0.0032 × t × τ mm
Lancing: It is a cutting operation in which a hole is par- Where t = sheet thickness in mm
tially cut and the cut portions is bent. t = material shear stress in MPa
Nibbling: It is removing metal in small increments.
When a specified contour is to be cut in a sheet metal a Punching Force
small punch is used to punch repeatedly by along the The force required on a punch in order to shear out the blank
contour. from the stock is given by the formula.
Squeezing: In this operation, metal is caused to flow to F = Ltt
all portions of die cavity under the action of compressive Where L = length (perimeter)
forces. t = thickness
Spinning: It is the operation of shaping of thin metal by t = shear strength
pressing it against a form, while it is rotating. Spinning is Punching force for holes with diameter less than the thick-
done on lathe like machines. ness may be estimated as,
Stretch forming: is the process of producing contoured
parts by stretching metal sheets over a shaped form block. dts
F=
Embossing: It is the operation of producing raised or 3
d
depressed impression of figures, letter or design on sheet t
metal parts.
Coining: It refers to the cold squeezing of metal under Where d = diameter of punch, mm
compressive forces, while all the surfaces are confined s = tensile strength of stock, MPa
3.816  |  Part III  •  Unit 7  •  Manufacturing Technology

Provision of Shear Energy in Presswork


Shear is provided on a punch or die to reduce the required Energy or work done to make a cut is given as
shearing force. For example, this may be required to accom- E = Fmax × punch travel
modate a component on a smaller capacity punch press.
Shear is ground on the face of die or punch. This distributes = Fmax × k × t
cutting force over a period of time depending on the amount Where k = percentage of penetration required to cause
of shear provided. This reduces the maximum force to be rupture.
applied, but does not alter the total work done. t = thickness of sheet
Provision of shear on the punch will change the slug and Fmax =Ltt
shear provided on the die will change the stock. So shear is
provided on the die for blanking and on the punch for pierc- To account for energy lost in machine friction the energy
ing. Therefore for blanking operations, equation may be modified as
blank size = die size T = Fmax × k × t × C
And for piercing/punching operation, where C = factor to account for machine friction.
blank size = punch size When shear is provided, let F be the punching force
Force required
Stock Punch with Then E = F(kt + S)
concave shear Where S = shear
F(kt + S) = Fmax kt
Fmax kt
Die or F =
kt + S
Slug
Example 4:  12 mm diameter holes are to be punched in a
steel plate of 3 mm thickness and shear strength 400 MPa.
Piercing operation Shear provided on the punch is 2 mm and penetration is 40%.
Determine the blanking force required for the operation.
Punch
Punch Solution:  d = 12 mm
t = 3 mm
S S
tu = 400 MPa = 400 N/mm2
Unbalanced Balanced Penetration = 40% = 0.4
shear shear
Shear S = 2 mm
Example 3:  Calculate the die and punch sizes for blanking Fmax = pdt tu
a circular disc of 22 mm diameter from a C 20 steel sheet  = p × 12 × 3 × 400
with thickness 1.5 mm. Determine also the punching force
 = 45,239 N
(Shear strength for C 20 steel = 294 MPa).
Fmax kt = F(kt + S )
Solution:  Clearance
C = 0.0032 × t × τ Fmax kt
or F =
= 0.0032 × 1.5 × 294 kt + S
= 0.0823 mm ~ 0.10 mm 45239(0.4 × 3)
For blanking operation, =
(0.4 × 3) + 2
Die size = blank size
= 22 mm = 16965 N.
Punch size = blank size – 2C
= 22 – 0.2 = 21.8 mm Powder Metallurgy
For piercing operations, Powder metallurgy is a process of making reliable ferrous
Punch size = hole size = 22 mm and non-ferrous components from ferrous and non-ferrous
Die size = hole size + 2C powders. Originally it was used to replace castings of met-
= 22 + 0.2 = 22.2 mm als of very high melting point. The development of the tech-
Punching force =  Ltt nique made it possible to produce the product economically
= p × 22 × 1.5 × 294 × 10-3 and now a days it occupies and important place in the metal
= 30.4 kN. processing field.
Chapter 3  •  Sheet Metal Working Joining Processes  |  3.817

Basic Steps of the Process added to develop the required green strength. Lubricants are
Basic steps of the manufacturing of parts by powder added to reduce inter particle friction and to reduce die wall
metallurgy are friction.

1. Production of metal powders Compaction


2. Blending and mixing of powders
Compaction is the process of pressing the blended powder
3. Compaction
to shape in dies. A green compact of accurate size and shape
4. Sintering
is obtained in this process. Pressing, centrifugal compact-
5. Finishing operations
ing, slip casting, extrusion, gravity sintering, rolling, explo-
Production of Metal Powders sive moulding etc are some of the compacting methods.
Depending on the type and nature of the metal, various
methods are available for production of metal powders. Sintering
Some of the methods are Sintering is the process of heating the green compact at high
temperatures below melting point in a controlled atmos-
1. Atomization phere. Sintering increases the bond between particles and
2. Machining increases strength of the powder metal compact. Sintering
3. Crushing and milling temperature is usually 0.6 to 0.8 times the melting point of
4. Reduction the powder. If powders of different melting points are used,
5. Electrolytic deposition the sintering temperature is above the melting point of one
6. Shotting of the low melting point consistent.
7. Condensation
In atomization, molten metal is forced through a small Finishing Operations
orifice and is broken into the form of small particles by a Finishing operations are secondary operations intended for
powerful jet of compressed air, inert gas or water. These providing dimensional tolerances, better surface finish etc.
particles are allowed to solidify to get the powder. This Finishing operations are
method is used for low melting point alloys. 1. Coining: It is the repressing of the sintered compo-
In machining, metal chips produced by filing, turning nent in the die to increase density and to provide the
etc are pulverized by crushing and milling. The powders required surface details.
obtained are coarse and irregular in shape. This method is 2. Forging or hot densification: It is the method in which
mainly used for production of magnesium powder. desired shape is given to the sintered part at elevated
Crushing and milling methods are used for brittle mate- temperature and pressure in dies.
rials. Ceramic powder is produced by this method. It is also 3. Machining: Machining is the operation carried out on
used for powders of some metals and alloys. the sintered part for making threads, holes slots etc,
In the method of reduction pure metal is produced by the which cannot be provided during the powder metal-
reduction of metallic oxides using suitable reducing gas at lurgy process.
high temperature before melting point. Later this reduced 4. Sizing: It is the process in which the sintered compo-
product is crushed and milled to powder. nent is repressed to achieve the required accuracy.
Electrolytic deposition method is mainly used for pro- 5. Heat treatment: It is the heating and cooling treat-
duction of iron and copper powders. Metallic powder ment done on the part to achieve wear resistance, grain
deposited on cathode plates are scrapped off, washed, structure and strength.
dried and pulverized to produce powder of the required 6. Plating: Plating is done on the parts to attain surface
grain size. finish and colour, protection from corrosion and to
Shotting is used for metals of low melting points. Molten improve electrical conductivity etc.
metal is poured through a sieve or orifice and cooled by 7. Infiltration: In this process pores of the part are filled
dropping into water. This produces larger size spherical par- with molten metal to improve physical properties.
ticles. This is used for low melting point metals. 8. Stream treating: In this process sintered part is given
In the method of condensation metals are boiled to pro- oxide coating.
duce vapour and condensed to obtain powder. Powders of 9. Joining: Two sintered parts are joined by welding,
volatile metals like zinc, magnesium and cadmium are pro- brazing etc.
duced by this method. 10. Impregnation: It is the process in which the sintered
part is filled with lubricants such as oil, greeze etc for
Blending and Mixing using as bearings.
In the blending and mixing process metallic powders in 11. Repressing: By repressing, density of the sintered part
the required proportion are mixed uniformly. Binders are is increased. It also improves the mechanical properties.
3.818  |  Part III  •  Unit 7  •  Manufacturing Technology

Joining Processes Fusion or Non-pressure welding


Joining two or more elements to make a single part is
termed as a joining or fabrication process. Joining process Heat generated by
include mechanical joining by means of bolts, screws or riv-
ets; adhesive bonding by employing synthetic glues such as
epoxy resins, welding, brazing and soldering .
Choice of a particular process depends on type of assem- Radiant Electric
Chemical
bly (permanent, semi-permanent or temporary); materials, reaction
energy arc
economy, type of service (heavy loading, light loading, high
temperature etc.) Electron
Atomic Gas Thermit
beam
Joining by adhesive bonding is done using adhesives hydrogen welding welding
welding
welding
such as thermosetting resins, thermoplastic resins, silicone
resins and elastomers. Elastomers such as natural rubbers Laser
when mixed with thermosetting resins reduce brittleness Beam W
and provide toughness for shock resistance.
Carbon Shielded Inert gas Submerged Stud arc
Welding laser metal welding arc welding
Welding is the most extensively used joining method. In welding arc welding
welding
welding, the joining takes place through atomic bonding.
Atomic bonding may be solid state, liquid state or solid liq-
uid state.
For the bonding to take place heat or pressure or both TIG MIG
welding welding
heat and pressure are to be applied at the joints.
In fusion welding heat is applied at the joints and in pres-
sure type welding pressure also applied apart from heat. Solid State Welding
In all welding process, except cold welding process, heat Cold welding, friction welding, diffusion welding, forge
is applied for the bonding to take place. Therefore there welding etc are examples of solid state welding. Cold weld-
should be one heat source. ing includes pressure welding, ultrasonic welding, and
Fusion welding is a non pressure liquid state welding explosive welding.
Bonding takes place in a metallurgical fusion process where In fusion welding process melting of metal takes place.
interface of two parts are brought above melting point and Therefore there should be a source of heat. In electric arc
then allowed to solidify. welding, electric resistance welding and induction welding
Electric arc welding, Induction welding, gas welding, heat is generated by thermo chemical actions.
thermit welding etc are examples of fusion welding. In gas welding and thermit welding heat is generated by
Forge welding, resistance welding, friction welding thermo chemical actions.
and cold pressure welding are examples of pressure type In new generation welding processes such as electron
welding. beam welding, laser beam welding etc radiant energy is the
Classifications of welding processes are shown in the source of heat for welding.
figures.
Electric Arc Welding
Pressure welding
Electric arc welding is a fusion welding process. Welding
heat is obtained from an electric arc between the work (or
Heat generated by base metal) and electrode.
Cold pressure
Electric arc is produced when two conductors in an elec-
welding tric circuit, which are touched together are separated by a
small distance, such that there is sufficient voltage in the
Fire Electric current Friction
circuit to maintain the flow of current through the gaseous
medium. Temperature produced is 6000°C to 7000°C.
Blacksmiths Resistance Friction The depression created on the base metal due to the arc
forge welding welding
is called crater.
welding
DC or AC current can be used for arc welding. DC volt-
age required is 60 to 80 V for striking the arc and 15 to 25 V
Spot Seam Projection Upset Flash butt Percussion
for maintaining the arc. For AC current they are 80 to 100 V
welding welding welding welding welding welding and 30 to 40 V respectively.
Chapter 3  •  Sheet Metal Working Joining Processes  |  3.819

An arc is a sustained electric discharge through the ion- Bare electrodes may be used for welding mild steel and
ized gas column, called plasma between the electrodes. wrought iron.
When electrons hit the anode at high velocity, kinetic energy Primary function of a light coating is to increase the arc
is converted producing a large amount of heat. Similarly stability. These are also called ionizing coatings.
positively charged ions hitting the cathode also produces Coated electrodes contain arc stabilizing ingredients,
heat. 65 to 75% of the total heat generated is at the anode. slag forming ingredients, binding materials, alloying con-
So work is connected as anode if it is required to gener- stituents etc. Some times iron powder is added to improve
ate more heat at anode. This is called straight polarity or deposition rate.
DCEN (direct current electrode negative) DCEN is required
for thicker plate and materials of higher thermal conduc- Types of Electric Arc Welding Process
tivity. For thinner plates reverse polarity or DCEP (Direct
1. Carbon arc welding
current electrode positive) is used. Weld penetration is more
2. Shielded metal arc welding
in DCEN. Weld penetration for AC is between DCEP and
3. Flux cored arc welding
DCEN.
4. Gas metal arc welding
Ac Welding Equipment 5. Gas tungsten arc welding
6. Submerged arc welding
1. AC Machines 7. Atomic hydrogen welding
(i) Transformer 8. Plasma arc welding
(ii) Alternater engine driven by motor or engines 9. Stud welding
10. Electro slag welding
2. DC Machines
(i)  Transformer with rectifier Carbon Arc Welding
(ii)  DC generator driven by motor or engine
In carbon arc welding carbon or graphite electrodes are
Transformer sets are more commonly used in AC weld-
used. If required filler material also is used. Shielding is not
ing. As there is no moving parts power consumed and
generally used. So carbon arc welding is used in metals that
noise are less. Also maintenance cost is low and efficiency
are not sufficiently contaminated by oxygen and nitrogen
is more.
in the atmosphere (copper alloys, brass, bronze, Aluminium
Specification of Arc Welding Machines alloys etc.).

1. Maximum rated open circuit voltage Shielded Metal Arc Welding (SMAW)
2. Rated current in amperes This method is also known as manual metal arc weld-
3. Duty cycle ing. This is the most generally used welding type. Coated
electrodes are used in this type of welding. Shielding is
American welding society (AWS) defines duty cycle as obtained from the decomposition of the coating. The ingre-
the percentage time in a ten minute period that a welding dients in the vaporized coating creates a protective gas
machine can be used at its rated output without overload- atmosphere over the weld puddle. As the coating melts at
ing. Normally 40% duty cycle is suggested (Indian standard a slower rate than the metal this welding rod will be having
specifies 5 minutes as the cycle time.) a concave end. This helps to concentrates the heat from the
Types of Welding Electrodes arc. The flux coating helps removal of impurities through
formation of slag. The electrode diameter depend upon
1. Non consumable the thickness of the metal being welded and the type of
2. Consumable the joint.
Welding current is determined on the basis of the elec-
Non consumable type of electrodes are made of trode diameter.
­carbon, graphite or tungsten. Carbon and graphite elec- Welding current = k.d amperes
trodes are used in DC welding only tungsten can be Where d is diameter in mm.
used for both AC and DC welding. As this electrode K = constant
is not consumed arc length is constant and it is sta- = 45 to 60 for ordinary steel electrodes
ble and easy to maintain. Separate filler rods are used in = 18 to 22 for graphite
this case. = 5 to 8 for carbon
There are three types of consumable electrodes Voltage depends only on the arc length. It is given by,
1. Bare electrodes V = k1 + k2 volts
2. Fluxed or lightly coated electrodes Where k1 = 10 to 12 and k2 = 2 to 3
3. Coated or extruded/shielded electrodes L = arc length in mm.
3.820  |  Part III  •  Unit 7  •  Manufacturing Technology

Minimum arc voltage, Vmin = (20 + 0.04I) volts. Atomic Hydrogen Welding
Arc length depends upon the kind of electrodes used, its In atomic hydrogen welding arc is produced between two
coating, its diameter, current used and position of ­welding. tungsten electrodes. A stream of hydrogen passes to the
Shorter arc lengths are used for overhead and vertical weld area through nozzles through which the electrodes are
positions. held. High temperature of the arc breaks up the hydrogen
An arc length of 0.6 to 0.8 times the electrode diameter molecules into hydrogen atoms absorbing heat from the
can produce stable arcs and high quality welding. arc (421.2 KJ/mol). The hydrogen atoms are highly reac-
tive. They combine with atmospheric oxygen to form water
Flux Cored Arc Welding vapor and form hydrogen molecule at the surface to be
In this method a hollow tubular electrode inside which the welded releasing intense heat necessary for melting of the
flux is provided, is used. Continuous welding is possible as metal. Because of its reactivity hydrogen atoms also breaks
the electrode can be supplied in coils. the oxides on the base metals allowing formation of a clean
weld. Hydrogen also acts as a shielding gas. As the molten
Gas Metal Arc Welding (GMAW) metals becomes highly fluid atomic hydrogen welding is
This is also known as metal inert gas (MIG) welding. In used only for flat positions. The main advantage of this pro-
this method an inert gas such as Argon is used for shield- cess is its ability to provide high heat concentration. Thin
ing the welding area. Consumable electrode is fed through metal sheets or smaller diameter wires can be welded using
a welding gun through which the inert gas also is supplied this method because of its lower thermal efficiency com-
for shielding weld area. Electrode is supplied in coils and pared to direct arc processes. Aluminium, stainless steel
continuous welding is possible. Other inert gases are helium sheets etc are welded using this method.
and carbon dioxide.
Plasma Arc Welding
Gas Tungsten Arc Welding (GTAW) In plasma arc welding, welding is done using a plasma
This process is also known as tungsten inert gas weld- jet. Plasma is a gas sufficiently ionized, containing posi-
ing (TIG welding). This is similar to mig welding. But a tive and negative ions and with very high temperature and
non-consumable electrode of tungsten is used. For shield- conducts current freely. In plasma welding argon gas and
ing, inert gas such as argon is used. A filler metal may or tungsten electrode is used. Argon gas is used for producing
may not be used. In the tungsten electrode 1 to 2% tho- the plasma jet as well as acting as a shielding gas. Plasma
rium and zirconium are added improve electron emission, jet is created when the arc is passed through a constrictive
arc stability, arc striking and current carrying capacity etc. nozzle. As a result of this, the plasma jet will take a nar-
TIG welding was originally developed for welding magne- row columnar shape with unique properties ideal for weld-
sium which is highly oxidizing. Now it is used for weld- ing. The plasma welding torch has passages for orifice gas,
ing, aluminium and its alloys, stainless steel, cast iron, shield gas and water for cooling.
silicon bronze, titanium, nickel, copper and carbon steels. There are two methods of plasma welding
This method is suitable for welding thinner metals, below
1. Transferred plasma arc
6 mm thick. Both AC and DC can be used. DCEP is not
2. Non-transferred plasma arc
used as this tend to melt electrode due to overheating. For
more penetration DCEN is preferable. For metals like mag- In both cases electrode is negative. In transferred arc work
nesium and aluminium high frequency AC supply is used is positive and in non-transferred arc the nozzle is con-
which break up the surface oxides. nected as positive.

Submerged Arc Welding (SAW) Stud Arc Welding


Submerged arc welding is generally used for welding thick Stud arc welding is an arc welding process used for welding
plates which require straight welds in flat position. In this studs to flat metal surfaces. A stud welding gun is used for
method arc electrode is continuously fed from reals. The this, purpose. An arc is produced between the work and end
arc and the welding occurs inside a blanket of granu- of the stud held in the gun. When the stud end and work spot
lar flux which is continuously fed ahead of the electrode. melts the stud is pressed and allowed to cool. The whole
The granular flux shields the weld area from atmosphere. cycle of operations are automatically controlled.
Molten flux acts as a cleanser, absorbing impurities from
the molten metal and producing the slag which floats on Electro Slag Welding
top of molten metal. The flux may also contain powder Electro slag welding is used for welding thick plates, struc-
metal alloying elements, In order to prevent molten metal tures for turbine shafts, boiler parts etc. In this process the
running out of the joints, water cooler backup plates plates to be welded are held in a vertical position with a gap
are used. of 15 to 30 mm. Filler wires and flux are fed automatically
Chapter 3  •  Sheet Metal Working Joining Processes  |  3.821

into the gap. Filler wires are used as electrodes. Initially an metals can be welded using this method. For example alu-
arc is produced melting the flux into slag. The arc is stopped, minium or copper to stainless steel.
and the slag is maintained in the molten state by the heat
produced by the resistance. The molten metal in the gap is Electric Resistance Welding
held by water cooled copper shoes (dams). As the cooling In resistance welding heat required for welding is produced
rate of molten metal is low course grains are formed and by means of electrical resistance at the joint of two parts
a further heat treatment is required to restore the strength. to be joined. Low voltage (4 to 12 V) and high current is
applied. Heat generated during time, t is given by
Electro Gas Welding H = I 2 R t k
When I = current in amperes
This process is a development of electro-slag welding. The
R = resistance in ohms
main difference with the electro-slag welding is that no flux
t = time in secs
is fed into the joint and the heat is produced by electric arc
k = constant to account for loss by conduction and
through out. An inert gas is fed into the joint for shielding
radiation
the arc. This process is used for welding 20 to 80 mm thick-
ness plates. Types of resistance welding
1. Spot welding
Non-conventional Welding
2. Seam welding
Cold welding, diffusion welding explosion welding, resist- 3. Projection welding
ance welding etc can be grouped under non-conventional 4. Butt welding
welding. 5. Percussion welding
Cold Welding In spot welding the tips of two solid cylindrical electrodes
Cold welding is a solid state welding done at room tempera- are placed on either side of the lap joint of two sheet metals
ture under pressure. Coalescence of the metal parts occur and high current is passed across the point of contact. Heat
due to the deformation under the great pressure applied by generated melts the metals locally at the point of contact
roller or die. when pressure is applied. Low voltage and high current is
applied during a very short time.
Diffusion Welding Seam welding is a specialized case of spot welding. In
In diffusion welding process, strength of the joint is obtained this case rotating disc type electrodes are used. Therefore a
primarily from diffusion of the atoms across the interface. continuous weld is obtained. The seam is made of continu-
It is a solid state welding process where coalescence of the ous overlapping spot welds. If the spot welds are spaced it is
parts occur by the application of pressure under elevated known as roll spot welding.
temperatures. Generally the temperature is above 0.5 times Projection welding is another variation of spot welding.
the melting point. One sheet is provided with a number of projections to help
to locate the current at predetermined spots. These projec-
Explosion Welding tions are obtained by embossing.
Explosion welding or explosive welding is a solid state Upset butt welding is a butt joint welding. Heat is gen-
welding where the parts are joined by high velocity move- erated at the contact area between the two plates. The joint
ments produced by a controlled detonation. get slightly upset due to the pressure applied. It is used for
joining ends of rods or similar pieces.
Explosive Flash butt welding is another butt welding process. But
Detonater Buffer plate here heat is generated by an arc. Two pieces to be welded
Flyer plate
arc brought together and power supply is switched on.
Target plate Momentarily the pieces are separated a little to produce
an arc due to which the ends get melted. They are pressed
Anvil
together and power is switched off.
Percussion welding is a recent development. The weld-
ing heat is obtained from the arc produced by a rapid dis-
charge of stored electric energy. One piece is held in a fixed
clamp and other in a spring loaded movable clamp. When
When the explosive is detonated, the flyer plate moves to movable clamp is released discharge occurs when the gap
the target plate under great velocity and bonding occurs is about 1.5 mm. The arc is extinguished by the percussion
with the target plate or base plate. This method is used for blow of the moving part coming with sufficient force and
metal cladding of heat exchanger tube plates etc. Dissimilar the weld is effected.
3.822  |  Part III  •  Unit 7  •  Manufacturing Technology

Welding Defects Lamellar Tears


For a strong welding joint the welding should have mini- Lamellar tears are caused by non-metallic inclusion such
mum defects. as sulphides and oxides. These inclusions are elongated by
The following defects are noticed in welding rolling process. This generally happens in plates of low duc-
tility in the thickness direction.
1. Porosity
Lamellar tears are more dominant in T and corner joints
2. Slag inclusions
where fusion boundary is parallel to the rolling plane.
3. Incomplete penetration
4. Underfilling Welding Equations
5. Undercutting
6. Inclusions 1. Voltage–arc length characteristic of DC V = A + BL
7. Cracks Where V = voltage drop across the arc
8. Lamellar tears A = electrode drop
BL = column drop
Porosity is caused by the gases such as oxygen, nitrogen
2. Power source characteristic equation
and hydrogen absorbed during melting.
As the molten metal is cooled solubility of the gases is OCV . I
V = OCV −
decreased and they try to escape. If escaping is not possible ISC
they remain with in the weld causing porosity. Where OCV = open circuit voltage
In inclusions slag, scale or dirt may get entrapped in the ISC = short circuit current
weld deposit during welding. Contaminated base plate, non- I = arc current
uniform melting of electrode coating, high melting point V = arc voltage
or high viscosity of slag or insufficient deoxidizing of the
3. Power, P = V.I
metal in the weld etc can cause inclusions. Slag inclusion
δP
can occur in a multipass welding if the slag solidified in the For maximum power, =0
previous pass is not cleaned before the next pass. δΙ
Incomplete fusion or lack of fusion may occur if the tem- Heat Flow Characteristics
perature of the base metal is not raised to its melting point In welding processes heat source may be moving or sta-
and due to faulty welding conditions. When the gap is not tionary. For example in fusion welding processes like arc
totally filled by molten metal, it leads to incomplete fusion. welding heat source is moving and in spot welding the heat
Underfilling occurs when proper amount of molten source is stationary. For analysing purpose moving heat
metal is fed to the gap. source is treated as stationary and work piece is treated as
In Undercutting An undercut is a groove formed adja- moving with the same velocity in opposite direction. This
cent to the toe of the weld. Under cutting may be caused by speed is called welding speed. Heat source can be a point
excessive heating, improper positions of electrode or torch source of three dimensional heat flow or a line source as
tip or non-uniform feed of filler rod. Undercutting reduces in the case of butt welding of thin plates. For three dimen-
fatigue strength of the joint sional heat source,
Undercut Rate of heat input
5  2 vw 
Q= π wkθ m  + 
4  5 4α 
Where w = width of the weld
k = thermal conductivity of the work piece
qm = 
difference between melting point and ­ initial
Cracks temperature of the work material
Cracks may occur in or near the weld. Cracks can be v = welding speed
micro cracks, macro cracks or wide cracks (fissures) k
a = thermal diffusivity =
depending upon the size. Micro cracks will be visible r = density ρ c
only through a microscope. Macro cracks can be seen by c = specific heat
unaided eye or through a low power magnifier. Cracks that For two dimensional heat source the equation is
occur at low temperature (around 200 °C) is known as
cold cracks.  1 vw 
Q = 8kθ m h  +
Cracks that occur when the metal is very hot is known as  5 4α 
hot cracks. Depending upon location and direction, cracks Where h = thickness of plate
can be toe crack, under bead crack, longitudinal crack, In arc welding heat input rate is given by Q = C VI
transverse crack etc.
Chapter 3  •  Sheet Metal Working Joining Processes  |  3.823

where V = arc voltage SAW 0. 90 to 0.99


I = arc current For resistance welding, H = I 2 Rt.
C = fraction of total time during which the arc acts
Heat Flow
Example 5:  The voltage – arc length characteristic of DC
arc is given by V = 20 + 40L where L = arc length in cm. The For relatively thick plates, cooling rate is given by,
power source characteristic can be approximately a straight 2ρk (Tc − To ) 2
line. Open circuit voltage is 80. and short circuit current is R=
H net
1000 A k. Determine optimum arc length.
Solution:  For welding arc, Where T0 = initial plate temperature °C
V = 20 + 40L(1) k = thermal conductivity of base metal J/mm s °C
For power source Tc = 
temperature at which cooling rate is calculated
80 (2) ~ 550°C for most steels.
V = 80 − I
1000 R = cooling rate at the weld centre line.
For stable arc, If the plates are relatively thin requiring less than
(1) = (2) 3 passes, the following equation can be used.
80  
20 + 40 L = 80 − I R = π k ρc  (Tc − To )
1000  
net 
80
I = 60 − 40 L
1000 Where h = thickness of base metal, mm
I = 750 – 500 L ρ = density of base metal gm/mm3
Power, VI = (20 + 40L) (750–500L) C = sp. heat J/g °C
1000 (15 + 20L – 20L2) Example 6:  Calculate the melting efficiency, in the case of
arc welding of steel with a potential of 20 V. and a current
For optimum arc length,
δp
=0 of 220 A. Travel speed is 5 mm/s and cross sectional area of
δI the joint is 24 mm2. Heat required to melt steel may be taken
i.e. 20 – 40L = 0 as 12 J/mm3 and the heat transfer efficiency 0.85.
i.e. L = 0.5 cm Solution:  Net heat supplied = 0.85 × 20 × 220 = 3740 W
Volume of base metal melted = 24 × 5 = 120 mm3/s
Heat Input and Heat Flow in Welding Heat required for melting = 120 × 12
Heat Input  = 1440 J/s
For arc welding, P = VI  Watts or J/s. Melting efficiency
Let v = the travel speed of electrode in mm/s 1440
Heat input is given by =
3740
P
H= J/mm = 0.385
v = 38.5%.
VI
= J/mm Example 7:  Two steel sheets of 1.2 mm thickness are
v resistance welded in a Lap joint with a current of 10000
VI A for 0.1 second. The effective resistance of the joint can
H net = f1 J/mm be taken as 100 micro ohms. The joint can be considered
v
as a cylinder of 6 mm diameter and 1.5 mm height. The
Where f1 = a factor for heat transfer efficiency. density of steel is 0.00786 g/mm3. Calculate heat lost to
All the Hnet is not available for melting as part of it is surroundings.
conducted away by the base metal. Considering this another
efficiency factor f2 is used. Solution:  Heat supplied
= (10000)2 × 100 × 10-6 × 0.1
Heat required to melt the joint = 1000 J.
f2 =
Net heat supplied π × 6 2 × 1.5
Volume of the joint =
Values of f1 for various processes are, 4
GTAW 0.21 to 0.48 = 42.412 mm3
SMAW and GMAW 0.66 to 0.85 Heat required for melting = 412 × 10
= 424 J
3.824  |  Part III  •  Unit 7  •  Manufacturing Technology

Heat lost to surroundings = 1000 – 424 Three types of flames can be set is oxy-acetylene welding
= 576J = 57.6%. 1. Neutral
Example 8:  If the two sheets in the above problem is 2. Carburizing
resistance welded in a projection welding with a current 3. Oxidizing
of 30,000 A for 0.005 seconds. The effective resistance of
Neutral flame is obtained when equal amounts of oxygen
the joint can be taken as 100 micro ohms. The joint can
and acetylene are mixed. The flame is recognized by the
be considered as a cylinder of 5 mm diameter and 1.5 mm
inner white luminous cone and outer blue flame envelope.
height. Calculate heat lost to surroundings.
Carburizing flame or reducing flame is obtained
Solution:  Heat supplied when an excess of acetylene is supplied than theoretically
= 30,0002 required. Oxygen to acetylene ratio may be 0.85 to 0.95.
× 100 × 10-6 × 0.006 There are three zones in carburizing flame. In between the
= 540 J inner cone (not sharply defined) and outer bluish envelope
a white zone appears. This is called intermediate flame
π × 62
Volume of the joint = × 1.5 = 42.412 mm3 feather or acetylene feather. Its length is an indication of the
4 amount of excess acetylene.
Heat required for melting = 42.412 × 10 = 424 J For obtaining a neutral flame, first carburizing flame is
Heat lost to surroundings = 540 – 424 = 116 J obtained. Then supply of oxygen is increased till the inter-
mediate feather disappears.
= 21.48%.
Intermediate feather can be adjusted by the amount of
acetylene imbalance induced at the torch. If inner cone
Gas Welding
length is x and intermediate feather length is 3x, it is called
It is a fusion welding process in which heat is generated by a 3x flame.
combustion of a fuel gas. Oxy-acetylene gas welding is the Oxidizing flame has excess oxygen than required for a
most commonly used. Oxygen and acetylene are the gases neutral flame. Oxygen acetylene ratio may be 1.15 to 1.50.
used in this case. To get an oxidizing flame acetylene supply is reduced in
Various gas welding methods with their flame tempera- a neutral flame. Inner cone is slightly shorter and more
ture are as follows. pointed in this case. An oxidizing flame has a harsh sound
1. Oxy-acetylene, 3200°C and outer envelop is short and narrow.
2. Oxy-hydrogen, 2400°C
Flame Temperatures
3. Oxy-propane, 2200°C
4. Oxy-town gas, 2100°C In a neutral flame, inner cone temperature is around 3100°C
5. Air-acetylene, 2400°C and outer blue envelope is around 1275°C. In a reducing
6. Air-town gas, 1800°C flame the inner cone temperature is about 2900°C and in an
7. Air-propane, 1750°C oxidizing flame it is about 3300°C.

Oxygen and Acetylene Cylinders


Oxy-acetylene Gas Welding
Oxygen for gas welding is stored in steel cylinder, 7 M3
Equipment for oxy-acetylene welding consists oxygen and
capacity and painted black for easy identification. Right
acetylene cylinders, welding torch, pressure regulator hose
Hand threads are used in cylinder valves and regulators etc.
and hose fittings etc.
for oxygen. It is stored under a pressure of 13.8 to 18.2 MPa
Combustion of acetylene with pure oxygen at the tip of
(about 150 kg/cm2).
the welding torch takes place in two steps. Acetylene reacts
Acetylene cylinder is made of steel and painted in
with oxygen from the oxygen cylinder to form carbon mon-
maroon colour. They and their valves, regulators etc are left
oxide and hydrogen. i.e, C2H2 + O2 = 2 CO + H2
hand threaded to avoid wrong usage of oxygen valves, regu-
This reaction takes place with in the white inner cone of
lators etc. Acetylene is stored around 1.5 MPa.
the flame.
Free acetylene is highly explosive if stored at a pres-
In the second stage, carbon monoxide and hydro-
sure more than 200 kPa. So it is used dissolved in acetone.
gen produced in the first reaction reacts with oxygen
Acetone can absorb 420 times its volume of acetylene at a
from the atmosphere to form carbon dioxide and water
pressure of 1.75 MPa. Acetylene molecules fit in between
vapor. i.e.,
acetone molecules, thus storing at a higher pressure is made
4 CO + 2H2 + 3O2 = 4 CO2 + 2H2O possible.
These reactions take place in the larger blue flame which Acetylene cylinder is filled with 80 to 85% porous mate-
surrounds the white inner cone. This larger blue flame con- rial (calcium silicate) and then filled with acetone.
tributes to a preheating effect in welding. It also protects the Acetylene is released from acetone at a slow rate.
molten metal from oxidation.
Chapter 3  •  Sheet Metal Working Joining Processes  |  3.825

Application of Different Brazing, Braze Welding and Soldering


Flames in Gas Welding Brazing, braze welding and soldering are metal joining
If possible, the welding should be done with a neutral flame processes by atomic bonding in which a filler material of
as it has minimum chemical effect on most heated metals. lower melting point than the parent metal is used to make
Neutral flame is used for welding of steel, cast iron, cop- the bond. In these processes the parent metal is not melt-
per and aluminium. ing as in the case of welding. Joint is obtained by means of
Materials that tend to absorb Carbon should not be diffusion of filler metal into the base metal and by surface
welded using reducing flame. Reducing flame can be used alloy formation.
for materials that oxidizes rapidly like steel and aluminium. Brazing is the coalescence of a joint with the help of a
Reducing flame can also be used for non-ferrous materi- filler metal whose liquids temperature is above 427°C and
als that do not absorb carbon. below the solids temperature of the base metal. Filler metal
Oxidizing flame has limited use. The presence of excess is drawn into the joint by capillary action. Flux usually used
oxygen with oxidizing flame causes an oxide film to form is borax. Due to the many advantages over welding, brazing
quickly which provides a protective cover over the base is very widely used in industries. Brazing can join almost all
metal pool. Due to this it is used in materials like brass metals. Dissimilar metals can be brazed. For example cast
and Zinc which have a tendency to separate and fume iron to stainless steel.
away. As temperature is less, there is less distortion in brazed
joints.
Braze welding is similar to brazing in the matter of tem-
Oxy-fuel Gas Cutting (OFC)
perature. The main difference is that the filler metal reaches
In cutting of metals using gas, the torch tip differs from that the joint by gravity instead of capillary action as the joint
of welding, For gas cutting, the torch tip has two sets of ori- gap is bigger. Thick layers of filler metal are deposited in
fices–an inner orifice for oxygen jet and surrounding it, the the joint.
outer orifices for oxygen-acetylene mixture. Braze welding is also known as bronze welding. The
It is possible to rapidly oxidize (burn) iron and steel filler rod usually consists of 60% copper and 40% Zinc,
when heated to a temperature of 800°C to 1000°C. Metal with small amounts deoxidizers such as silicon and tin.
burnt is blown off by the high pressure oxygen jet causing Metals with high melting point such as steel, cast iron, cop-
the cut. The cut formed is known as Kerf. per etc are braze welded.
Chemical reactions Soldering is the process of joining two metal pieces
3Fe + 202 = Fe3O4 + Heat by means of heat and a filler metal whose melting point is
2Fe + O2 = 2 FeO + Heat below 427°C and below the solids temperature of the metals
4 Fe + 3O2 = 2Fe2O3 + Heat to be joined. The filler metal known as solder is an alloy of
The heat from the above exothermic reactions is not suf- lead and tin. Higher Tin solders have greater strength and
ficient to maintain the kindling temperature. So the pre silvery appearance. The joint is filled by capillary action as
heating flames through the outer orifices are continued at in the case of brazing. Fluxes used are rosin and rosin plus
a lower rate. In the gas cutting process, about 30 to 40% of alcohol based fluxes for electrical soldering. Other fluxes
iron is simply blown off without forming oxides. used are zinc chloride and ammonium chloride.

Exercises
Practice Problems 1 on the punch is 2 mm. The blanking force during the
operation is
1. A 4 mm thick circular blank is used to make a cylindri- (A) 26.8 kN (B) 45.2 kN
cal cup of outside diameter 105 mm and height 25 mm. (C) 33.9 kN (D) 35.7 kN
The corner radius is 6 mm. The trimming allowance
3. In arc welding of steel, voltage and current are 20 V
provided is 3 mm. The blank diameter is
and 200 A respectively. Travel speed is 5 mm/s and
(A) 140 mm (B) 147 mm
cross sectional area of joint is 20 mm2. Heat required
(C) 150 mm (D) 145 mm
to melt steel is 10 J/mm3 and heat transfer efficiency is
2. 10 mm diameter holes are to be punched in a steel sheet 0.85. The melting efficiency in the welding is
of 4 mm thickness. Shear strength of the materials is (A) 29.4% (B) 35.2%
450 N/mm2 and penetration is 40%. Shear provided (C) 27.5% (D) 22.7%
3.826  |  Part III  •  Unit 7  •  Manufacturing Technology

4. The spot welding of sheets of 1 mm thickness, a current (A) 24.3 mm (B) 25.2 mm
of 10000 A was required for 8 seconds. Heat generated (C) 24.8 mm (D) 24.4 mm
in joules, assuming effective resistance as 10 mW, is 1 2. A 100 mm diameter hole is to be punched in a steel
(A) 1000 J (B) 800 J plate of 6 mm thickness. Shear strength of the plate is
(C) 900 J (D) 700 J 550 N/mm2. Cutting is complete at 40% penetration of
5. In the above problem, if 5 mm diameter electrodes are the punch with normal clearance.If a 250 kN press is
used the temperature rise will be (assume heat gen- to be used for this work, what is the shear angle to be
erated is confined to the volume of material directly provided on this punch, assuming a balanced shear.
between the two electrodes and temperature is distrib- (A) 8.59°C (B) 9.23°C
uted uniformly and sp. Heat of steel is 0.46) (C) 11.35°C (D) 7.62°C
(A) 5397°C (B) 6492°C 13. In blanking operations ‘shear’ is provided on
(C) 6860°C (D) 5574°C (A) Punch
6. In a DC arc welding operation, the voltage – Arc length (B) Both punch and die
characteristic was obtained as Varc = 20 + 5 L where the (C) Not provided
arc length L was varied between 5 mm and 7 mm. Here (D) Die
Varc denotes the arc voltage in volts. The arc current was 14. In piercing operation, the clearances is provided on
varied from 400 A to 500 A. Assuming linear power (A) Punch
source characteristic, the open circuit voltage and short (B) Die
circuit current for the welding operation are (C) Half on punch and half on die
(A) 45 V, 450 A (B) 75 V, 750 A (D) Either on punch or die (designer’s divia)
(C) 95 V, 950 A (D) 150 V, 1500 A
15. In gas welding, neutral flame inner cone has a
7. Two sheets of low carbon steel 1.5 mm thick each are temperature about
spot welded by passing a current of 10,000 amp for (A) 3200°C
5 Hz to 50 Hz supply. The maximum intendation is (B) 2100°C
10% of sheet thickness and density of spot weld nugget (C) 1000°C
is 8 gm/mm3. If 1380 J are required to melt one gram (D) 1250°C
of steel. Find the percentage of heat actually utilized
16. Carburising flame is used to weld
in making the spot weld. assume effective resistance
(A) Steel, cast iron etc.
as 200 mW and d = 6 t to determine nugget diam-
(B) Brass and bronze
eter. Also assume the nugget size to be equal to metal
(C) Hard surfacing materials such as stellite
between the two electrodes
(D) All of the above
(A) 72.13% (B) 63.24%
(C) 58.73% (D) 66.91% 17. In arc welding temperature of heat produced by the
electric arc is of the order of
8. A metal disc of 20 mm dia is to be punched from a
(A) 3000°C – 4000°C
sheet of 3 mm thickness. The punch and die clearance
(B) 4000°C – 5000°C
is 3%. The required punch diameter will be
(C) 5000°C – 6000°C
(A) 19.88 mm (B) 20.06 mm
(D) 6000°C – 7000°C
(C) 19.82 mm (D) 20.12 mm
18. In DC arc welding when work is connected to the posi-
9. A 25 mm square hole is to be cut in sheet metal 1 mm
tive terminal it is called a
thick. Shear strength of the material is 2900 kg/cm2
(A) Straight polarity
The cutting force in kN is
(B) Reversed polarity
(A) 21.45 (B) 28.42
(C) Cross polarity
(C) 32.12 (D) 25.31
(D) None of the above
10. A cup of 75 mm diameter and 40 mm deep in to be
19. In resistance welding, the voltage required for heating is
drawn in 2 mm thick material. The maximum drawing
(A) 1 to 5 V (B) 11 to 20 V
force, if the ultimate tensile stress of the material is
(C) 6 to 10 V (D) 50 to 100 V
3000 kg/cm2, will be
(A) 138.7 kN (B) 142.8 kN 20. The welding process used in joining mild steel shanks
(C) 141.4 kN (D) 135.3 kN to high speed drills is
(A) Spot welding
11. A blank of 2.5 mm diameter is to be made from a sheet
(B) Flash butt welding
of 2 mm thickness. Shear strength of sheet material is
(C) Seam welding
295 MPa. The size of the punch will be
(D) Projection welding
Chapter 3  •  Sheet Metal Working Joining Processes  |  3.827

Practice Problems 2
1. Blanking and piercing operation can be performed (A) 797.2 (B) 822.3
simultaneously in a (C) 843.8 (D) 861.1
(A) Simple die 10. A DC welding machine with a linear power source
(B) Progressive die characteristic provides open circuit voltage of 80 V and
(C) Combination die short circuit current of 800 A. During welding with the
(D) Compound die machine the measured arc current is 500 A. correspond-
2. The operation of cutting a triangular hole in a sheet ing to an arc length of 5 mm. and the measured arc cur-
metal using a punch and die is under the process rent is 460 A corresponding to an arc length of 7 mm.
(A) Shearing (B) Piercing The voltage–arc length characteristic can be given as
(C) Punching (D) Blanking (A) 20 + 8 L (B) 80 + 2 L
3. Cutting a sheet metal through part of its length and then (C) 20 + 2 L (D) 80 + 8 L
bending the cut portion is called 11. The force requirement in a blanking operation of low
(A) Stitting (B) Lancing carbon steel sheet is 6 kN. If the diameter of the blanked
(C) Nibbling (D) Notching part is increased to 1.5 times and thickness of the sheet
4. Two one mm thick sheets are to be spot welded at a is reduced to 0.4 times. The blanking force in kN will
current of 4500 A. Assuming effective resistance as 250 be
micro ohms and current flow time as 0.2 second, heat (A) 3.6 (B) 3.3
generated during the process will be (C) 3.0 (D) 2.8
(A) 1013 J (B) 1020 J 12. A contour having a perimeter of 250 mm is pierced out
(C) 1000 J (D) 955 J from a 3 mm sheet having an ultimate shear strength of
5. Which one of the following is a solid state welding 250 N/mm2. What will be the amount of shear, if the
process punch force is to be reduced to 60%. Assume a penetra-
(A) Electrons beam welding tion of 30%.
(B) Friction welding (A) 0.3 mm (B) 0.6 mm
(C) Thermit welding (C) 3 mm (D) 6 mm
(D) Percussion welding 13. Estimate reduction in piercing load for producing cir-
6. Which of the following use non-consumable electrode cular hole of 50 mm diameter in a 3 mm thick steel strip
(A) GMAW (B) SAW when the punch was provided with a shear of 1 mm.
(C) GTAW (D) SMAW Assume 30% penetration and shear strength of steel as
400 N/mm2
7. Match the correct combination:
(A) 48.34% (B) 52.63%
P  Blanking 1.  Tension (C) 45.83% (D) 47.91%
Q  Stretch forming 2.  Compression 14. The voltage arc length characteristic of a power source
R  Coining 3.  Shear is V = 20 + 40 L. Where V = operating voltage and
S  Deep drawing 4.  Tension and compression L = arc length. Determine the open circuit voltage and
5.  Tension and shear
short circuit current for arc length varying from 3 to 5
mm and current from 400 to 500 amp during welding
(A) P-2, Q-1, R-3, S-4 operation
(B) P-3, Q-4, R-1, S-5 (A) 540 V, 675 A (B) 560 V, 695 A
(C) 520 V, 720 A (D) 580 V, 710 A
(C) P-3, Q-1, R-2, S-4
(D) P-5, Q-3, R-3, S-1 15. A circular piece of 25 mm diameter is to be blanked
from a sheet of thickness 2 mm. Radial clearance in the
8. A shell of 100 mm diameter and 90 mm height with punch and die is 0.06 mm. Die allowance is 0.05 mm.
corner radius of 4 mm is to be produced by cup draw- The punch size in mm will be
ing. The required blank diameter in mm is (A) 25.01 mm (B) 24.89 mm
(A) 228 (B) 224 (C) 25.17 mm (D) 24.83 mm
(C) 232 (D) 215
16. A hole of 25 mm diameter is to be punched in a sheet
9. In an arc welding process, voltage and current are 25 V of 2 mm thickness, the shear strength of which is
and 300 A respectively. The arc heat transfer efficiency 290 MPa. Estimate the size of the die required
is 0.90 and welding speed is 8 mm/s. The net heat input (A) 25.22 mm (B) 25.42 mm
in J/mm is (C) 24.88 mm (D) 24.78 mm
3.828  |  Part III  •  Unit 7  •  Manufacturing Technology

17. In arc welding of steel with a potential of 20 V and a 22. A blank holder is used in deep drawing to
current 230 A; travel speed of the rod is 4 mm/s. Cross (A) Guide the punch through the die
sectional area of the joint is 20 mm2. Heat requirement (B) Exactly locate the blank on the die
for melting is 12 J/mm3 and melting efficiency is 40%. (C) Avid wrinkles
The heat transfer efficiency in the welding is (D) None of the above
(A) 48% (B) 52% 23. Spinning operation is carried out on
(C) 46% (D) 56% (A) Hydraulic press
18. In a spot welding, two 1 mm thick plates in a lap joint (B) Mechanical press
are welded using a current of 10000 A for 0.1 sec. The (C) Lathe
effective resistance of the joint is 100 mW. The joint (D) Milling machine
can be considered as a cylinder of 5 mm diameter and 24. Cutting and forming operations are performed in single
1.5 mm height. Assuming density of steel as 0.0079 operation in
gm/mm3 and heat required for melting steel as 1380 J/ (A) Simple die (B) Combination die
gm efficiency of the welding is (C) Progressive die (D) Compound die
(A) 36.2% (B) 52.3%
25. In progressive dies
(C) 32.1% (D) 41.8%
(A) Two or more cutting operation can be performed
19. Two 1 mm thick sheets are spot welded by passing cur- simultaneously
rent for 0.1 sec through the electrodes. The resultant (B) Cutting and forming operations can be combined
weld nugget formed is 5 mm in diameter and 1.5 mm and carried in simple operations
thick. If the latent heat of fusion of steel is 1400 kJ/kg (C) Work piece moves from one station to other with
and the effective resistance is 200 micro ohms, the current separates operations performed at each station
passed through was (assume steel density – 8000 kg/m3) (D) All of the above
(A) 1480 A (B) 3300 A
26. Gases used in TIG welding are
(C) 4060 A (D) 9400 A
(A) Hydrogen and oxygen
20. In arc welding process of a butt joint area of cross sec- (B) CO2 and H2
tion of weld is 5 mm2 and unit energy required to melt (C) Argon and neon
the metal is 10 J/mm3. If welding input power is 2 kW, (D) Argon and helium
and melting and heat transfer efficiency are 0.5 and 0.7 27. Temperature of plasma torch is the order of
respectively the welding speed will be (A) 1000°C (B) 5000°C
(A) 14 mm/s (B) 34 mm/s (C) 10000°C (D) 10 – 100 A
(C) 24 mm/s (D) 4 mm/s
28. Current range in SMAW is
21. Two metallic sheets of 2 mm thickness each are spot (A) 10 – 500 A (B) 10 – 50 A
welded in a lap joint with a welding current of 10000 A (C) 100­  –  200 A
  (D) 10 – 100 A
and welding time of 10 milli second. A spherical fusion 29. The flux commonly used in brazing is
zone of radius 2 mm is formed. If melting temperature (A) Rosin plus alcohol
is 1793 K, density is 7000 kg/m3, latent heat of fusion =
(B) Zinc chloride
300 kJ/kg, specific heat = 800 J/kg K and ambient tem- (C) Copper
perature = 293 K, the melting efficiency of the process (D) Tin and lead alloys
is (assume contact resistance = 500 mW)
30. In hard soldering filler material used is
(A) 50.38%
(A) Aluminium alloys
(B) 70.38%
(B) Silver alloys
(C) 60.38%
(C) Copper
(D) 80.38%
(D) Tin and lead alloys

Previous Year’s Questions


1. Two 1 mm thick steel sheets are to be spot welded at a 2. 10 mm diameter holes are to be punched in a steel
current of 5000 A. assuming effective resistance to be sheet of 3 mm thickness. Shear strength of the mate-
200 micro-ohms and current flow time of 0.2 second, rial is 400 N/mm2 and penetration is 40%. Shear
heat generated during the process will be [2004] provided on the punch is 2 mm. The blanking force
(A) 0.2 J (B) 1 J during the operation will be [2004]
(C) 5 J (D) 1000 J (A) 22.6 kN (B) 37.7 kN
(C) 61.6 kN (D) 94.3 kN
Chapter 3  •  Sheet Metal Working Joining Processes  |  3.829

3. Spot welding of two 1 mm thick sheets of steel (den- 8. Two metallic sheets, each of 2.0 mm thickness, are
sity = 8000 kg/m3) is carried out successfully by pass- welded in a lap joint configuration by resistance spot
ing a certain amount of current of 0.1 second through welding at a welding current of 10 kA and welding
the electrodes. The resultant weld nugget formed is 5 time of 10 ms. A spherical fusion zone extending
mm in diameter and 1.5 mm thick. If the latent heat of up to the full thickness of each sheet is formed. The
fusion of steel is 1400 kJ/kg and the effective resist- properties of the metallic sheets are given as:
ance in the welding operation is 200 mW, the cur- Ambient temperature = 293 K
rent passing through the electrodes is approximately
Melting temperature = 1793 K
 [2005]
(A) 1480 A (B) 3300 A Latent heat of fusion = 300 kJ/kg
(C) 4060 A (D) 9400 A Density = 7000 kg/m3
4. Match the items in columns I and II. [2006] Specific heat = 800 J/kgK
Assume: (i) contact resistance along sheet-sheet
Column I Column II
interface is 500 micro-ohm and along electrode-sheet
P  Wrinkling 1.  Yield point elongation interface is zero; (ii) no conductive heat loss through
Q  Orange peel 2.  Anisotropy the bulk sheet materials; and (iii) the complete weld
R  Stretcher strains 3.  Large grain size fusion zone is at the melting temperature.
S  Earing 4.  Insufficient blank holding force The melting efficiency (in%) of the process is [2007]
5.  Fine grain size
(A) 50.37
(B) 60.37
6.  Excessive blank holding force
(C) 70.37
(A) P-6 Q-3 R-1 S-2 (D) 80.37
(B) P-4 Q-5 R-6 S-1 9. Match the correct combination for following metal
(C) P-2 Q-5 R-3 S-4 working processes. [2007]
(D) P-4 Q-3 R-1 S-2
Processes Associated state of stress
5. In an arc welding process, the voltage and current are
P  Blanking 1.  Tension
25 V and 300 A respectively. The arc heat transfer
efficiency is 0.85 and welding speed is 8 mm/sec. The Q  Stretch forming 2.  Compression
net heat input (in J/mm) is [2006] R  Coining 3.  Shear
(A) 64 S  Deep drawing 4.  Tension and compression
(B) 797
5.  Tension and Shear
(C) 1103
(D) 79700 (A) P-2, Q-1, R-3, S-4
6. Which one of the following is a solid state joining (B) P-3, Q-4, R-1, S-5
process? [2007] (C) P-5, Q-4, R-3, S-1
(A) Gas tungsten arc welding (D) P-3, Q-1, R-2, S-4
(B) Resistance spot welding 1 0. The force requirement in a blanking operation of low
(C) Friction welding carbon steel sheet is 5.0 kN. The thickness of the
(D) Submerged arc welding sheet is ‘t’ and diameter of the blanked part is ‘d’. For
7. A direct current welding machine with a linear power the same work material, if the diameter of the blanked
source characteristic provides open circuit voltage of part is increased to 1.5 d and thickness is reduced to
80 V and short circuit current of  800 A. During weld- 0.4 t, the new blanking force in kN is [2007]
ing with the machine, the measured arc current is 500 A (A) 3.0 (B) 4.5
corresponding to an arc length of 5.0 mm and the (C) 5.0 (D) 8.0
measured arc current is 460 A corresponding to an arc 11. In arc welding of a butt joint, the welding speed is to
length of 7.0 mm. The linear voltage (E)–arc length be selected such that highest cooling rate is achieved.
(L) characteristic of the welding arc can be given as Melting efficiency and heat transfer efficiency are 0.5
(where E is in Volt and L is in mm) [2007] and 0.7, respectively. The area of the weld cross sec-
(A) E = 20 + 2 L tion is 5 mm2 and the unit energy required to melt the
(B) E = 20 + 8 L metal is 10 J/mm3. If the welding power is 2 kW, the
(C) E = 80 + 2 L welding speed in mm/s is closest to [2008]
(D) E = 80 + 8 L (A) 4 (B) 14
(C) 24 (D) 34
3.830  |  Part III  •  Unit 7  •  Manufacturing Technology

12. In the deep drawing of cups, blanks show a tendency 18. In a DC arc welding operation, the voltage-arc length
to wrinkle up around the periphery (flange). The most characteristic was obtained as Varc = 20 + 5l where
likely cause and remedy of the phenomenon are, the arc length l was varied between 5 mm and 7 mm.
respectively [2008] Here Varc denotes the arc voltage in Volts. The arc cur-
(A)  Buckling due to circumferential compression; rent was varied from 400 A to 500 A. Assuming linear
increase blank holder pressure power source characteristic, the open circuit voltage
(B) High blank holder pressure and high friction; re- and the short circuit current for the welding operation
duce blank holder pressure and apply lubricant are [2012]
(C) High temperature causing increase in circumfer- (A) 45 V, 450 A (B) 75 V, 750 A
ential length; apply coolant to blank (C) 95 V, 950 A (D) 150 V, 1500 A
(D)  Buckling due to circumferential compression;
19. Match the pairs: [2013]
decrease blank holder pressure
Processes Characteristics/Applications
13. The operation in which oil is permeated into the pores
of a powder metallurgy product is known as [2011] P  Friction welding 1  Non-consumable electrode
(A) Mixing Q  Gas metal arc welding 2  Joining of thick plates
(B) Sintering R  Tungsten inert gas 3  Consumable electrode wire
(C) Impregnation welding
(D) Infiltration S  Electroslag welding 4  Joining of cylindrical dissimilar
14. Which one among the following welding processes materials
uses non-consumable electrode? [2011]
(A) P-4, Q-3, R-1, S-2
(A) Gas metal arc welding
(B) P-4, Q-2, R-3, S-1
(B) Submerged arc welding
(C) P-2, Q-3, R-4, S-1
(C) Gas tungsten arc welding
(D) P-2, Q-4, R-1, S-3
(D) Flux coated arc welding
15. The shear strength of a sheet metal is 300 MPa. The 2 0. The major difficulty during welding of aluminium is
blanking force required to produce a blank of 100 mm due to its [2014]
diameter from a 1.5 mm thick sheet is close to[2011] (A) High tendency of oxidation
(A) 45 kN (B) 70 kN (B) High thermal conductivity
(C) 141 kN (D) 3500 kN (C) Low melting point
(D) Low density
16. Match the following metal forming processes with
their associated stresses in the work piece. [2012] 21. In solid-state welding, the contamination layers
between the surfaces to be welded are removed by
Metal forming process Type of stress
 [2014]
1.  Coining P  Tensile
(A) Alcohol
2.  Wire drawing Q  Shear (B) Plastic deformation
3.  Blanking R  Tensile and compressive (C) Water jet
4.  Deep drawing S  Compressive (D) Sand blasting
(A) 1-S, 2-P, 3-Q, 4-R 22. A rectangular hole of size 100 mm × 50 mm is to be
(B) 1-S, 2-P, 3-R, 4-Q made on a 5 mm thick sheet of steel having ultimate
(C) 1-P, 2-Q, 3-S, 4-R tensile strength and shear strength of 500 MPa and
(D) 1-P, 2-R, 3-Q, 4-S 300 MPa, respectively. The hole is made by punching
process. Neglecting the effect of clearance, the punch-
17. Calculate the punch size in mm, for a circular blank- ing force (in kN) is [2014]
ing operation for which details are given below: (A) 300 (B) 450
 [2012] (C) 600 (D) 750
Size of the blank 25 mm
Thickness of the sheet 2 mm 23. For spot welding of two steel sheets (base metal) each
Radial clearance between of 3 mm thickness, welding current of 10000 A is
punch and die 0.06 mm applied for 0.2 s. The heat dissipated to the base metal
Die allowance 0.05 mm is 1000 J. Assuming that the heat required for melting
1 mm3 volume of steel is 20 J and interfacial contact
(A) 24.83 (B) 24.89 resistance between sheets is 0.0002 W, the volume
(C) 25.01 (D) 25.17 (in mm3) of weld nugget is ____ [2014]
Chapter 3  •  Sheet Metal Working Joining Processes  |  3.831

24. Within the heat affected zone (HAZ) in a fusion weld- 29. Under optimal conditions of the process the tempera-
ing process, the work material undergoes [2014] tures experienced by a copper work piece in fusion
(A) Microstructural changes but does not melt welding, brazing and soldering are such that [2016]
(B) Neither melting nor microstructural changes (A) Twelding > Tsoldering > Tbrazing
(C) Both melting and microstructural changes after (B) Tsoldering > Twelding > Tbrazing
solidification (C) Tbrazing > Twelding > Tsoldering
(D) Melting and retains the original microstructure (D) Twelding > Tbrazing > Tsoldering
after solidification 30. The welding process which uses a blanket of fusible
25. A butt weld joint is developed on steel plates having granular flux is: [2016]
yield and ultimate tensile strength of 500 MPa and (A) Tungsten inert gas welding
700 MPa, respectively. The thickness of the plates is (B) Submerged arc welding
8 mm and width is 20 mm. Improper selection of (C) Electroslag welding
welding parameters caused an undercut of 3 mm (D) Thermit welding
depth along the weld. The maximum transverse ten- 31. The voltage-length characteristic of a direct current
sile load (in kN) carrying of the developed weld joint arc in an arc welding process is V = (100 + 40l),
is ____  [2014] where l is the length of the arc in mm and V is arc
26. A DC welding power source has a linear voltage-cur- voltage in volts. During a welding operation, the arc
rent (V-I) characteristic with open circuit voltage of length varies between 1 and 2 mm and the welding
80 V and a short circuit current of 300 A. For maxi- current is in the range 200–250 A. Assuming a linear
mum arc power, the current (in Amperes) should be power source, the short circuit current is ________ A.
set as ______. [2015]  [2016]
27. During a TIG welding process, the arc current and arc 32. Spot welding of two steel sheets each 2 mm thick is
voltage were 50 A and 60 V, respectively, when the carried out successfully by passing 4 kA of current
welding speed was 150 mm/min. In another process, for 0.2 seconds through the electrodes. The resulting
the TIG welding is carried out at a welding speed of weld nugget formed between the sheets is 5 mm in
120 mm/min at the same arc voltage and heat input diameter. Assuming cylindrical shape for the nugget,
to the material so that weld quality remains the same. the thickness of the nugget is _______ mm. [2016]
The welding current (in A) for this process is: [2015]
Latent heat of fusion for steel 1400 kJ/kg
(A) 40.00 (B) 44.72
Effective resistance of the weld joint 200µΩ
(C) 55.90 (D) 62.25
Density of steel 8000 kg/m3
28. Which two of the following joining processes are
autogeneous? [2015] 33. In a sheet metal of 2 mm thickness a hole of 10 mm
(i) Diffusion welding diameter needs to be punched. The yield strength in
(ii) Electroslag welding tension of the sheet material is 100 MPa and its ulti-
(iii) Tungsten inert gas welding mate shear strength is 80 MPa. The force required to
(iv) Friction welding punch the hole (in kN) is _________. [2016]
(A) (i) and (iv) (B) (ii) and (iii)
(C) (ii) and (iv) (D) (i) and (iii)
3.832  |  Part III  •  Unit 7  •  Manufacturing Technology

Answer Keys
Exercises
Practice Problems 1
1.  C 2.  B 3.  A 4.  B 5.  D 6.  C 7.  D 8.  C 9.  C 10.  A 
11.  C 12.  A 13.  D 14.  B 15.  A 16.  C 17.  D 18.  A 19.  C 20.  B 

Practice Problems 2
1.  D 2.  B 3.  B 4.  A 5.  B 6.  C 7.  C 8.  D 9.  C 10.  C 
11.  A 12.  B 13.  B 14.  A 15.  D 16.  A 17.  B 18.  C 19.  C 20.  A 
21.  C 22.  C 23.  C 24.  B 25.  C 26.  D 27.  D 28.  A 29.  C 30.   B

Previous Years’ Questions


1.  D 2.  None 3.  C 4.  D 5.  B 6.  C 7.  A 8.  C 9.  D 10.  A
11.  B 12.  A 13.  C 14.  C 15.  C 16.  A 17.  A 18.  C 19.  A 20.  A
21.  B 22.  B 23.  140 to 160 24.  A 25.  68 to 72 26.  149 to 151
27.  A 28.  A 29.  D 30.  B 31.  423 to 428 32.  2.85 to 2.95 33.  4.9–5.1
Chapter 4
Machining and Machine Tool
Operations
chapter highlights

☞☞ Mechanics of Machining ☞☞ Tool Wear


☞☞ Geometry of Single Point Cutting Tool ☞☞ Tool Life
☞☞ Tool Signature or Tool Designation ☞☞ Principal Elements in Metal Machining
☞☞ Chip Thickness Ratio ☞☞ Material Removal Rate (MRR) in Turning
☞☞ Forces on the Chips ☞☞ Grinding
☞☞ Merchant Theory ☞☞ Non-traditional Machining Processes
☞☞ Lee and Shaffer Theory ☞☞ Electrochemical Machining Process
☞☞ Heat Generation During Metal Cutting ☞☞ Electric Discharge Machining (EDM)

Mechanics of Machining 2. According to the method of cutting


(i) Right hand tools
Metal cutting or machining is the process of removing
(ii) Left hand tools
unwanted material from a block metal in the production of
3. According to the use of the tool
a dimensional work piece.
(i) Turning tool
In metal cutting, a wedge shaped tool is made to move
(ii) Facing tool
relative to the work piece. The tool exerts pressure on the
(iii) Parting tool
metal resulting in a compression near the tool tip. Material
(iv) Thread cutting tool
ahead of the tool is sheared continuously along a plane
(v) Boring tool etc.
called ‘shear plane’. Cutting edge of the tool is formed
by two intersecting surfaces called rake surface and flank. Solid tool is formed by grinding a tool steel stock.
Rake surface is the top surface along which the metal chips Forged tool is made by forging of high carbon steel or
formed by cutting moves upwards. The other surface, flank high speed steel and then hardened and ground.
is relieved from rubbing with the machined surface. In tipped type tool a tip of high grade material is fixed to
the shank of a low grade material by brazing.
In bit type tool the high grade tool tip of square or rec-
Tool Geometry tangular shape is held mechanically in a tool holder by
Cutting tools are classified into two groups clamping.
1. Single point tools A right hand cutting tool or right cut tool will cut the
2. Multipoint tools material when the tool is fed from right to left. The cutting
edge will be on the thump side of right hand with palm kept
Single point tools have one cutting edge. The cutting edge is down ward and the fingers pointed towards the tool nose.
also called point. These may be further classified as follows. For left hand or left cut tool the cutting edge is the other side
1. According to the method of manufacturing which can be identified by using left hand.
(i) Solid tool
Multipoint Cutting Tools
(ii) Forged tool
(iii) Tipped type tool Multipoint cutting tools have more than one effective cut-
(iv) Bit type tool ting edge to remove the excess material. These can have
3.834  |  Part III  •  Unit 7  •  Manufacturing Technology

rotary travel as in the case of drilling or milling cutters or a Side rake angle: It is the angle between the tool face and
linear travel as in the case of broaching tools. a line parallel to the base of the tool and measured in a plane
perpendicular to the base at the side cutting edge. Side rake
Elements of Single Point Tool is negative, if the slope is towards the cutting edge. It also
The elements of the single point tool is given in the follow- guides the direction chip away from the job. Amount of
ing figure. bending of a chip depends upon the angle.
End relief angle: It is the angle between a plane per-
pendicular to the base and end flank. This angle prevents
Shank cutting tool from rubbing against the job.
Side relief angle: It is the angle, made the side flank of
Face the tool and a plane perpendicular to the base just under the
Nose
cutting edge. This angle permits the tool to be fed sideways
Base
End cutting into the job, so that it can cut without rubbing.
edge End cutting edge angle: It is the angle between the end
Side (main) cutting
End flank edge cutting edge and a line normal to the tool shank side. It acts
as a relief angle that allows only a small section of the end
Side flank
cutting edge to contact the machine surface.
Face: It is the surface over which the chips flow. Side cutting edge angle: It is also known as the lead
Shank: It is the surface below the cutting edge. angle. It is the angle between side cutting edge and side of
Nose: It is the junction of side and end cutting edges. the tool shank. It avoids the formation of built up edges,
Side cutting edge: It is the intersection of face and side controls the direction of chip flow and distributes the cut-
flank. Main cutting work is done by this cutting edge. ting force and heat produced over large cutting edge.
End or auxiliary cutting edge: It is the intersection of face Nose angle: It is angle between the two cutting edges.
and end flank Nose radius is provided to increase the finish and strength of
the cutting tip of the tool. It increases tool life and provides
Geometry of Single Point Cutting Tool good surface finish. As nose radius is increased from zero
In a single point tool there are various angles. Each angle values there is improvement in surface finish and permissi-
has definite purpose. ble cutting speed. But too large a nose radius leads to chatter
of the tool.
Ce Tool angle specification system or tool nomenclature
Face Shank
system.
1. ASA system
Nose angle 2. Orthogonal rake system (ORS)
Cs
3. British system or Maximum normal rake system (MRS)
4. German (DIN) system
αb
αs In ASA system the geometry of the rake face is expressed in
terms of the side rake angle and back rake angle. The back
θs Lip Shank rake angle is the angle between the rake face and the base
angle
of the tool measured in a plane perpendicular to the base of
θe Base
and parallel to the longitudinal axis of the tool. Side rake
angle is measured in a plane normal to the plane in which
ab – Back rake angle side rake angle is measured.
as – Side rake angle
qe – End relief angle Tool Signature or Tool Designation
qs – Side relief angle A single point cutting tool may be specified by a sequence
Ce– End cutting edge angle of numbers which represents various tool angles and nose
Cs – Side cutting edge angle radius. Tool angles have been standardized by American
Back rake angle or top rake angle is the angle between Standards Associations (ASA). Under ASA system, tool
the face of the tool and a line parallel to the base of the tool signature comprises seven elements. These are in the order
and measured in a plane (perpendicular) through the side given below.
cutting edge. Its purpose is to guide the direction of chip Back rake angle (ab)
flow. The size of the angle depends upon the material to Side rake angle (as)
be machined. Negative rake angle is used for high tensile End relief angle (qe)
strength materials. Side relief angle (qs)
Chapter 4  •  Machining and Machine Tool Operations  |  3.835

End cutting edge angle (Ce) α1 r


r1 α
Side cutting edge angle (Cs)
A
And nose radius (R) β
If a tool signature is
10, 10, 6, 6, 8, 8, 2 N M Section N-N
it means that λ
Section
ab - 10° M -M
as - 10°
qe - 6° M
qs - 6° N
Ce - 8°
Cs - 8° i
And nose radius = 2 mm View from A
In ASA system angles are specified with out considering the ORS system
position of cutting edge. So it does not give the true behav-
iour of cutting tool.
According to British system rake angle is specified as the
In orthogonal rake system (ORS) this problem is
steepest slope of the rake face.
rectified.
Orthogonal rake system is also known as International Types of Metal Cutting Processes
system (ISO). The end rake angle is the angle between the
base plane and the rake face, measured in a plane normal Depending up on the movement of cutting edge with respect
to the end cutting edge. The side rake angle is the angle to the direction of relative work tool motion, metal cutting
between the base plane and the rake face, measured in a process is classified into two
plane normal to the side cutting edge. This plane is known 1. Orthogonal cutting process or two dimensional cutting
as orthogonal plane, or the chief plane. The angles in this 2. Oblique cutting process or three dimensional cutting
plane are side relief angle (r).
Side rake angle (a) In orthogonal cutting edge of tool is arranged perpendicular
Wedge (lip angle) (b) to the velocity vector V, where as in oblique cutting it is set
And the cutting angle (d) at some angle.
In ORS, back rake angle is measured through the side
(principal) cutting edge. It is the inclination angle (i)
between the principal cutting edge. And a line passing
Types of Chips
through the point of tool parallel to the principal plane, The type of chip formed is a function of the work material
measured in a plane perpendicular to basic plane passing and cutting conditions. Chips can be classified into 3 types
through the cutting edge. 1. Continuous chips
Tool designation for ORS is 2. Discontinuous chips
i - a - r - r1 - C -l - R 3. Continuous chip with cutting edge
e
where l is the plan approach angle.
b + r = d = cutting angle. Continuous chips are obtained from ductile materials such
as mildsteel. As very lengthy chip in coil form is unsafe and
αs hazardous it become necessary to break the chip for which
Section B-B chip breakers are used.
θs
A
Dicontinuous chips are obtained while machining brittle
B
materials like cast iron. Chips are small individual segments
B
Ce
which may adhere to each other, loosely. Low cutting speed,
θe small rake angle etc. contributes to the formation of discon-
αb tinuous chips.
Cs
High pressure and temperature during the cutting causes
alloying and welding actions of chip material to tool face
Feed near the nose. This is called built up edge. The accumulated
built up of chip material break away. But some part adhere
A Section A-A to the chip and some to the work piece. Low cutting speeds
and larger depth of cut promote the formation of built up
ASA system edges.
3.836  |  Part III  •  Unit 7  •  Manufacturing Technology

Chip Thickness Ratio cos α


r = (1 − r sin α )
tan α
r cos α
t2 tan φ =
1 − r sin α

B α Forces on the Chips


t1 D
φ Fs F
Chip
C A
Tool
R1
R
t1 = thickness of chip before cutting s.p N
FN
t2 = thickness of chip after cutting
t
r = chip thickness ratio = 1 F
t2 Fs F R’
As there is no change in the volume of metal cut t

φ
t1 b1 L1 = t2 b2 L2 R
N
Fc
Where b1 and b2 are width of cut before and after the cut and FN
L1 and L2 are length of chip before and after cutting.
When there is no side flow of metal, b1 = b2 Force components on the chip
\ t1L1 = t2 L2 Merchant Theory
t1 L2 Merchant has worked out the relation ships amongst various
= = r <1
t 2 L1 forces. The following are the assumptions made
1. The tool is very sharp and there is no contact between
If side flow is to be considered, thickness ratio is to be mul-
b clearance face and work piece
tiplied by λ = 1 2. The chip does not flow to either side, i.e. there is no
b2
From the right angle triangle ABC, side spread
BC 3. A continuous chip with out built up edge is produced.
sin f = 4. The cutting velocity remains constant
AB 5. The chip behaves as a free body in stable equilibrium
BC t under the action of two equal, opposite and almost col-
∴ AB = = 1  (1)
sin ϕ sin ϕ linear resultant forces
From right angle triangle ABD Forces Fs and Fn acts on the chip, F along the shear plane and
Fn normal to the shear plane. Fn is the backing up force pro-
BD
sin (90 − f + α ) = vided by the work piece on the chip. R is the resultant of Fs
AB and Fn.
t2 Force F is the frictional resistance of the tool against the
AB =
sin (90 − ϕ + α ) motion of the chip and N is the force normal to the tool face.
R′ is the resultant of these forces.
t2   
= (2) i.e. R1 = F + N
cos (ϕ − α )    
From (1) and (2) R = Fs + Fn = F c + Ft

t1 t2 Merchant represented various forces inside a circle, with
=
sin φ cos( φ − α ) diameter R or R′ passing through the tool point.

t1 sin φ R α
Fs Tool
= Fc FN
t 2 cos( φ − α ) F φ
A
sin φ
i.e. r = Ft R’
α
cos φ cos α + sin φ sin α
R α
r cos α β
+ sin α = 1 D
F
tan φ N
Chapter 4  •  Machining and Machine Tool Operations  |  3.837

Q Fc cos( β − α )
=
R Fs cos(ϕ + β − α )

Fn F cos( β − α )
Fs
Fc = s
F Fc φ A cos(ϕ + β − α )
F β−α F Fc sin α + Ft cos α
=
P 4. N Fc cos α − Ft sin α
Ft
φ Ft + Fc tan α
R
=
Fc − Ft tan α
D
F
also = tan β = µ
N
Ft
and = tan(β − α )
Fc

A Velocity ratio
F
α β−α Tool
E
Vf
α
Vs
B φ
α Vc

β
D F C
N
Vf
C 90-(φ-α) Vs

In this diagram the horizontal component is the cutting tone α


Fc and the vertical component is the thrust force Ft. These 90-α φ
two are measured by using a dynamometer. t1, t2 and f are
found by calculations. Other components are expressed in B Vc A
terms of known parameters. Let Vc be the cutting velocity,
As chip slides over the tool face under pressure the i.e., velocity of the tool relative to the work piece
kinetic coefficient of friction (m) may be expressed as. Vf = flow velocity
F i.e, velocity of chip relative to tool
µ = = tan β
N Vs = velocity of shear
Other force relationships are i.e. velocity of displacement or formation of the newly
1. F = Ftcos a + Fc sina cut chip elements relative to the work piece.
N = Fccos a - Ft sina From the triangle ABC,
As F = AB + BC Vc Vf Vs
= =
= AB + DE sin 90 − (ϕ − α ) sin ϕ sin(90 − α )
= Fc sina + Ft cos a Vc Vf V
N = FB – FE = = s
= Fc cos a – Ftsina cos(ϕ − α ) sin ϕ cos α
2. Fs = Fccos f - Ftsinf Vc cos α
FN = Ft cos f + Fc sinf ∴Vs =
cos(ϕ − α )
as Fs = AQ ­– QR
AQ - FP = Fccos f - Ft sin f Vc sin ϕ
Vf =
FN = DR = DP + PR = DP + FQ cos(ϕ − α )
= Ft cosf + Fc sinf
3. Fc = AD cos (b - a) Condition for Minimum Cutting Force
= R cos (b - a) To find the value of shear angle f which makes cutting force
Fs = R cos (f + b - a) Fc a minimum, the following expression is used.
3.838  |  Part III  •  Unit 7  •  Manufacturing Technology

π Tool Life
2f + β − α =
2 Tool life is defined as the time clasped between two succes-
In the above expression (2f + b - a) is known as machin- sive grinding of the tool. It is expressed in minutes. It is the
ing constant total cutting time accumulated before total failure occurs.
Two most commonly used criteria for measuring tool life are
Lee and Shaffer Theory 1. Total destruction of the tool when it ceases to cut
Lee and Shaffer theory is based on theory of plasticity. In 2. A fixed size of wear land on tool flank
this theory, the following assumptions are made.
Tool Life Equation
1. Material ahead of tool in work piece behaves as an
FW Taylor gave the following relationship between cutting
ideal plastic
speed and tool life.
2. Chip hardening does not take place
VTn = C
3. Between chip and work piece, there is a slip plane
Where V = cutting speed in m/minute
Based on the above, Lee and Shaffer’s shear angle relation- T = tool life in minutes
ship is. n = tool life index
π = 0.1 to 0.15 for HSS tools
f + β −α = = 0.2 to 0.4 for tungsten carbide tools
4
= 0.4 to 0.6 for ceramic tools
For built up nose formation the above relationship was fur-
C = constant
ther modified by introducing a factor q, as
π Tool life also depends to a great extend on the depth of cut d
f + β − α −θ =
4 and feed rate per revolution, f. Assuming a logarithmic vari-
ation of C with d the equation can be written as VTn.dm = C
Heat Generation During Metal Cutting Considering feed rate also the general equation can be
VT n.dm.f  x = C
During metal cutting, heat is generated due to the rubbing
of tool with work piece and chip moving with high velocity Constants n and c for different work material and tool
Factors that cause excessive heat generation are material.

1. Very high cutting speed Work Tool material n C


material
2. Cutting tool of poor surface finish
3. Incorrectly ground or worn cutting tool Steel HSS 0.1–0.16 160–190
4. Built up edge formation on cutting tool Carbide 0.18–0.2 220–290
5. High friction between tool and work piece CI HSS 0.08–0.1 100–180
Carbide 0.2–0.28 250–325
Tool Wear
During a machining process, the tool is subjected to
forces, temperature and sliding action due to relative Solved Example
motion between tool and work piece. Cutting edge of Example 1:  While machining cast iron using a HSS tool,
the tool gradually wears out and needs regrinding as the tool life of 50 min was observed with a cutting speed of 100
wear increases. This leads to down time loss of produc- m/min. Tool life for a cutting speed of 80 m/min will be
tion etc. Tool wear can occur due to diffusion, adhesion, (Assume n = 0.09 in Taylor’s equation).
fatigue, oxidation, abrasion, chemical decomposition
etc. Solution:  V = 100 m/min
T = 50 min
Flank Wear and Crater Wear n = 0.09
Tool life equation
Flank wear produces wear areas on side and end flanks of VTn = C
the tool. log V + n log T = log C
Flank wear occur more at the corner and nose due to the log 100 + 0.09 log 50 = log C
sudden entry and sudden exit of the corner and nose por- C = 142.20
tions. It is also due to the work hardening of the material in \VT 0.09 = 142.2
the width direction.
142.2
Crater wear is due to diffusion and abrasion between chip T 0.09 =
and face of the tool, near the cutting edge. 80
T = 596.57 min
Chapter 4  •  Machining and Machine Tool Operations  |  3.839

Example 2: In an machining operation cutting speed is Material Tool life Cutting speed
reduced by 25% (min) (m/min)
Assuming n = 0.5 and C = 300 in Taylor’s equation, cal- A 25 100
culate the increase in tool life. 10 150
Solution:  N = 0.5,
B 40 200
V1T10.5 = 300 (1)
20 250
V2 = 0.75 V1
V 2T20.5 = 300
elative machinability, taking cutting speed for a tool life of
R
0.75 V, T20.5 = 300 (2)
50 min as criteria and material A as standard material is
From (1) and (2)
⎛ T2 ⎞
0.5 Solution:  Material A (standard material)
⎜⎝ T ⎟⎠ × 0.75 = 1 V1 = 100 m/min
1
T1 = 25 min
2
T2 ⎛ 1 ⎞ V2 = 150 m/min
=⎜ ⎟ = 1.78
T2 ⎝ 0.75 ⎠ T2 = 10 min

log V + n log T = log C
T2 − T1 T2
= −1 log 100 + n log 25 = log C
T1 T1
log 150 + n log 10 = log C
= 1.78 - 1 n (log 25 – log 10) = log 150 – log 100
= 0.78 n × 0.398 = 0.176
i.e. increase in tool life = 78% n = 0.44
Example 3:  A carbide cutting tool was used for machining
C = 415.5
a m.s. work piece at a cutting speed of 50 m/min. A tool
Taylor’s equation is
life of 100 min was obtained. Life of the tool when cutting
speed is increased by 25% is VT0.44 = 415.5
Speed at T = 50 m/m
Solution:  V1 = 50 m/min
415.5
T1 = 100 min V50 =
N = 0.26 500.44
V2 = 1.25 V1 = 74 m/min
V1 T1 = V2 T2
n n
Material B (test material)
0.26 log 200 + n log 40 = log C
⎛ T1 ⎞
⎜⎝ T ⎟ = 1.25 log 250 + n log 20 = log C
2⎠
N (log 40 - log 20) = log 250 - log 200
⎛ T1 ⎞ N = 0.32
⎜⎝ T ⎟⎠ = 2.36 C = 651
2
V50 = 186 m/min
T1 100 Relative machinability or machinability index
T2 = = = 42.37 min
2.36 2.36
V50 for B
= × 100
Machinability
V50 for A
Machinability may be defined as the earness or difficulty 186
with which a material can be machined under a given set of = × 100
74
conditions.
It is common to express machinability in terms of cutting = 251.4%
speed for a given tool in minutes. Cutting speed for produc- Machinability rating or index of different material is taken
ing a predetermined value of tool life, termed as the specific relative to the index which is standardized sometimes index
cutting speed, could be made as the basis of comparison of of free cutting steel is arbitrarily taken as 100%. Then,
machinability of materials. Machinability or machinability
index of test material can be expressed as [Cutting speed of test material
V for 20 minn tool life]
Machinability = t × 100% Machinability index % =
Vs [Cutting speed of free cutting
steel for romin.tool lifee]
Example 4: In assessing machinability for different
materials the following data was obtained.
3.840  |  Part III  •  Unit 7  •  Manufacturing Technology

Principal Elements in Metal Machining 35 × 1000


Rpm = N =
Principal elements in metal machining are π × 50
1. Cutting speed 700 × π × 50
2. Feed Time required (L/fN) = = 3.14 min
35 × 1000
3. Depth of cut
Example 6:  In orthogonal cutting of a work piece
Cutting speed is the distance travelled by the work surface in Feed force = 350 kgf
a unit time with reference to the cutting edge of the tool. It Cuting force = 150 kgf
is a relative term, since either the tool or the job or both may Chip thickness ratio = 0.44
π DN Rake angle = 10°
be moving during cutting. Cutting speed V = m/s
where N = rpm and D = diameter in mm. 1000 Calculate the shear angle, force Fs and Ft
Feed is the distance advanced by the tool relative to the Solution:  Let shear angle = f
job for each revolution of the work. It is generally expressed
r cos α
in mm/rev. Some times it is expressed in mm/min or mm/s tan ϕ =
1 − r sin α
and is called as feed rate. In machine tools such as shaper or
planer it is expressed as mm/stroke. 0.44 cos 10
=
Depth of cut is the measured perpendicular distance 1 − 0.44 sin10
between the machined surface and the un machined sur- = 0.4691
face of the job. Or it is the thickness of the layer of metal = 25.13°
removed in one cut, measured in a direction perpendicular   Fs = Fc cosf - Ft sinf
to the machined surface. It is expressed in mm. = 150 × 0.906 - 50 × 0.4225
= 114.8 kgf
Machining Time   Fn = Ft cos f + Fc sin f
L = 50 × 0.906 + 150 × 0.4225
T= = 108.7 kgf
fN
Example 7:  A 40 mm diameter and 150 mm long shaft is
Where L = length of cut
to be converted to a stepped shaft of dimensions as shown
f = feed, N = rpm
in the figure by turning.
Material Removal Rate (MRR) in Turning
Material removal rate is the volume of metal removed in 40 mm
30 mm
unit time. It is expressed in mm3/min 35 mm
t
50 mm 50 mm 50 mm

D d The depth of cut should not exceed 2.5 mm. Cutting speed
is to be 20 m/min and feed to be 0.3 mm/rev for each cut.
f = feed/revolution Estimate the machining time.
N = number of revolutions per minute (rpm) Solution:  Reduce 40 mm to 35 mm dia for a length of 100
Material removal rate mm. This can be done in a single cut.
π
MRR =
4
( D 2 − d 2 ) fN Machining time
L
T1 =

( D + d ) ( D − d ) fN fN
2 2 100
= pDavtfN mm /min
3 = min
0.3N 1
Where t = chip thickness in mm
Dav = average diameter of work piece in mm. 20 × 1000
but N1 = = 159.15 rpm
π × 40
Example 5:  Find the time required, for one complete cut on
100
a work piece 350 mm long and 50 mm in diameter. Cutting ∴ T1 = = 2.09 min
speed is 35 m/min and feed is 0.5 mm/rev. 0.3 × 159.15
20 × 1000
350 N2 =
Solution:  No. of revolution = = 700 π × 35
0.5
V = p DN = p × 50 × N = 35 m/min = 181.89
Chapter 4  •  Machining and Machine Tool Operations  |  3.841

50 From the above,


T2 =
0.3 × 181.89 w L (1 + m )
Tm =
= 0.92 min f 1000VC
Total machining time = T1 + T2
= 2.09 + 0.92 Drilling
= 3.01 min
Cutting speed (V) is the peripheral speed of the cutter
Operating Conditions in a Shaping Machine (drill bit)
Length of stroke π DN
Cutting speed VC = V= m/min
Time required for cutting stroke 1000
Length of stroke where D = drill diameter in mm
Return speed VR = N = rotational speed in rpm
Time required for return stroke
Feed is the distance that the drill enters the work piece
L L L (VC + VR ) for each revolution of the drill bit. It is expressed in mm/
∴ Time per cycle = + =
VC VR VC .VR revolution.

Machining Time
Feed
Feed is the relative distance moved by the work piece in a L + x+ y
Tm = =
direction perpendicular to the line of tool travel, per cycle. fN fN

w (VC + VR ) Where  = hole length or depth


Machining time Tm = L
f VC .VR x = tool approach
= 0.3 D
where w = width of the job y = tool over travel (1 to 2 mm)
L = length of stroke f = feed
f = feed per cycle Material removal rate
⎛ w⎞
⎜⎝ Total number of cycles or double strokes required = f ⎟⎠ . π D2
MRR = fN mm3/min
4
1
But time/cycle = min Grinding
N
Grinding is the process of material removal in the form of
Where N = number of cycles per minute
small chips from the surface of a work piece, by the abrasive
\ Machining time can also be obtained from the relation
action of a revolving wheel called grinding wheel. It is a
w finishing process for the production of high quality surface
Tm =
fN with close tolerance. It is also used for removal of excess
Volume of metal removed material from a work piece and machining of hard materials.
Material removal rate (MRR) = Grinding wheel is made of two main elements-abrasives
Machining time
and bonding agents. Abrasives are hard substances suitable
wdL for cutting action. Bonding agents hold the abrasive grains
=
w during grinding operation. So they should have good bind-
fN ing properties.
= fdLN mm3/min There are two types of abrasives – natural and synthetic
where d = depth of cut or manufactured. Sand abrasives, emery, corundum and dia-
Cutting speed mond are examples of natural abrasives. Silicon carbide and
aluminium oxide are examples of manufactured abrasives.
LN (1 + m )
VC = m/min Properties of Abrasive Grains
1000
Where m = ratio of return time to cutting time. A good abrasive should have uniform physical properties of
hardness, toughness and friability.
VC
= Hardness of abrasive grain is required to scratch or pen-
VR etrate the work piece during operation.
N = number of double strokes Toughness is required to absorb shock loads.
3.842  |  Part III  •  Unit 7  •  Manufacturing Technology

The ability to fracture under pressure is called friability. conventional grinding wheels truing and dressing are done
When cutting edges of grains become smooth, they break simultaneously.
off under pressure exposing new cutting edges.

Characteristics of a Grinding Wheel Non–Traditional Machining


Knowledge of characteristics of grinding wheels and abra-
sive particles are essential to understand accuracy, surface Processes
finish and material removal rate etc. that can be obtained Conventional manufacturing process include casting, form-
using different grinding wheels. The characteristics are grit, ing, joining etc. Non conventional resources such as laser,
grade and structure. plasma, chemicals etc. can be made use of in shaping mate-
rials. When these are applied for manufacturing processes
Grit or Grain Size they are called unconventional manufacturing process, which
It is the number of the finest sieve through which the grain include machining process also. These processes are also
pass. Smaller the grain size, larger is the number. For exam- called non-traditional machining processes or non-­traditional
ple grain size 10 is very coarse and grain size 100 is fine. manufacturing methods (NTMM) or new technology.
NTMM generally are non-mechanical. They do not pro-
Grade duce chips or a lay pattern on the work. There is no direct phys-
The term grade refers to the hardness with which the bond ical contact between tool and work piece. So tool need not be
holds the abrasive grains. It is indicated by English alpha- harder than the job as in the case of conventional machining.
bets A to Z, A denoting weakest and Z the strongest.
Classification
Structure NTMM can be classified in to various groups according to
The relative spacing of the abrasive grains in the bond is the type of energy used. They are
called structure. Dense structure is denoted by numbers 1 to
1. Mechanical
8 and open structure is denoted by 9 to 15 or higher. Open
2. Chemical
structure is used for soft and ductile materials and dense
3. Electro chemical
structure is used for brittle materials and finishing cuts.
4. Thermoelectric
Grinding Wheel Wear In mechanical energy methods, material is removed by
During use, a grinding wheel slowly wears out. The wear mechanical erosion of the work piece material. Energy
can be due to grain fracture, bond fracture or attrious wear. source is pneumatic or hydraulic pressure cutting tool. The
Due to attrious wear the cutting edges become dull. processes include
Loading and Glazing 1. Waterjet machining (WTM)
Chips formed during operation get entrapped in the inner 2. Ultra sonic machining (USM)
granular space of abrasive particles. This is called loading. 3. Abrasive jet machining
When the bond of the abrasive wheel is very hard, abra- Chemical energy methods involve controlled etching of the
sive particles are not dislodged and become blunt leading to work piece material in contact with a chemical solution.
a shining appearance. This is called glazing. Energy source is a chemical corrosive agent. An example is
By loading and glazing the grinding wheel operation chemical machining method (CHM).
become ineffective. Electro chemical energy methods involve electrolytic
(anodic) dissolution of the work piece material in contact
Dressing and Truing with a chemical solution. Energy source is high current and
Dressing and truing are operations done on grinding wheel electrolyte solution. Energy source is high current and elec-
to remove the ineffectiveness due to loading, glazing and trolyte solution. These methods include
wear. 1. Electro chemical machining (ECM)
Dressing is the process of sharpening abrasive elements 2. Electro chemical grinding (ECG)
using various dressing tools like star dressing tool, diamond 3. Electro chemical debarring (ECD)
dressing tool and round abrasive dressing sticks.
Truing is the process of changing the shape of the In thermoelectric machining, thermal energy is employed to
periphery of the grinding wheel when it becomes worn due melt and vapourise tiny particles of work material by con-
to breaking away of abrasive grains and bond. The wheel centrating the heat energy as a small area of the work piece.
periphery is made concentric with its axis of rotation. In The processes are
Chapter 4  •  Machining and Machine Tool Operations  |  3.843

1. Ion beam machining (IBM) Reservoir Electrode


Spark
2. Plasma arc machining (PAM)
3. Electrical discharge machining (EDM)
4. Electron beam machining (EBM)
5. Laser beam machining (LBM)

Water Jet Machining Process


In this process a very high velocity water jet of small cross
-sectional area is directed to the area to be cut. The very high Pump
Dielectric
kinetic energy of the jet is converted to pressure energy, Work piece fluid
which erodes the material at the small area of impingement. Variable
On movement of the jet crack is developed thus forming the register
cut. Water in the pressure range of 1500 to 4000 MPa is used.

Chemical Machining Process DC Power


Tool
In this method acids react with the metal to be cut are used.
Acids form compounds which are soluble in water. The
chemical reagent used is known as ‘etchant’ and the process Work piece
Capacitor
is also called etching. The areas from where the material is
not to be removed are protected by a chemically resistant EDM process can be used for machining hard and tough
material known as ‘maskant’. materials for manufacturing of dies for forging and extru-
sion, intricate mould cavities, carbide tools etc.
Electrochemical Machining Process The disadvantage of EDM process are that the metal-
In the electro chemical machining (ECM) a direct current lurgical properties of the material will change at very high
with high amperage and low voltage is passed between the temperatures, machining of complicated profiles, are not
work piece (anode) and a pre-shaped tool (cathode) through possible, and specific power consumption is very high.
electrolyte. At the work piece metal is dissolved in electro-
lyte as metallic ions. Areas where metal is not to be removed Economics of Machining
as masked by suitable maskants. The process is similar to The main aim of a machining operation is to produce qual-
electroplating. In ECM, the electrotype is so chosen such ity parts at minimum cost and maximum production rate.
that no electroplating takes place and the tool shape is Production costs and production rates depend upon the
maintained. If a close gap of 0.1 to 0.2 mm is maintained cutting parameters namely
between the tool and work, the machined surface takes the
replica of the tool shape. The electrolyte is forced to flow 1. Cutting speed
through the gap. The positive metallic ions from the work 2. Feed
reacts with the negative ions in solution, forming metallic 3. Depth of cut
hydroxides and other compounds known as ‘sludge’ and Experience gained over years has led to certain empirical
precipitate. rules or guiding principles for choosing the optimum cut-
ting condition.
Electric Discharge Machining (EDM) Production time is defined as the average time taken to
It is observed that a powerful spark, such as the terminals produce one component and production cost is defined as
of an automobile battery causes pitting or erosion of metal the average cost of performing the machining operation.
at both anode and cathode. This principle is used in EDM When cutting speed and feed are increased while other
which is also called spark erosion. If anode and cathode conditions held constant the machining time will be reduced,
are of same material, it is observed that more erosion takes the tool wear rate increases. It can be seen that at very slow
place at anode. So in EDM work is connected as anode. speeds and feeds the production time is increased due to the
In EDM the work piece and electrode (cathode) are gen- long machining time.
erally immersed in a moving insulating fluid called ‘dielec- Very high speeds and feeds also will result in a high pro-
tric fluid’. A suitable gap known as spark gap is maintained duction time due to the frequent need to change the cutting
between the electrode and work piece. The control of ero- tools. Therefore there will be an optimum condition at which
sion a metal is achieved by controlling the rapidly recurring production time will be minimum. Similarly there will be
spark impinging against the work piece surface. an optimum condition for minimum production cost also. A
As soon as the spark occurs high pressure is developed in manufacturing engineer should aim at reducing both produc-
the gap and temperature of the order of 10000°C is reached at tion time and production cost. In general, these objectives
the spot hit by the spark. Here metal gets melted and eroded. cannot be reached simultaneously so a compromise is sought.
3.844  |  Part III  •  Unit 7  •  Manufacturing Technology

Choice of feed depends upon type of operations whether Jigs and Fixtures
a roughing operation or finishing operation.
In mass production when articles are to be produced with
It is seen that equal changes in speed or feed affect the
high degree of accuracy some specially designed tooling
tool temperature by the same amount an increase in feed
is required. Jigs and fixtures come under such category of
will not affect the relative speed of sliding at the wearing
tools. Jigs and fixtures are work holding devices designed
surface of the tool, whereas the speed of sliding increases
and built to hold, support and locate every component to
with increase in cutting speed. Tool wear is a function of
ensure that each is drilled or machined within the specified
both temperature and relative speed of sliding. There fore an
limits. They are accurately made and the material used must
increase in cutting speed will result in a greater reduction of
be able to with stand wear and operational forces experi-
tool life than similar increases in feed. Therefore in a rough
enced during metal cutting.
machining for increased production rate feed is increased
Jig is a frame or body which holds and positions the work
rather than the cutting speed. But there is a limitation in
and guides the cutting tool during, the machining operation
increasing the feed. Because tool forces increase with the
such as drilling reaming, lapping etc.
increase in feed. The guiding principle in choosing opti-
Fixture is a production tool that locates, holds and sup-
mum conditions in a roughing operations is that the feed
port the work securely in a fixed orientation with respect to
should always be set maximum possible.
the tool such that the required machining operation can be
performed.
Economic Cutting Speed Jigs are connected with operation while fixtures are most
An increase in cutting speed has two main effects upon the commonly related to specific machine tools. Most common
economics of cutting jigs are drilling jigs, reaming jigs assembly jigs etc. They
1. The metal removal rate is increased are not fastened to a machine tool or tables, but are free to
2. Tool life is decreased be moved so as to permit proper resisting of work and the
tool. But fixtures are attached to some machine tool or table.
Due to the first the direct cost of metal removal is decreased. Examples are milling fixtures broaching fixtures, grinding
Reduction in tool life increases the cost of servicing and fixtures, assembly fixtures etc. Fixtures are heavier in con-
replacement of worn out tools. struction and are bolted rigidly on machine table.
As cutting speed ‘v’ increases cost of cutting falls.
1 Principles of Design of Jigs and Fixtures
Therefore cost of cutting α
υ
As v increases the tool life T falls. Cost of tooling Jigs and fixtures have the following components.
1
increases. Therefore cost of tooling α Location  This ensure that the work piece is given the
T desired constraint and determine the position of the work
From Taylor’s equation,
piece with respect to the cutting tool.
1
1 V  n Clamping  The clamps should be positioned to give best
= 
T C resistance to the cutting force. The clamp should be such
1 that they should not cause reformation of work piece.
Therefore cost of tooling αν n
Clearance  Enough clearance should be allowed for varia-
The above costs and total costs can be plotted against tion of work piece size, for operator’s hand etc.
cutting speed as shown. The ideal cutting speed is where the
total cost is minimum. Stability and Rigidity
Uneven seating should be avoided (provide four feets). The
1
Cost of cutting α equipment should be made sufficiently rigid to make it suit-
V able for the operation.
1 1
Cost of cutting+ Cost of tooling α +V
V n Handling
The equipment should be as light as possible and easy to
Cost of tooling α V 1 handle. Sharp corners should be avoided and lifting points
Costs n should be provided for heavy equipment.

General
Optimum cutting speed
The design should be simple to minimize the cost. Standard
Cutting speed V parts should be utilized as much as possible.
Chapter 4  •  Machining and Machine Tool Operations  |  3.845

Types of Jigs and Fixtures Z’ Y’

1. Used with machine tools, drilling fixtures, milling fix-


tures, broaching fixtures etc.
2. Devices for locating and clamping, arbors, holders etc. X’
3. Assembly fixtures
4. Inspection fixtures
5. Special jigs and fixtures

X
Degrees of Freedom
An unrestricted object in space is free to move in any of the
twelve possible directions and is said to have twelve degrees
Z
of freedom. An object is free to revolve around or move
parallel to any axis in either direction. y
Degree of freedom can be taken as six if we consider In order to locate the block correctly, with in a jig all
three freedom of translation and three freedoms of rotation these six movements must be restrained by arranging suit-
about the three axes irrespective of the directions. able locating points and then clamping the block in position.

Exercises

Practice Problems 1 Rake angle, a = 10°


Cutting speed = 100 m/min
1. While machining steel using HSS steel a tool life of 40 Coefficient of friction in the cutting is
min was observed with cutting speed of 100 m/min. Tool (A) 0.54 (B) 0.67
life for a cutting speed of 75 m/min, assuming value (C) 0.48 (D) 0.59
n = 0.12 in Taylor’s equation will be
6. A work piece 400 mm long and 50 mm diameter was
(A) 420 min (B) 440 min
reduced in diameter by turning. Cutting speed was
(C) 430 min (D) 450 min
30 m/min and feed was 0.5 mm/revolution. The time
2. In a cutting operation the cutting speed was reduced by required for a cut of length 300 mm is
20%. Assuming n = 0.5 and C = 350, increase in tool (A) 6.28 min (B) 3.54 min
life is (C) 3.14 min (D) 3.98 min
(A) 56% (B) 48%
(C) 59% (D) 46% 7.
50 40
Direction for questions 3 and 4: 30
60 50
A cutting tool when used for machining a work piece 40 mm
material at a cutting speed of 45 m/min, lasted for 120
A stepped shaft of the dimensions as per the sketch shown
min. Value of n = 0.26 in Taylor’s equation.
is to be made from a shaft of 50 mm diameter and 150
3. Life of the tool when cutting speed is increased by 25% mm long by turning. The depth of cut should not exceed
will be 2.5 mm. The cutting speed is to be 25 m/ min and feed to
(A) 60.5 min (B) 50.9 min be 0.3 mm per revolution. The required machining time is
(C) 46.2 min (D) 48.7 min (A) 4.97 min (B) 4.83 min
4. Cutting speed of the tool to get a tool life of 180 min (C) 3.86 min (D) 3.54 min
will be
(A) 45.2 m/min (B) 38.6 m/min Direction for questions 8 and 9:
(C) 40.5 m/min (D) 43.7 m/min A 150 mm long 12 mm dia SS rod is to be reduced
5. In an orthogonal cutting, the following observations to 10 mm dia in a single cut by turning. The spindle
were made. rotates at 500 rpm and tool speed is 200 mm/ min
Cutting force = 150 kgf 8. Cutting speed of the lathe in the operation is
Feed force = 50 kgf (A) 19.54 m/min (B) 16.76 m/min
Chip thickness ratio, r = 0.44 (C) 17.92 m/min (D) 18.85 m/min
3.846  |  Part III  •  Unit 7  •  Manufacturing Technology

9. The material removal rate in mm3/min is Machining process Metal removal method
(A) 7540 (B) 8630 (B) 9960 (D) 8190 (S)  LBM (4)  |melting and vaporization
10. A 170 mm long 60 mm diameter shaft is to be connected (5)  Ion displacement
to 166 mm long and 50 mm dia shaft by turning and fac-
(6)  Plastic shear and ion
ing operations. The work piece rotates at 450 rpm, feed displacement
is 0.3 mm per revolution and maximum depth of cut is
2 mm. Turning operation is performed first. Assuming (A) A-3, B-5, C-1, D-6
total approach and over travel distance of 5 mm for turn- (B) A-1, B-3, C-4, D-2
ing operation, the total machining time will be (C) A-5, B-1, C-2, D-4
(A) 3.92 min (B) 4.26 min (D) A-2, B-4, C-6, D-5
(C) 3.76 min (D) 4.33 min 15. In electro discharge machining, the tool is made of
11. 1000 Nos 8 mm diameter and 100 mm length pieces (A) Plain carbon steel (B) Copper
are to be produced from 10 mm diameter and 100 mm (C) Cast iron (D) High speed steel
length pieces. 16. In ECM, the material removal is due to
Cutting speed is 31.42 m/min and feed rate is 0.7 mm/ (A) Erosion (B) Corrosion
rev. Assuming n = 1.2 and C = 180 in Taylors’s expres- (C) Ion displacement (D) Fusion
sion, the number of times sharpening required for the 17. In EDM process, the work piece is connected to
cutting tool is (A) Anode (B) Cathode
(A) 44 (B) 22 (C) 33 (D) 55 (C) Earth (D) Any of these
12. A batch of ten cutting tools could produce 500 com- 18. A 500 mm × 42 mm flat surface of a plate is to be
ponents while working at 50 rpm with a tool feed of machined on a shaper, fixing the longer side in the
0.25 mm/rev and depth of cut of 1 mm. A similar batch direction of tool travel. If the tool over travel is 20 mm
of 10 tools of the same specifications could produce at each end, average cutting sped is 8m/min, feed rate is
122 components while working at 80 rpm with a feed 0.3 mm/stroke and ratio of return to cutting time is 1:2,
of 0.25 mm/rev and 1 mm depth of cut. How many the time required for machining will be
components can be produced with one cutting tool at (A) 12.3 min (B) 15.8 min
60 rpm? (C) 14.2 min (D) 13.6 min
(A) 29 (B) 31 (C) 37 (D) 42
19. 10 mm diameter through holes are to be drilled in a
13. During life testing of tool on a lathe under dry cutting
MS plate of 25 mm thickness. Drill point angle is 120°,
condition gave n and C of Taylor’s equations as 0.12 and
feed is 0.2 mm/revolution, and spindle speed is 300
130 m/min respectively. When a coolant was used C
rpm. Assuming a drill over travel of 2 mm, the time for
increased by 8%. Find the percentage increase in tool life
producing one hole will be
with the use of coolant at a cutting speed of 90 m/min
(A) 25 sec (B) 28 sec
(A) 85% (B) 90%
(C) 32 sec (D) 30 sec
(C) 88% (D) 97%
20. The tool signature of a cutting tool in ASA system is
14. Match correct pairs
5 – 5 – 7 – 7 – 8 – 15 – 0
Machining process Metal removal method In a cutting operation if the feed rate is f, the peak value
(P)  ECM (1)  Plastic shear height of surface produced is
(Q)  EDM (2)  Erosion/britt fracture (A) 0.135 f mm (B) 0.95 f mm
(R)  USM (3)  corrosive reactions (C) 0.236 f mm (D) 0.156 f mm

Practice Problem 2
1. In a machining operation when cutting speed was Direction for questions 2 and 3:
reduced to half tool life reached 8 times of the original
value. The exponent in Taylor’s tool life equation is In a machining experiment tool life varied with cutting
speed as follows.
1 1
(A) (B) Cutting speed Tool life
2 4 (m/min) (minutes)
1 60 80
(C) (D) 1 90 35
3 5
Chapter 4  •  Machining and Machine Tool Operations  |  3.847

2. Exponent n and constant C of the Taylor’s equation are 9. The percentage of total energy dissipated due to fric-
(A) 0.35, 540.2 (B) 0.38, 520.8 tion at the tool chip interface is
(C) 0.42, 515.9 (D) 0.49, 513.6 (A) 32% (B) 25% (C) 34% (D) 30%
3. Percentage increase in tool life when cutting speed is Direction for questions 10 and 12:
halved. Orthogonal turning is performed on a cylindrical work
(A) 306.4 (B) 311.5 piece with shear strength of 260 N/mm2. The following
(C) 313.7 (D) 315.6 data were observed.
4. In an orthogonal cutting test on steel the following con- Cutting velocity: 170 m/min
ditions were obtained. Feed: 0.2 mm/rev
Cutting speed = 45 m/min Depth of cut: 3 mm
Depth of cut = 0.3 mm Chip thickness of ratio: 0.5
Tool rake angle = +6° Orthogonal rake angle: 6°
Chip thickness = 1.5 mm Use Mechant’s theory:
Cutting force= 900 N 10. Shear plane angle and shear force are
Thrust force = 450 N (A) 28°, 336 N (B) 26°, 332 N
Using Merchant’s analysis, the friction angle during (C) 30°, 340 N (D) 24°, 328 N
the machining will be 11. Cutting and frictional forces respectively are
(A) 32.6° (B) 35.4° (A) 598 N, 475 N (B) 595 N, 471 N
(C) 30.8° (D) 28.9° (C) 578 N, 465 N (D) 582 N, 467 N
5. In orthogonal turning of a mild steel bar of diameter 12. In an orthogonal turning of carbon steel specific
150 mm with a carbide tool the cutting velocity was 80 machining energy is 2.5 J/mm3. Cutting velocity, fed
m/min. Feed was 0.25 mm/rev. Chip thickness obtained and depth of cut are 125 m/min; 0.2 mm per revolu-
was 0.5 mm. If the orthogonal rake angle is zero and tion and 2 mm respectively. The main cutting force in
principal cutting edge angle is 90°, the shear angle in Newton is
degrees is (A) 1000 N (B) 800 N
(A) 28.32 (B) 26.56 (C) 900 N (D) 1100 N
(C) 24.81 (D) 22.78
13. Taylor’s tool life exponent for tool A is 0.4 and for tool
6. In a single point turning tool, side rake angle and orthogo- B is 0.3. Constants of Taylor’s equation for A and B
nal rake angle are equal. The principal cutting edge angle are 90 and 60 respectively. Cutting speed in m/min
is between zero and 90°. The chip flows in the orthogonal above which tool A will have a higher tool life than tool
plane. The value of f is approximately equal to B is
(A) 0° (B) 45° (A) 15.6 m/min (B) 17.8 m/min
(C) 60° (D) 90° (C) 18.2 m/min (D) 14.4 m/ min
Direction for questions 7, 8 and 9:
14. Match the lists and select correct answer.
In an orthogonal machining operation, the following Machining process Associated medium
data were obtained.
P  USM 1.  Kerosene
Uncut thickness = 0.5 mm
Cutting speed = 18 m/min Q  EDM 2.  Abrasive slurry
Rake angle = 15° R  ECM 3.  Vacuum
Width of cut = 5 mm S  EBM 4.  Salt solution
Chip thickness = 0.7 mm (A) P-2, Q-3, R-4, S-1
Thrust force = 200 N (B) P-4, Q-1, R-2, S-3
Cutting force = 1200 N (C) P-2, Q-1, R-4, S-1
Assume Merchant’s theory (D) P-4, Q-3, R-2, S-1
7. Values of shear angle and shear strain respectively are 15. Holes in nylon button are made by
(A) 38.4°, 1.187 (B) 42.3°, 1.198 (A) EDM (B) CHM
(C) 40.2°, 1.1645 (D) 45.7°, 1.147 (C) USM (D) LBM
8. Coefficient of friction at the tool chip interface 16. In ECM, the material removal is due to
(A) 0.455 (B) 0.536 (A) Corosion (B) Erosion
(C) 0.392 (D) 0.477 (C) Fusion (D) Ion displacement
3.848  |  Part III  •  Unit 7  •  Manufacturing Technology

17. Two tools P and Q have tool signature (A) 4.26 min (B) 4.24 min
5- 5- 6- 6- 7- 20- 0 and (C) 3.88 min (D) 3.92 min
5- 5-7- 7- 8- 30- 0 respectively as per ASA system. 23. A hole is to be drilled in a metal block with a 10 mm
They are used under same machining conditions. Ratio drill at a feed of 0.2 mm/rev. Spindle speed is 800 rpm.
of peak to valley heights of the tools will be The material removal rate in mm3/min will be
(A) 0.7 (B) 0.8 (A) 12932 (B) 12356
(C) 0.9 (D) 1.1 (C) 12566 (D) 12289
18. In a single pass drilling operation a through hole of 16 24. 20 mm diameter through holes are to be drilled in
mm diameter is to be drilled in a 45 mm thick steel work pieces of 100 mm length and 50 mm diameter.
plate. Details of the drill used are Approach and over travel instance is 20 mm. Feed is 0.5
Spindle speed = 500 rpm mm/rev and rpm is 200. Assuming a set up time of 25%
Feed = 0.2 mm/rev of the processing time the number of pieces that can be
Drill point angle = 118° drilled in a shift of 8 hrs is
Clearance at approach and exit = 3 mm (A) 320 (B) 240
Time required to drill a hole will be (C) 360 (D) 280
(A) 33.5 sec (B) 35.1 sec
25. A 600 × 800 mm steel pieces is to surface machined on
(C) 30.2 sec (D) 31.3 sec
a shaping machine. Cutting speed is 8m/min. Return to
19. 3 numbers 600 mm × 30 mm flat pieces have to be fin- cutting time ratio is 1:4, feed is 2 mm/double strokes.
ish machined as a shaper, fixing 600 mm side along Clearance at each end is 75 mm, 600 mm is in tool
tool travel direction. Tool travel at each end of the plate travel direction. The machining time will be
is 20 mm. Average cutting speed is 8 m/min. (A) 36.2 min (B) 38.5 min
Feed ratio is 0.3 mm/stroke. Cutting to return time ratio (C) 42.7 min (D) 46.9 min
is 2:1. Set up time required per piece is 3 min. Total
26. A slot of 300 × 25 mm is to be milled in a work piece
time required for the 3 pieces will be
of 300 mm length with a side and face milling cutter
(A) 50 min (B) 45 min
of 100 mm diameter, 25 mm wide and having 18 teeth.
(C) 48 min (D) 40 min
The depth of cut is 5 mm. feed per tooth is 0.1 mm and
cutting speed is 30 m/min. Assuming an approach and
20. 50 45 30 mm over travel distance of 50 mm the required time to mill
40 the slot will be
50
60 (A) 1.8 min (B) 2.2 min
A stepped shaft as per dimensions given is to be made by (C) 2.4 min (D) 1.9 min
turning. A 150 mm long 50 mm diameter shaft is avail- 27. If under a given condition of plain turning, the life of
able. Estimate the time required to make the shaft. The cutting tool decreases by 50%, due to an increase of
depth of cut should not exceed 2.5 mm. Cutting speed is velocity by 25%. The index of Taylor equation will be
to be 25 m/min and feed to be 0.3 mm per revolution. (A) 0.252 (B) 0.244
(A) 3.89 min (B) 4.26 min (C) 0.264 (D) 0.268
(C) 3.38 min (D) 4.73 min 28. The tool life of a single point cutting tool has been
21. A 200 mm long, 20 mm diameter ms. rod is to be found to be 1500 sec at a cutting speed of 0.5 m/s. How
reduced to 18 mm diameter, in a single cut by turning. many pieces can be produced with in one tool life, if
Cutting speed is 20 m/min feed rate is 200 mm/min each piece is 50 mm in diameter and 80 mm long, using
rpm and material removal rate in mm3/min are a feed of 0.1 mm/rev and a cutting speed od 0.5 m/s
(A) 302 rpm, 12322 mm3/min, (approach and overshoot can be neglected)
(B) 328 rpm, 12722 mm3/min (A) 5 (B) 7
(C) 326 rpm, 12820 mm3/min (C) 4 (D) 3
(D) 318 rpm, 12566 mm3/min
29. Choose the best set of operation–process combinations
22. A work piece 170 mm long and 60 mm diameter is to
be machined to 166 mm lonf and 50 mm diameter by Operation Process
facing and turning. The work piece rotates at 450 rpm, P  Deburring (internal surface) 1.  Plasma arc machining
feed is 0.3 mm/rev and maximum depth of cut is 2 mm. Q  Die sinking 2.  Abrasive flow machining
Facing operation is performed first. Assuming total R  Fine hole drilling in thin 3.  Electric discharge
approach and over travel distance of 5 mm for turning, sheets machining
the total machining time will be
Chapter 4  •  Machining and Machine Tool Operations  |  3.849

Operation Process 30. In abrasive jet machining, as the distance between


S  Tool sharpening 4.  Ultra sonic machining
the nozzle tip and work surface increases the material
removal rate
5.  Laser beam machining
(A) Increases continuously
6.  Electrochemical grinding (B) Decrease continuously
(A) P-1, Q-5, R-3, S-4 (C) Decreases, becomes stable and then increases
(B) P-1, Q-4, R-1, S-2 (D) Increases, becomes stable and then decreases
(C) R-5, Q-1, R-2, S-6sf
(D) P-2, Q-3, R-5, S-6

Previous Years’ Questions


1. The mechanism of material removal in EDM process 5. In a machining operation, doubling the cutting speed
th
is [2004]
reduces the tool life to 1 of the original value. The
(A) Melting and evaporation 8
(B) Melting and corrosion exponent n in Taylor’s tool life equation VT n = C is
(C) Erosion and cavitation  [2004]
(D) Cavitation and evaporation 1 1
(A) (B)
2. A standard machine tool and an automatic machine 8 4
tool are being compared for the production of a com- 1 1
(C) (D)
ponent. Following data refers to the two machines. 3 2
 [2004] 6. Typical machining operations are to be performed
Standard Automatic on hand-to-machine materials by using the processes
machine tools machine tool listed below. Choose the best set of operation–process
Setup time 30 min 2 hours combinations [2004]
Machining Operation Process
time per 22 min 5 min
piece P.  Deburring 1.  Plasma arc machining
(internal surface)
Machine rate 200 per hour 800 per hour
Q.  Die sinking 2.  Abrasive flow machining

The breakeven production batch size above which the R.  Fine hole drilling in thin 3.  Electric discharge
automatic machine tool will be economical to use will be sheets machining
(A) 4 (B) 5 S.  Tool sharpening 4.  Ultrasonic machining
(C) 24 (D) 225
3. Through holes of 10 mm diameter are to be drilled in 5.  Laser beam machining
a steel plate of 20 mm thickness. Drill spindle speed
6.  Electrochemical grinding
is 300 rpm, feed 0.2 mm/rev and drill point angle is
120°. Assuming drill over travel of 2 mm, the time for (A) P-1, Q-5, R-3, S-4
producing a hole will be [2004] (B) P-1, Q-4, R-1, S-2
(A) 4 sec (B) 25 sec (C) P-5, Q-1, R-2, S-6
(C) 100 sec (D) 110 sec (D) P-2, Q-3, R-5, S-6
4. In an orthogonal cutting test on mild steel, the follow- 7. A zigzag cavity in a block of high strength alloy is to
ing data were obtained:  [2004] be finish machined. This can be carried out by using
Cutting speed:  40 m/min  [2005]
Depth of cut:  0.3 mm
Tool rake angle:  + 5°
Chip thickness:  1.5 mm
Cutting force:  900 N
Thrust force :  450 N
Using Merchant’s analysis, the friction angle during
the machining will be
(A) 26.6° (B) 31.5°
(C) 45° (D) 63.4°
3.850  |  Part III  •  Unit 7  •  Manufacturing Technology

(A) Electric discharge machining 12. If each abrasive grain is viewed as a cutting tool, then
(B) Electro-chemical machining which of the following represents the cutting param-
(C) Laser beam machining eters in common grinding operations? [2006]
(D) Abrasive flow machining (A) Large negative rake angle, low shear angle and
8. When 3-2-1 principle in used to support and locate a high cutting speed
three dimensional work-piece during machining, the (B) Large positive rake angle, low shear angle and
number of degrees of freedom that are restricted is high cutting speed
[2005] (C) Large negative rake angle, high shear angle and
(A) 7 (B) 8 low cutting speed
(C) 9 (D) 10 (D) Zero rake angle, high shear angle and high cut-
ting speed
9. The figure below show a graph which qualitatively
relates cutting speed and cost per piece produced. 13. Arrange the processes in the increasing order of their
 [2005] maximum material removal rate.
Total cost Electrochemical machining (ECM)
Ultrasonic machining (USM)
Cost of per piece

Electron beam machining (EBM)


Laser beam machining (LBM) and
3
2
Electric discharge machining (EDM) [2006]
(A) USM, LBM, EBM, EDM, ECM
1 (B) EBM, LBM, USM, ECM, EDM
(C) LBM, EBM, USM, ECM, EDM
(D) LBM, EBM, USM, EDM, ECM
Cutting speed
Direction for questions 14 to 16:
The three curves 1, 2 and 3 respectively represent
In an orthogonal machining operation:
(A) Machining cost, non-productive cost, tool chang-
Uncut thickness = 0.5 mm
ing cost
Width of cut = 5 mm
(B)  Non-productive cost, machining cost, tool
Thrust force = 200 N
changing cost
(C) Tool changing cost, machining cost, non-produc- Rake angle = 15°
tive cost Cutting speed = 20 m/min
(D)  Tool changing cost, non-productive cost, ma- Chip thickness = 0.7 m
chining cost Cutting force = 1200 N
Assume Merchant’s theory
10. A 600 mm × 30 mm flat surface of a plate is to be
finish machined on a shaper. The plate has been fixed 14. The values of shear angle and shear strain, respec-
with the 600 mm side along the tool travel direc- tively, are [2006]
tion. If the tool over-travel each end of the plate is (A) 30.3° and 1.98 (B) 30.3° and 4.23
20 mm, average cutting speed is 8 m/min, feed rate is (C) 40.2° and 2.97 (D) 40.2° and 1.65
0.3 mm/stroke and the ratio of return time to cutting 15. The coefficient of friction at the tool-chip interface is
time of the tool is 1:2, the time required for machin-  [2006]
ing will be [2005] (A) 0.23 (B) 0.46
(A) 8 min (B) 12 min (C) 0.85 (D) 0.95
(C) 16 min (D) 20 min 16. The percentage of total energy dissipated due to fric-
11. Two tools P and Q have signatures 5°-5°–6°-6°- tion at the tool-chip interface is: [2006]
8°-30°-0 and 5°-5°-7°-7°-8°-15°-0 (both ASA) (A) 30% (B) 42%
respectively. They are used to turn components under (C) 58% (D) 70%
the same machining conditions. If hP and hQ denote 17. In orthogonal turning of a low carbon steel bar of
the peak-to-valley heights of surfaces produced by the diameter 150 mm with uncoated carbide tool, the
h cutting velocity is 90 m/min. The feed is 0.24 mm/
tools P and Q, the ratio P will be [2005]
hQ rev and the depth of cut is 2 mm. The chip thickness
tan 8° + cot 15° tan 15° + cot 8° obtained is 0.48 mm. If the orthogonal rake angle is
(A) (B) zero and the principal cutting edge angle is 90o, the
tan 8° + cot 30° tan 30° + cot 8°
shear angle in degree is  [2007]
tan 15° + cot 7° tan 7° + cot 15°
(C) (D) (A) 20.56 (B) 26.56
tan 30° + cot 7° tan 7° + cot 30° (C) 30.56 (D) 36.56
Chapter 4  •  Machining and Machine Tool Operations  |  3.851

18. In electrodischarge machining (EDM), if the thermal (A) 5 (B) 10


conductivity of tool is high and the specific heat of (C) 15 (D) 20
work piece is low, then the tool wear rate and material 24. Internal gear cutting operation can be performed by
removal rate are expected to be respectively. [2007]  [2008]
(A) High and high (B) Low and low (A) Milling
(C) High and low (D) Low and high (B) Shaping with rack cutter
19. In orthogonal turning of medium carbon steel, the spe- (C) Shaping with pinion cutter
cific machining energy is 2.0 J/mm3. The cutting veloc- (D) Hobbing
ity, feed and depth of cut are 120 m/min, 0.2 mm/rev 25. In a single point turning tool, the side rake angle and
and 2 mm respectively. The main cutting force in N is orthogonal rake angle are equal. φ is the principal cut-
 [2007] ting edge angle and its range is 0° ≤ φ ≤ 90°. The chip
(A) 40 (B) 80 flows in the orthogonal plane. The value of φ is closest
(C) 400 (D) 800 to  [2008]
20. In orthogonal turning of low carbon steel pipe with (A) 0o (B) 45o
(C) 60o (D) 90o
principal cutting edge angle of 90o, the main cut-
ting force is 1000 N and the feed force is 800 N. 26. A researcher conducts electrochemical machining
The shear angle is 25o and orthogonal rake angle is (ECM) on a binary alloy (density 6000 kg/m3) of iron
zero. Employing Merchant’s theory, the ratio of fric- (atomic weight 56, valency 2) and metal P (atomic
tion force to normal force acting on the cutting tool is weight 24, valency 4). Faraday’s constant = 96500
 [2007] coulomb/mole. Volumetric material removal rate of
(A) 1.56 (B) 1.25 the alloy is 50 mm3/s at a current of 2000 A. The per-
(C) 0.80 (D) 0.64 centage of the metal P in the alloy is closest to[2008]
21. Match the most suitable manufacturing processes for (A) 40 (B) 25
the following parts. [2007] (C) 15 (D) 79
Spindle
Manufacturing Work piece
Parts Motor UV
processes P Q R S
(P) Computer chip (1) Electrochemical machining Cutting cool
Half-
(Q) Metal forming dies (2) Ultrasonic machining E
Nut
Lead screw
and molds
(R) Turbine blade (3) Electrodischarge machining 27. The figure shows an incomplete schematic of a con-
(S) Glass (4) Photochemical machining ventional lathe to be used for cutting threads with, dif-
ferent pitches. The speed gear box Uv is shown and the
(A) P-4, Q-3, R-1, S-2 feed gear box Us is to be placed P, Q, R and S denote
(B) P-4, Q-3, R-2, S-1 locations and have no other significance. Changes in
(C) P-3, Q-1, R-4, S-2 UV should NOT affect the pitch of the thread being
(D) P-1, Q-2, R-4, S-3 cut and changes in Uv should NOT affect the cutting
speed.
Direction for questions 22 and 23: The correct connections and the correct placement of
A low carbon steel bar of 147 mm diameter with a length Us are given by  [2008]
of 630 mm is being turned with uncoated carbide insert. (A)  Q and E are connected. Us is placed between P
The observed tool lives are 24 min and 12 min for cutting and Q
velocities of 90 m/min and 120 m/min respectively. The (B)  S and E are connected. Us is placed between R
feed and depth of cut are 0.2 mm/rev and 2 mm respec- and S
tively. Use the unmachined diameter to calculate the cut- (C)  Q and E are connected. Us is placed between Q
ting velocity. and E
22. When tool life is 20 min, the cutting velocity in m/ (D)  S and E are connected. Us is placed between S
min is [2007] and E
(A) 87 (B) 97 28. A displacement sensor (a dial indicator) measures
(C) 107 (D) 114 the lateral displacement of a mandrel mounted on
23. Neglect over-travel or approach of the tool. When tool the taper hole inside a drill spindle. The mandrel axis
life is 20 min, the machining time in min for a single is an extension of the drill spindle taper hole axis
pass is [2007] and the protruding portion of the mandrel surface
3.852  |  Part III  •  Unit 7  •  Manufacturing Technology

is perfectly cylindrical. Measurements are taken (A) 0.0 (B) 0.5


with the sensor placed at two positions P and Q as (C) 1.0 (D) 2.0
shown in the figure. The readings are recorded as 33. Electrochemical machining is performed to remove
RX = maximum deflection minus minimum deflec- material from an iron surface of 20 mm × 20 mm
tion, corresponding to senor position at X, over one under the following conditions:  [2009]
rotation. [2008]
Inter electrode gap = 0.2 mm
Supply voltage (DC) = 12 V
Specific resistance of
Drill electrolyte = 2 W cm
spindle
P Atomic weight of iron = 55.85
Mandrel Sensor Valency of iron = 2
Q
Faraday’s constant = 96540 Coulombs
The material removal rate (in g/s) is
Drill table (A) 0.3471 (B) 3.471
(C) 34.71 (D) 347.1

Direction for ­questions 34 and 35:


If RP = RQ > 0, which one of the following would be In a machining experiment, tool life was found to vary
consistent with the observation? with the cutting speed in the following manner:
(A) The drill spindle rotational axis is coincident Cutting speed (m/min) Tool life (min)
with the drill spindle taper hole axis 60 81
(B) The drill spindle rotational axis intersects the
90 36
drill spindle taper hole axis at point P
(C) The drill spindle rotational axis is parallel to the
drill spindle taper hole axis 34. The exponent (n) and constant (k) of the Taylor’s tool
(D) The drill spindle rotational axis intersects the life equation are [2009]
drill spindle taper hole axis at point Q (A) n = 0.5 and k = 540
(B) n = 1 and k = 4860
Direction for questions 29 and 30: (C) n = -1 and k = 0.74
Orthogonal turning is performed on a cylindrical work (D) n = -0.5 and k = 1.155
piece with shear strength of 250 MPa. The following con-
ditions are used: cutting velocity is 180 m/min, feed is 35. What is the percentage increase in tool life when the
0.20 mm/rev, depth of cut is 3 mm, chip thickness ratio cutting speed is halved? [2009]
= 0.5. The orthogonal rake angle is 7°. Apply Merchant’s (A) 50%
theory for analysis. (B) 200%
(C) 300%
29. The shear plane angle (in degrees) and the shear force (D) 400%
respectively are [2008]
(A) 52; 320 N (B) 52; 400 N 36. For tool A, Taylor’s tool life exponent (n) is 0.45 and
(C) 28; 400 N (D) 28; 320 N constant (K) is 90. Similarly for tool B, n = 0.3 and
K = 60. The cutting speed (in m/min) above which tool
30. The cutting and frictional forces, respectively, are A will have a higher tool life than tool B is [2010]
 [2008] (A) 26.7
(A) 568 N; 387 N (B) 565 N; 381 N (B) 42.5
(C) 440 N; 342 N (D) 480 N; 356 N (C) 80.7
31. Friction at the tool-chip interface can be reduced by (D) 142.9
 [2009]
(A) Decreasing the rake angle Direction for ­questions 37 and 38:
(B) Increasing the depth of cut In a shear cutting operation, a sheet of 5 mm thickness is
(C) Decreasing the cutting speed cut along a length of 200 mm. The cutting blade is 400 mm
(D) Increasing the cutting speed long and zero-shear (S = 0) is provided on the edge. The
ultimate shear strength of the sheet is 100 MPa and pen-
32. Minimum shear strain in orthogonal turning with a etration to thickness ratio is 0.2.
cutting tool of zero rake angle is [2009]
Chapter 4  •  Machining and Machine Tool Operations  |  3.853

Neglect friction 42. A CNC vertical milling machine has to cut a straight
400 slot of 10 mm width and 2 mm depth by a cutter of 10
mm diameter between points (0, 0) and (100, 100) on
the XY plane (dimension in mm). The feed rate used
for milling is 50 mm/min. Milling time for the slot (in
seconds) is [2012]
(A) 120 (B) 170
(C) 180 (D) 240
43. Details pertaining to an orthogonal metal cutting pro-
S
cess are given below. [2012]
Chip thickness ratio = 0.4
37. Assuming force vs displacement curve to be rectan- Undeformed thickness = 0.6 mm
gular, the work done (in J) is [2010] Rake angle = +10o
(A) 100 (B) 200 Cutting speed = 2.5 m/s
(C) 250 (D) 300 Mean thickness of primary
38. A shear of 20 mm (S = 20 mm) is now provided on shear zone = 25 mm
the blade. Assuming force vs displacement curve to The shear strain rate in s-1 during the process is
be trapezoidal, the maximum force (in kN) exerted is (A) 0.1781 × 105 (B) 0.7754 × 105
 [2010] (C) 1.0104 × 10 5 (D) 4.397 × 105
(A) 5 (B) 10 44. In a single pass drilling operation, a through hole of
(C) 20 (D) 40 15 mm diameter is to be drilled in a steel plate of 50
39. A single-point cutting tool with 12° rake angle is used mm thickness. Drill spindle speed is 500 rpm, feed is
to machine a steel work-piece. The depth of cut, i.e., 0.2 mm/rev and drill point angle is 118°. Assuming 2
uncut thickness is 0.81 mm. The chip thickness under mm clearance at approach and exit, the total drill time
orthogonal machining condition is 1.8 mm. The shear (in seconds) is [2012]
angle is approximately. (A) 35.1 (B) 32.4
 [2011] (C) 31.2 (D) 30.1
(A) 22° (B) 26° 45. A steel bar 200 mm in diameter is turned at a feed
(C) 56° (D) 76° of 0.25 mm/rev with a depth of cut of 4 mm. The
40. Match the following non-traditional machining pro- ­rotational speed of the work piece is 160 rpm. The
cesses with the corresponding material removal material removal rate in mm3/s is [2013]
mechanisms: [2011] (A) 160 (B) 167.6
Mechanism (C) 1600 (D) 1675.5
Machining
of material 46. During the electrochemical machining (ECM) of iron
process
removal
(atomic weight = 56, valency = 2) at current of 1000
(P) Chemical machining (1) Erosion A with 90% current efficiency, the material removal
(Q) Electro-chemical rate was observed to be 0.26 gm/s. If titanium (atomic
machining (2) Corrosive reaction weight = 48, valency = 3) is machined by the ECM
(R) Electro-discharge process at the current of 2000 A with 90% current
machining (3) Ion displacement efficiency the expected material removal rate in gm/s
(S) (4) Fusion and will be [2013]
Ultrasonic machining
vaporization (A) 0.11 (B) 0.23
(C) 0.30 (D) 0.52
(A) P-2, Q-3, R-4, S-1
47. Two cutting tools are being compared for a machining
(B) P-2, Q-4, R-3, S-1
operation. The tool life equations are
(C) P-3, Q-2, R-4, S-1
Carbide tool: VT1.6 = 3000
(D) P-2, Q-3, R-1, S-4
HSS tool: VT0.6 = 200
41. In abrasive jet machining, as the distance between the where V is the cutting speed in m/min and T is the
nozzle tip and the work surface increases, the material tool life in min. The carbide tool will provide higher
removal rate [2012] tool life if the cutting speed in m/min exceeds
(A) Increases continuously  [2013]
(B) Decreases continuously (A) 15.0 (B) 39.4
(C) Decreases, becomes stable and then increases (C) 49.3 (D) 60.0
(D) Increases, becomes stable and then decreases
3.854  |  Part III  •  Unit 7  •  Manufacturing Technology

Direction for questions 48 and 49: (B) Plasma arc machining


In orthogonal turning of a bar 100 mm diameter with a (C) Laser beam machining
feed of 0.25 mm/rev, depth of cut of 4 mm and cutting (D) Electro discharge machining
velocity of 90 m/min, it is observed that the main (tangen- 55. A hole of 20 mm diameter is to be drilled in a steel
tial) cutting force is perpendicular to the friction force act- block of 40 mm thickness. The drilling is performed
ing at the chip-tool interface. The main (tangential) cutting at rotational speed of 400 rpm and feed rate of 0.1
force is 1500 N. mm/rev. The required approach and over run of the
48. The orthogonal rake angle of the cutting tool in degree drill together is equal to the radius of drill. The drill-
is [2013] ing time (in minute) is  [2014]
(A) zero (B) 3.58 (A) 1.00 (B) 1.25
(C) 5 (D) 7.16 (C) 1.50 (D) 1.75
49. The normal force acting at the chip-tool interface in N 56. The process utilizing mainly thermal energy for
is  [2013] removing material is  [2014]
(A) 1000 (B) 1500 (A) Ultrasonic machining
(C) 2000 (D) 2500 (B) Electrochemical machining
(C) Abrasive jet machining
50. The main cutting force acting on a tool during the
(D) Laser beam machining
turning (orthogonal cutting) operation of a metal is
400 N. The turning was performed using 2 mm depth 57. A straight turning operation is carried out using a
of cut and 0.1 mm/rev feed rate. The specific cutting single point cutting tool on an AISI 1020 steel rod.
pressure (in N/mm2) is [2014] The feed is 0.2 mm/rev and the depth of cut is 0.5
(A) 1000 (B) 2000 mm. The tool has a side cutting edge angle of 60°. The
(C) 3000 (D) 4000 uncut chip thickness (in mm) is _______ [2014]
51. During pure orthogonal turning operation of a hollow 58. Cutting tool is much harder than the workpiece. Yet
cylindrical pipe, it is found that the thickness of the the tool wears out during the tool-work interaction,
chip produced is 0.5 mm. The feed given to the zero because [2014]
degree rake angle tool is 0.2 mm/rev. The shear strain (A) Extra hardness is imparted to the workpiece due
produced during the operation is ____ [2014] to coolant used
(B) Oxide layers on the workpiece surface impart ex-
52. If the Taylor’s tool life exponent n is 0.2, and the tool tra hardness to it
changing time is 1.5 min, then the tool life (in min) (C) Extra hardness is imparted to the workpiece due
for maximum production rate is ____ to severe rate of strain
 [2014] (D) Vibration is induced in the machine tool
53. Match the machine tools (Group A) with the probable 59. Which pair of following statements is correct for
operations (Group B): [2014] orthogonal cutting using a single-point cutting tool?
 [2014]
Group A Group B
P.  Reduction in friction angle increases cutting
(P) Centre Lathe (1) Slotting
force
(Q) Milling (2) Counter-boring Q.  Reduction in friction angle decreases cutting
(R) Grinding (3) Knurling force
(S) Drilling (4) Dressing
R.  Reduction in friction angle increases chip
thickness
(A) P-1, Q-2, R-4, S-3 S.  Reduction in friction angle decreases chip
(B) P-2, Q-1, R-4, S-3 thickness
(C) P-3, Q-1, R-4, S-2 (A) P and R (B) P and S
(D) P-3, Q-4, R-2, S-1 (C) Q and R (D) Q and S
60. Two separate slab milling operations, 1 and 2, are
54. The following four unconventional machining pro-
performed with identical milling cutters. The depth
cesses are available in a shop floor. The most appro-
of cut in operation 2 is twice that in operation 1. The
priate one to drill a hole of square cross section of
other cutting parameters are identical. The ratio of
6 mm × 6 mm and 25 mm deep is  [2014]
maximum uncut chip thickness in operations 1 and 2
(A) Abrasive jet machining
is ____ [2014]
Chapter 4  •  Machining and Machine Tool Operations  |  3.855

61. The principle of material removal in electrochemical the average power input required is 1 kW. The capaci-
machining is[2014] tance (in mF) in the circuit is:[2015]
(A) Fick’s law (B) Faraday’s laws (A) 2.5 (B) 5.0
(C) Kirchoff’s laws (D) Ohm’s law (C) 7.5 (D) 10.0
62. Better surface finish is obtained with a large rake 70. In a machining operation, if the generatrix and direc-
angle because [2014] trix both are straight lines, the surface obtained is:
(A) The area of shear plane decreases resulting in the [2015]
decrease in shear force and cutting force (A) cylindrical
(B) The tool becomes thinner and the cutting force is (B) helical
reduced (C) plane
(C) Less heat is accumulated in the cutting zone (D) surface of revolution
(D) The friction between the chip and the tool is less 71. Orthogonal turning of a mild steel tube with a tool of
63. A cast iron block of 200 mm length is being shaped rake angle 10° is carried out at a feed of 0.1 mm/rev.
in a shaping machine with a depth of cut of 4 mm, If the thickness of the chip produced is 0.28 mm, the
feed of 0.25 mm/stroke and the tool principle cut- values of shear angle and shear strain will be respec-
ting edge angle of 30°. Number of cutting strokes tively.[2015]
per minute is 60. Using specific energy for cutting as (A) 28° 20′ and 2.19 (B) 22° 20′ and 3.53
1.49 J/mm3, the average power consumption (in watt) (C) 24° 30′ and 4.19 (D) 37° 20′ and 5.19
is ____ [2014] 72. A shaft of length 90 mm has a tapered portion of length
64. Under certain cutting conditions, doubling the cutting 55 mm. The diameter of the taper is 80 mm at one
th end and 65 mm at the other. If the taper is made by
⎛ 1⎞
speed reduces the tool life to ⎜ ⎟ of the original. tailstock set over method, the taper angle and the set
⎝ 16 ⎠
over respectively are:[2015]
Taylor’s tool life index (n) for this tool-work piece (A) 15° 32′ and 12.16 mm
combination will be ______.[2015] (B) 18° 32′ and 15.66 mm
65. In a linear arc welding process, the heat input per unit (C) 11° 22′ and 10.26 mm
length is inversely proportional to:[2015] (D) 10° 32′ and 14.46 mm
(A) Welding current 73. The non-traditional machining process that essen-
(B) Welding voltage tially required vacuum is:[2016]
(C) Welding speed (A) electron beam machining
(D) Duty cycle of the power source (B) electro chemical machining
66. An orthogonal turning operation is carried out under (C) electro chemical discharge machining
the following conditions: rake angle = 5º, spindle (D) electro discharge machining
rotational speed = 400 rpm; axial feed = 0.4 m/min 74. In an orthogonal cutting process the tool used has
and radial depth of cut = 5 mm. The chip thickness, rake angle of zero degree. The measured cutting force
tc, is found to be 3 mm. The shear angle (in degrees) and thrust force are 500 N and 250 N, respectively.
in this turning process is ____.[2015] The coefficient of friction between the tool and the
67. The primary mechanism of material removal in elec- chip is _______.[2016]
tro chemical machining (ECM) is:[2015] 75. The tool life equation for HSS tool is VT 0.14 f 0.7 d 0.4 =
(A) Chemical corrosion constant. The tool life (T) of 30 min is obtained using
(B) Etching the following cutting conditions:
(C) Ionic dissolution V = 45 m/min, f = 0.35 mm, d = 2.0 mm
(D) Spark erosion If speed (V), feed (  f  ) and depth of cut (d) are
68. A single point cutting tool with 0° rake angle is used increased individually by 25%, the tool life (in min)
in an orthogonal machining process. At a cutting is:[2016]
speed of 180 m/min, the thrust force is 490 N. If the (A) 0.15 (B) 1.06
coefficient of friction between the tool and the chip (C) 22.50 (D) 30.0
is 0.74, then the power consumption (in kW) for the 76. The following data is applicable for a turning operation.
machining operation is _______.[2015] The length of job is 900 mm, diameter of job is 200 mm,
69. A resistance–capacitance relaxation circuit is used in feed rate is 0.25 mm/rev and optimum cutting speed is
an electrical discharge machining process. The dis- 300 m/min. The machining time (in min) is ______.
charge voltage is 100 V. At spark cycle time of 25 µs, [2016]
3.856  |  Part III  •  Unit 7  •  Manufacturing Technology

77. In an ultrasonic machining (USM) process, the mate- 80. In a wire–cut EDM process the necessary conditions
rial removal rate (MRR) is plotted as a function of that have to be met for making a successful cut are
the feed force of the USM tool. With increasing feed that:[2016]
force, the MRR exhibits the following behavior: (A) wire and sample are electrically non-conducting
[2016] (B) wire and sample are electrically conducting
(A) Increases linearly (C)  wire is electrically conducting and sample is
(B) Decreases linearly electrically non-conducting
(C) Does not change (D)  sample is electrically conducting and wire is
(D) First increases and then decreases electrically non-conducting
78. For a certain job, the cost of metal cutting in `18C/V 81. Internal gears are manufactured by[2016]
and the cost of tooling is `2701C/(TV), where C is a (A) hobbing
constant, V is the cutting speed in m/min and T is the (B) shaping with pinion cutter
tool life in minutes. The Taylor’s tool life equation is (C) shaping with rack cutter
VT 0.25 = 150. The cutting speed (in m/min) for the (D) milling
minimum total cost is ________.[2016] 82. For an orthogonal cutting operation, tool material
79. The surface irregularities of electrodes used in an is HSS, rake angle is 22o, chip thickness is 0.8 mm,
electrochemical machining (ECM) process are 3 µm speed is 48 m/min and feed is 0.4 mm/rev. The shear
and 6 µm as shown in the figure. If the work-piece plane angle (in degrees) is[2016]
is of pure iron and 12 V DC is applied between the (A) 19.24
electrodes, the largest feed rate is ________ mm/min. (B) 29.70
Conductivity of the electrolyte 0.02 ohm–1 mm–1
(C) 56.00
Over-potential voltage 1.5 V
(D) 68.75
Density of iron 7860 kg/m3 83. In a single turning operation with cemented carbide
Atomic weight of iron 55.85 gm
tool and steel work piece, it is found that the Taylor’s
exponent is 0.25. If the cutting speed is reduced by
Assume the iron to be dissolved as Fe+2 and the 50% then the tool life changes by _______ times.
Faraday constant to be 96500 Coulomb.[2016] [2016]
Tool

6 µm
Nominal gap
3 µm

Work-piece (Iron)
Chapter 4  •  Machining and Machine Tool Operations  |  3.857

Answer Keys
Exercises
Practice Problems 1
1.  B 2.  A 3.  B 4.  C 5.  A 6.  C 7.  B 8.  D 9.  A 10.  B 
11.  C 12.  A 13.  B 14.  C 15.  B 16.  C 17.  A 18.  C 19.  D 20.  A 

Practice Problems 2
1.  C 2.  D 3.  B 4.  A 5.  B 6.  B 7.  C 8.  A 9.  D 10.  A 
11.  B 12.  A 13.  B 14.  C 15.  D 16.  D 17.  C 18.  A 19.  B 20.  A 
21.  D 22.  B 23.  C 24.  A 25.  D 26.  B 27.  C 28.  A 29.  D 30.  D

Previous Year’s Questions


1.  A 2.  D 3.  B 4.  B 5.  C 6.  D 7.  A 8.  C 9.  A 10.  B
11.  B 12.  A 13.  D 14.  D 15.  B 16.  A 17.  B 18.  D 19.  D 20.  C
21.  A 22.  B 23.  C 24.  C 25.  A 26.  B 27.  C 28.  C 29.  D 30.  B
31.  D 32.  D 33.  A 34.  A 35.  C 36.  A 37.  A 38.  B 39.  B 40.  A
41.  D 42.  B 43.  C 44.  A 45.  D 46.  C 47.  B 48.  A 49.  B 50.  B
51.  2.8 to 3.0 52.  5.9 to 6.1 53.  C 54.  D 55.  B 56.  D 57.  0.08 to 0.12
58.  C 59.  D 60.  0.70 to 0.72 61.  B 62.  A 63.  295 to 305
64.  0.25 65.  C 66.  18.5 to 19 67.  C 68.  2.0 to 2.2 69.  B 70.  C 71.  A
72.  A 73.  A 74.  0.5 75.  B 76.  7.5 to 7.6 77.  D 78.  57.8 to 58
79.  51 to 52 80.  B 81.  B 82.  B 83.  16
Chapter 5
Metrology and Inspection
Computer Integrated
Manufacturing
chapter highlights

☞☞ Combinational Logic Design ☞☞ Code Converter


☞☞ Arithmetic Circuit ☞☞ Decoder
☞☞ Half Adder ☞☞ Designing High Order Decoders from Lower
☞☞ Full Adder Order Decoder
☞☞ Half Subtractor ☞☞ Combinational Logic Implementation
☞☞ Full Subtractor ☞☞ Encoders
☞☞ n-bit Comparator ☞☞ Multiplexer
☞☞ Parity Bit Generator and Parity Bit Checker ☞☞ Demultiplexer

Limits, Fits and Tolerances without causing any functionable trouble, when assembled
with its mating part and put into actual service.
An article manufactured consists of assembly of a number
of components. Thus a component manufactured should
Tolerance
be matching with some other mating component. This is
Upper limit
important for the proper functioning and prolonged life of
the product. Lower limit
In the production of a component, it is not possible to
make any part precisely to a given dimension, due to vari-
ability of elements of production processes. If attempts
are made to achieve the perfect size, the cost of produc-
tion will increase tremendously. But for practical purposes,
perfect fitting of the mating components are not necessary.
Slight dimensional variations are acceptable for the proper Systems of  Writing Tolerances
functioning.
The allowable variation in the basic size required in pro- 1. Unilateral system
duction is called tolerance. 2. Bilateral system
Larger and smaller dimensions allowable are called lim-
its– the high limit and low limit. In unilateral system dimension of a part is allowed to vary
Thus difference between high and low limits is the toler- only on one side of the basic size i.e., tolerance zone is
ance. It is the margin allowed for variation in workmanship. either above or below the basic size line.
Tolerance can also be defined as the amount by which
the job is allowed to go away from accuracy and perfectness
Chapter 5  •  Metrology and Inspection Computer Integrated Manufacturing  |  3.859

Tolerance of metal 24.98 mm will be minimum or least metal limit


(LML) as shaft at this limit will have the least possible
amount of metal

Hole LML
LML
Basic size Tolerance MML
MML
Shaft

Unilateral tolerance

Terminology for Limits and Fits


With a 25 mm of basic size examples of unilateral toler- Shaft: It is the external dimension of a component (need
ance are not be a circular shaft)
+ 0.02 + 0.01 Hole: It is the internal dimension of a component (need not
25+ 0.01 , 25− 0.00 , be a circular hole)
etc
+ 0.00 − 0.02
25− 0.02 , 25− 0.03 Basic or Nominal Size
It is the standard size of a part with reference to which limits
The first case means upper limit is 25.02 mm and lower of dimensions are determined. Basic size is same for hole
limit is 25.01 mm and shaft. In design calculations basic size is used.
\ Tolerance = 25.02 – 25.01 Actual sizes is the measured size of a manufactured part.
= 0.01 mm Zero line: It is the horizontal line representing the basic
In the fourth case, size.
Upper limit = 24.98 Deviation is the algebraic difference between the size
Lower limit = 24.97 (maximum, minimum, actual etc) with the basic size.
Tolerance  = 24.98 – 24.97 Deviations are shown with respect to the zero line or datum
= 0.01 mm line.
In bilateral system, the dimension of the part is allowed to Upper deviation is a positive quantity when maximum
vary on both sides of the basic size. limit of size is greater than the basic size. It is negative when
maximum limit of size is less than the basic size. For hole it
Tolerance
denoted by ES and for shaft it is es
25 ±0.02
Lower deviation is a positive quantity when the lower
+0.01
−0.02 limit of size is greater than the basic size and negative when
25
it is less than the basic size. It is denoted by EI and ei for
hole and shaft respectively.
Basic size Tolerance is represented by IT which stands for interna-
tional tolerance grade.

Relationship of  Tolerance and Deviations


+ 0.01 For shaft, IT = es – ei
25± 0.02 25− 0.02 etc are examples of bilateral tolerance. (upper deviation – lower deviation)
In the first case, For hole, IT = ES – EI
Upper limit = 25.02 mm
Fundamental Deviation
Lower limit = 24.98 mm
Tolerance  = 25.02 – 24.98 It is the deviation (either upper or lower) nearest to the basic
= 0.04 mm line. It fixes the position of the tolerance zone in relation to
the zero line.
Maximum or Minimum Metal Limits Basic shaft is the shaft whose upper deviation is zero.
Basic size and maximum size is same in this case. It is
If the tolerance for the shaft is given as 25± 0.02 the shaft is
denoted by ‘h’.
said to have maximum metal limit (MML) of 25.02 mm.
Basic hole is the hole whose lower deviation is zero. Its
Since at this limit, the shaft has maximum possible amount
low limit is same as the basic size. It is denoted by letter ‘h’.
3.860  |  Part III  •  Unit 7  •  Manufacturing Technology

Tolerance In Interference fit, the minimum permissible diameter


zones of the shaft is larger than the maximum allowable diam-
eter of the hole. After fitting the members are permanently
Zero line Zero line attached.

Transition Fit
Transition fit is in between clearance fit and interference fit.
Depending upon the actual sizes of the parts clearance or
interference may occur.
Basic hole Basic shaft
Zero line Hole tolerance

Tolerance grade is an indication of degree of accuracy and


is represented by letters IT followed by a number. For exam-
ple, IT0, IT01, IT1 etc
Shaft
tolerance
Standard Tolerance Unit
(a) (b) (c)
It serves as a basis for determining standard tolerance (IT). Clearance fit Transition fit Interference fit
It is denoted by ‘i’ and expressed in microns.
Hole Basis System for Three Types of Fits
Fits
Figure above gives hole basis system where the fundamen-
When two parts are to be assembled, the relation resulting tal deviation of hole is zero (For shaft basis system funda-
from the difference between their sizes before assembly is mental deviation of shaft is zero)
called a fit. It is the degree of tightness or looseness between Hole basis system is generally used because it is easier to
two mating parts. vary the size of the shaft and to measure it than to vary the
hole size, provided the work is not very large.
Types of Fits
Shaft basis system is useful when a number of acces-
Depending on the basis of positive, zero or negative clear- sories such as bearings, collars etc are to be fitted on same
ance values between mating parts fits are classified as under shaft.

Fits Allowance
Allowance is the intentional difference between the lower
Clearance Transition Interference limit of hole and higher limit of the shaft.
fit fit fit It is the variation given for the purpose of providing dif-
ferent classes of fit. It is the difference between maximum
material size limits of mating parts.
(a)  Slide fit (a)  Push fit (a)  Force fit The allowance may be positive or negative. Positive
(b)  easy slide fit (b)  Wringing fit (b)  Tight fit allowance is the minimum clearance and negative allow-
(c)  Running fit (c)  Shrink fit ance is the maximum interference intended between the
mating parts.
Clearance Fit
Clearance is the difference between sizes of hole and shaft. Allowance vs Tolerance
Minimum clearance is the difference between minimum Allowance is the prescribed difference between the dimen-
size of hole and maximum size of shaft. sions of two mating parts (hole and shaft). Tolerance is the
Maximum clearance is the difference between maximum permissible variation in dimension of a part (either hole or
size of hole and minimum size of the shaft. a shaft)
In clearance fits, the largest permissible shaft diameter
is smaller than the diameter of the smallest hole. So the Standard Limit Systems
shaft can rotate or slide in the hole with different degrees The aim of a standard limit system is
of freedom.
1. To select basic functional clearances and interferences
Interference Fit for a given application or type of fit
Interference is the arithmetical difference between sizes of 2. to establish tolerances which will provide a reasonable
hole and shaft, when shaft is of bigger size. and economical balance fits, consistency and cost
Chapter 5  •  Metrology and Inspection Computer Integrated Manufacturing  |  3.861

British standard, international standard (ISO) and Indian Designation of Holes, Shafts and Fits
standards are some of the standard limit systems A shaft or a hole is completely described if the basic size
followed by the appropriate letter and the tolerance grade
Indian Standard System of Limits and Fits is given.
(IS-919 and 2709) For example,
Indian standards are in line with ISO recommendations. The 50 H6 means 50 mm H hole with tolerance grade IT6
standards cover holes and shafts from the smallest size to 50 f7 means 50 mm f shaft with tolerance grade IT7
3150 mm. For any size over this range there is a wide choice
of fits available and for each of the fits there is a series of
Tolerance and Fundamental Deviation
tolerance grades from very fine to wide tolerances.
Standard tolerance and fundamental deviations are used for Larger Sizes
in the limit system. The finer tolerance grades IT01 to IT5 are not provided for
18 grades of fundamental tolerances are used. These are sizes above 500 mm.
designated as IT01, IT0, IT1 to IT16 For size above 500 mm and up to 3150 mm
Fundamental deviations are indicated by 25 letters A to IS : 2101 specify various grades
ZC for holes and letters a to zC for shafts. (For holes: A, B, C, I (in m) = 0.004D + 2.1
D, E, F, G, H, JS, J, K, M, N, P, R, S, T, U, V, X, Y, Z, ZA, ZB,
ZC. For shafts corresponding small letters are used) Tolerance Analysis in Manufacturing
For A to H holes lower deviation is above zero line and and Assembly
for J to ZC it is below the zero line. Tolerance can be defined as the amount of variation permis-
For shafts a to h upper deviation is below the zero line sible from accuracy and perfectness of the dimension of a
and for j to zc it is above the zero line. component without causing any functional trouble. Human
Standard tolerances are expressed in terms of standard failure and machine limitations prevent the achievement of
tolerance unit, i. It is given by ideal dimensions of the part during fabrication.
By providing tolerance the cost of production can be
i = 0.45 3 D + 0.001D microns reduced; without sacrificing the functional requirement.
The difference between upper limit and lower limit of a
Where D is the geometric mean upper and lower values of
dimension is called the tolerance zone. It is the margin for
a diameter step in which the diameter lies. IS–919 specifies
variation in workmanship. Selection of tolerance is based
the following diameter steps.
on the following.
1–3, 3–6, 6–10, 10–14, 14–18, 18–24, 24–30, 30–40, 40–50,
etc upto 180–200 mm. 1. Functional requirement : With the permitted tolerance,
Values of tolerances for tolerance grades IT5 to IT16 are assembly should be possible and the equipment
obtained from the following table. should be able to perform the required function.
2. Standardisation: Standardisation of the parts are
required for interchangeability which is essential for
Grade IT5 IT6 IT7 IT8 IT9 mass production.
3. Manufacturing needs
Value 7i 10i 16i 25i 40i When the functional requirement is not so rigid tolerance
choice may be influenced and determined by factors like
methods of tooling, equipment available etc.

Need for Tolerance


IT 10

IT 11

IT 12

IT 13

IT 14

IT 15

IT 16

1. Different materials have different properties. Variation


in properties cause errors during machining.
1000i

2. Production machines may have inherent inaccuracies


100i

160i

250i

400i

640i
64i

and have limitation to produce parts with perfect


dimensions.
3. Machine operators have limitation to make perfect
Other tolerances– settings. There are chances of errors in setting up
For IT01 : 0.3 + 0.08 D machines, adjusting tools and work piece on the
For IT0 : 0.5 + 0.12 D machine etc.
For IT1 : 0.8 + 0.02 D
Aiming at ideal conditions of dimensions will result in
For IT2 to IT4 values are approximated between IT1 and
exhorbitant costs. If the components are made with large
IT5
3.862  |  Part III  •  Unit 7  •  Manufacturing Technology

tolerances without affecting the functional requirement, different craftsmen or operators in different batches in dif-
cost of production can be reduced. So the tolerance selected ferent machines. In this case the parts are produced within
should be just enough to the required job and not better. specified tolerance limits so that a component selected at
So it can be said that tolerance is a compromise between random will assemble correctly with any other mating com-
accuracy required for proper functioning and ability for ponent, selected at random. By this method manufacturing
economic production of this accuracy. cost is considerably reduced.

Solved Examples Selective Assembly


Example 1:  What is meant by 20 H 7 f8 in Indian standard When high degree of accuracy is required in mass produc-
limit system? tion, interchangeable assembly also become not economi-
Solution:  It means a 20 mm basic size H-hole with toler- cal. In this case selective assembly method can be used. In
ance grade IT7 is fitted with a f-shaft of tolerance grade IT8. selective assembly parts are produced with wider tolerances
so that the cost is reduced. The components produced are
0.050
classified into groups according to their sizes by automatic
Example 2:  A hole is specified as 400.000 mm. The mating
gauging. Assemblies can be made only with matched sets of
shaft has a clearance fit with minimum clearance 0.02 mm.
parts such that the clearances are within limits
The tolerance on the shaft is 0.04 mm. Maximum clearance
An example for selective assembly is the assembly of
between hole and shaft is
pistons with cylinder bores.
Solution:  Low limit of hole: 40.000 mm If the basic size is 50 mm for bore and 49.88 mm for
   High limit of hole: 40.050 mm piston and required clearance is 0.12 mm, these can be pro-
duced with a wide tolerance of 0.04 mm.
40.050
40.000 Hole Then dimension of bore = 50±0.02
Dimension of piston = 49.88±0.02
In interchangeable assembly the clearance can go upto
39.980 50.02 – 49.86 = 1.16 mm which is not acceptable.
Shaft But the components are grouped in match sets as follows
39.940
the required clearance of 0.12 mm can be achieved.

 High limit of shaft  = 40.000 – 0.02
      = 39.980 mm
Cylinder bore 49.98 50.00 50.02

 Low limit of shaft    = 39.980 – 0.04
      = 39.940 mm Piston 49.86 49.88 49.90
 Maximum clearance = 40.050 – 39.940

      = 0.110 mm
Gauges
Types of Assemblies In mass production where a large number of similar compo-
In assembly of components the mating parts can be made to nents are produced, inspection by measuring each compo-
fit in three ways. These are nent will be time consuming and not economical. Therefore
1. By trial and error the conformance of each part is checked with the tolerance
2. Interchangeable assembly specification using tools called gauges.
3. Selective assembly Gauges are inspection tools of rigid design without a
scale, which serve to check the dimensions of manufactured
Assembly by Trial and Error parts. They check only whether the inspected parts are made
When the assemblies required are only small in number and within the specified limits. Plain gauges are used for check-
they are similar, these can be produced by trial and error ing plain (unthreaded) holes and shafts.
by same craftsman. In this method one part is made to its Classification––
nominal size as accurately as possible and the mating part is
made to suit it by trial and error. 1. According to type
 (a)  standard gauges
Interchangeable Assembly  (b)  limit gauges
2. According to purpose
In mass production the quantities required are very large and
 (a)  Workshop
it is not economical to produce it by trial and error by same
 (b) Inspection
person. In mass production the components are produced by
 (c)  reference or master gauges
Chapter 5  •  Metrology and Inspection Computer Integrated Manufacturing  |  3.863

3. According to form of tested surface Gauge Maker’s Tolerance or Gauge


 (a)  plug gauge Tolerance
 (b)  snap and ring gauges
Gauges cannot be manufactured to the exact sizes. If closer
4. According to their design
limits are held, the gauge become expensive. Therefore
 (a)  Single limit and double limit
some allowance is given in the manufacturing of gauges
 (b)  Single ended and double ended
This is known as gauge tolerance.
 (c)  Fixed and adjustable gauges
Limit gauges are usually provided with a tolerance of
Standard gauge is an exact of copy of their mating part. 10% of the work tolerance. For inspection gauges it is 5%
For example a shaft and a bush. The shaft can be inspected of work tolerance.
with a gauge which is an exact copy of the bush.
Limit gauges are used to check whether the parts are within Gauge tolerance
the specified limits. HL NO HL GO
Limit plug gauges are used for checking holes. The GO plug GO
gauge has the size of low limit of the hole while NO GO plug
Work
gauge has the size of high limit of the hole. If the inspected tolerance
hole is within their tolerance limits the GO gauge will enter the
hole while the NO GO gauge cannot enter the hole. NO
GO LL
Plug gauges can be plain or screwed. GO
LL
Plain plug gauges are generally double ended type for Plug gauge Snap gauge
sizes up to 63 mm and single ended for sizes above 63 mm
size. Wear Allowance
Ring gauges are used to check shafts. The measuring surfaces of GO gauges which constantly
Plain ring gauges are made of suitable wear resisting rub against the surface of parts in inspection are conse-
steel and gauging surfaces are hardened. GO and NO GO quently subjected to wear and lose their initial size, while
type ring gauges are available. the size of snap GO gauges are increased. Therefore a wear
Snap and gap gauges are also used to check shafts. The allowance is added in a direction opposite to the wear. Wear
jaws of the gauge is made use of for checking. Double ended allowance is taken as 10% of the gauge tolerance. Wear
type plate snap gauges are used for sizes in the range of 2 to allowance is applied to a normal GO gauge diameter before
100 mm and single ended progressive type in the range of gauge tolerance is applied. As per British standards when
100 to 250 mm. In adjustable type gap gauges, the gauging the work tolerance is more than 0.09 mm, wear allowance
anvils are adjustable end wise in the horse shoe frame. is provided.
These gauges are set by means of slip gauges to any par- Inspection gauges are used by inspectors for final
ticular limit required. inspection of the manufactured parts. These have slightly
larger tolerance than work shop gauges. This is to ensure
Combined limit gauges are used for gauging of cylindrical
that work which passes the work shop gauges will be
holes. In these a single gauge is used to check both upper
accepted by inspection gauges. The tolerance on inspection
and lower limits.
gauges fall outside the work tolerance.
Contour gauges are employed for checking the dimen-
sional accuracy and shapes of irregular work. Radius gauge
is an example. Taylor’s Principle for The Design
Taper plug gauge is used for checking tapered holes and
of Limit Gauge
taper ring gauge is used for checking shafts. These check 1. GO gauges should inspect all the features of a
the diameter at the bigger end and the change in diameter component at a time and should be able to control
per unit length. the maximum metal limit, or in other words
Slip gauges or gauge blocks are used as standards of the maximum metal limit of as many related
measurement. It is the universally accepted end standard dimensions as possible should be incorporated in
of length. These are rectangular blocks of high grade steel the GO gauge.
with exceptionally close tolerances. Faces are round and 2. NOT GO gauge should check only one element at a
lapped. These are available in standard sets. The blocks are time for the minimum metal limit.
placed one over other by the phenomenon of wringing to get Example 3: What are the upper and lower limits of the
a specified length. Slip gauges are also made from Tungsten shaft represented by 60f 8. Diameter 60 mm lies in diameter
carbide which is extremely hard and wear resistant. They step of 50 –80 mm. Fundamental deviation of f shaft is –5.5
are carefully finished by high grade lapping to a high degree D0.41 mm.
of finish flatness and accuracy.
3.864  |  Part III  •  Unit 7  •  Manufacturing Technology

Example 6:  For the following hole and shaft assembly find
Solution:  Geometric mean diameter = 50 × 80
shaft tolerance, hole tolerance and state the type of fit.
 = 63.246 mm
+0.05 +0.05
Fundamental tolerance unit
Hole: 60 +0.002 shaft: 60 +0.005
i = 0.45 3 D + 0.001D Solution:  HL of hole: 60.025 mm
3
= 0.45 63.246 + 0.001 × 63.246 LL of hole: 60.00

Hole tolerance : 60.025 – 60.000 = 0.025 mm
= 1.859 mm
HL of shaft : 60.05
= 0.00186 mm
LL of shaft : 60.005
IT8 = 25i
Shaft tolerance = 60.05 – 60.005 = 0.045 mm
   = 25 × 0.00185
Allowance = LL of hole – HL of shaft
   = 0.04646 mm
= 60.00 – 60.05
Fundamental deviation of f shaft
= –0.05 mm (interference)
 = –5.5 D0.41
HL of hole – LL of shaft = 60.025 – 60.005
 = –5. 5 [63.246]0.41 mm
= 0.025 mm (clearance)
 = –0.03012 mm
\ The type of fit is transition fit
Upper limit of shaft = 60 – 0.03012
 = 59.97 mm Example 7:  In a limit system, the following limits are
Lower limit of shaft = 59.97 – 0.0465 specified to give a clearance fit between shaft and hole
= 59.924 mm. −0.006

Example 4:  Find the values of allowance and tolerances Shaft: 40 −0.020 mm
for hole and shaft assembly for the following mating parts +0.30
−0.03 +0.25 Hole: 40 −0.000 mm
Hole: 25−0.05 shaft: 60 +0.00
Determine–
Solution:
(A) basic size
(a) Hole
(B) shaft and hole tolerances
Tolerance = 25.04 – 25.00
(C) Minimum clearance
= 0.04 mm
(D) Maximum clearance
(b) Shaft
Tolerance = HL – LL Solution:
HL = 25.00 – 0.03 (a) Basic size : 40 mm
= 24.97 (b) Shaft tolerance:
LL = 25.00 – 0.05 (40 – 0.006) – (40 – 0.02)
= 24.95 = (–0.006) – (–0.020) = 0.014 mm
Tolerance = 24.97 – 24.95 Hole tolerance: (+0.030) – (0.00)
= 0.02 mm = 0.030 mm
(c) Allowance = LL of hole – HL of shaft (c) Minimum clearance = LL of hole – HL of shaft
= 25.00 – 24.97 = 50 – (50 – 0.006)
= 0.03 mm. = 0.006 mm
(d) Maximum clearance
Example 5:  A 60 mm diameter shaft is made to rotate in
HL of hole – LL of shaft
the bush. Tolerance for both bush and shaft are 0.050 mm.
= (50 + 0.030) – (50 – 0.020)
Determine dimension shaft and bush to give a minimum
= 0.050 mm
clearance 0.075 mm with hole basis system
In an assembly of two parts with 50 mm nominal diameter,
Solution: the lower deviation of the hole is zero and the higher is 5
microns. For the shaft lower and higher deviations are –8
0.05 Zero line and –4 microns respectively. Determine the allowance and
0.75
type of fit of the assembly.
0.05 Hole

0.005
Zero line
0.004
0.008
HL of hole = LL + tolerance = 60 + 0.050 = 60.050 mm
50.000
HL of shaft = LL of hole – Allowance
= 60.00 – 0.075 = 59.925 Shaft
LL of shaft = 59.925 – 0.050 = 59.875
Chapter 5  •  Metrology and Inspection Computer Integrated Manufacturing  |  3.865

HL of hole = 50 + 0.005 IT7 = 16 × i = 20.918 microns


= 50.005 = 0.021 mm
LL of hole = 50.000 IT8 = 25 × i = 32.69 microns
HL of shaft = 50.000 – 0.004 = 49.996 mm = 0.033 mm
LL of shaft: 50.000 – 0.008 = 49.992 mm Upper deviation of shaft
Allowance = LL of hole – HL of shaft = –5.5 × (23.24)0.41
= 50.000 – 49.996 = –20 microns
= 0.004 mm = –0.020 mm
(Clearance fit) Fundamental or lower deviation of hole = 0
+0.021
Example 8:  Between two mating parts of 90 mm basic size
\ Limits for 20 H7 = 20 +0.000
the actual interference fit is to be from 0.05 mm to 0.12 mm.
Limits for shaft f8
For shaft and hole tolerance is same. Determine the shaft
HL for shaft = 20 – 0.020
size and hole size in hole basis unilateral system.
LL for shaft = 20 – 0.02 – 0.033 = 20 – 0.053
Shaft HL of hole = 20.021 NO GO

0.12 mm 0.05 mm

Zero line
Hole
90 mm Gauge tolerance
Wear allowance
GO
Solution:
LL of hole = 90.00 mm LL of hole = 20.00
HL of shaft = 90.000 + 0.12
= 90.12 mm −0.020

Tolerance of shaft = Tolerance of hole \ Shaft limits are 20 −0.053

=
(0.12 − 0.05) Tolerance for plug gauge for gauging the hole = 10% of
work tolerance = 0.021 × 0.1 = 0.0021 mm
2
Wear allowance = 10% of gauge tolerance
= 0.035 mm = 0.1 × 0.0021
\ HL of hole = 90 + 0.035 = 90.035 mm = 0.0002 mm
LL of shaft = 90.035 + 0.05 = 90.085 mm Upper limit of GO plug gauge = 20 + 0.0002 + 0.0021
+0.12 = 20.0023 mm
\ Size of shaft = 90 +0.085
Low limit of GO gauge = 20 + 0.0002 = 20.0002
+0.035 +0.0023
Size of hole = = 90 +0.000 Limits of GO gauge = 20 +0.0002
Example 9:  Design the general GO and NO GO gauge for
hole of the assembly 20 H7 f 8 fit. The following data can Linear and Angular Measurements
be used
For linear measurements various standards followed are
(a) i = 0.001 D + 0.45 3 D microns 1. Line standard
(b) Upper deviation of f shaft 2. End standard
= −5.5 D
0.41
3. Wavelength standard
(c) Diameter step for 20 mm When length is measured between two engraved lines it is
= 18 –30 called line standard.
(d) IT7 = 16i When length is expressed as the distance between two
(e) IT8 = 25i flat parallel faces, it is called end standard. In wavelength
(f) Wear allowance = 10% of gauge tolerance standard, wavelength of mono chromatic light is used as
Solution: unit of length.
Imperial standard yard (England) and international
D = 18 × 30 = 23.24 mm standard meter (France) are line standards.
i = 0.001 × 23.24 + 0.45 3 23.24 Metre in wavelength standard is defined as 1650763.73
= 1.3074 microns wavelengths of orange radiation in vacuum of krypton 86
isotope.
3.866  |  Part III  •  Unit 7  •  Manufacturing Technology

Any system of measurement must be related to a known Straight Edge


standard for maintaining uniformity of measurement Straight edges are used for checking straightness and flat-
throughout the world. ness of work pieces with the help of spirit levels. Types of
Standards can be classified as international standard, straight edges are
national standard, national reference standard working
standard, Laboratory reference standard, Laboratory work- 1. Tool maker’s straight edge
ing standard and shop floor standard. 2. Wide edged straight
3. Angle straight edge
4. Box straight edge
Linear Measurements
Linear measurement means measurement of lengths diam- Spirit Level
eters, heights, thicknesses etc. It can be external or internal.
Spirit levels are used for measuring small angles or inclina-
For linear measurements the instruments used are, non-
tions, to determine position of a surface relative to horizon-
precision instruments, precision instruments, comparators,
tal position and for establishing horizontal datum.
measuring machines etc.
In spirit level a sealed glass tube is mounted on a base. A
Non-precision instruments are steel rule, calipers, divid-
scale is engraved on top of the glass tube. The tube contains
ers, telescopic gauges, depth gauges etc
ether or alcohol with a bubble. For measuring flatness or
Precision instruments are micrometers, vernier calipers,
angle the position of the bubble is made use of
height gauges, slip gauges etc.
Linear measuring instruments can also be classified as
Frame Level
1. Direct measuring instruments
Frame level is used for checking vertical surfaces. In frame
2. Indirect measuring instruments
levels base and side edge is exactly at 90°. A glass tube with
Direct measuring instruments can be graduated or non ether and bubble is fixed on the base. Side edge of the level
graduated. is kept in contact with the vertical edge. Position of the bub-
Rules, vernier calipers, micrometers, dial indications etc ble in the glass tube is noted for checking the verticality.
are examples of graduated instruments.
Callipers, trammels, straight edges, wire gauges, screw Vernier Instruments
pitch gauges, radius gauges, thickness gauges, slip gauges
Vernier instruments consists of a main scale and vernier
etc are examples of non-graduated instruments.
scale. The vernier scale can be moved on the main scale.
Callipers The difference between value of vernier scale division and
a main scale division forms the basis for precision meas-
Callipers are used when direct linear measurement is not urements. Vernier callipers and vernier height gauges are
possible. Callipers consist of two legs, hinged at top. The examples of vernier instruments. Least count of a vernier
leg ends are used for measurement. For example measure- instrument is the difference in value of a vernier scale divi-
ment of diameter of a rod. Value of the measurement is sion and a main scale division. For example if the value of
obtained with the help of a rule. The hinge joint can be firm one main scale division is 0.5 mm and 24 main scale divi-
joint or spring loaded. sions coincide with 25 vernier scale divisions,
Outside Calliper, inside calliper, Transfer calliper and
hermophrodite calliper are examples of callipers. ⎡ 24 ⎤
Least count = ⎢1 − ⎥ × 0.5
Hermophrodite calliper is also called odd leg calliper. This ⎣ 25 ⎦
is a scribing tool having one leg bent and the other leg equipped 1
with a scriber. This is used for scribing distances from the edge = × 0.5
25
of a work piece. The curved leg has a notch at its end.
1
= mm
Surface Plate 50
Surface plate is a flat plate on which measurement of parts = 0.02 mm.
are carried out. They are used in workshops and metrologi- Main scale reading + coinciding division on the vernier
cal laboratories. Surface plate form a datum surface for test- scale × least count gives the measurement value.
ing flatness of surfaces.
Micrometers (Screw Gauges)
V-block Micrometers are used when more accuracy in measure-
V-blocks are used in workshops as a support for round ment is required. Least count of micrometers is in the order
shaped work pieces. The work pieces are placed on this of 0.01 mm where as that of a vernier caliper is 0.02 mm.
V-shaped groove on top of the block. The angle of the Micrometer works on the principle of a screw and nut. For
groove is generally 90°. V-block is used for checking round- one revolution of the screw in the nut the axial distance
ness of rods, marking centres accurately etc. moved by it is equal to the pitch of the screw.
Chapter 5  •  Metrology and Inspection Computer Integrated Manufacturing  |  3.867

Micrometer consists of a U-shaped frame, spindle, bar- Sine bars are graded as A grade and B grade. A grade sine-
rel and thimble. By rotating the thimble on the barrel the bars are made with an accuracy of 0.01 mm/m of length
spindle advances. The object to be measured is placed in and B grade sine bars with an accuracy of 0.02 mm/m of
between spindle end and an anvil at the end of the frame. length
Main scale is marked on the barrel. The thimble has equal
divisions around its periphery, usually 50 nos. Therefore for Clinometers
rotation through one division of the thimble, axial distance Clinometer is a spirit level mounted as a rotary member car-
1 ried in a housing. One face of the housing forms the base
moved is × pitch of the screw.
50 of the instrument. There is a circular scale on the housing.
If pitch of the screw corresponds to one division on the Clinometers can be used to measure the included angle
1 between two adjacent slanted sides of a work piece.
main scale and if it is 0.5 mm, the least count is 0.5 × Different types are vernier clinometers, micrometer cli-
= 0.01 mm. 50
nometers and dial clinometers.
Measurement value using a micrometer is obtained simi-
lar to that of a vernier calliper i.e Main scale reading + read- Comparators
ing on the thimble × least count.
A comparator is a precision instrument employed to com-
A comparator is a precision instrument used for compar-
pare the dimension of a given component with a standard,
ing the dimension of a given component with a standard
such as slip gauges. It does not measure the actual dimen-
such as slip gauges. It does not measure the exact dimen-
sion but indicates how much it differs from the basic speci-
sion, but indicates the difference from the basic dimension.
men. The indicated difference is usually small and suitable
Dial gauge is the simplest form of mechanical comparator.
magnification device is provided.
Apart from mechanical comparators, mechanical optical
comparators, electrical and electronic comparators, pneu- Optimeters or Optical Comparators
matic comparators etc are available.
In these comparators the fundamental optical law is made
Measuring machines are generally used for measurement
use of If a ray of light falls on a mirror and is reflected and
of length over the outer faces of a length–bar or any other
the mirror is titled by an angle a, the reflected light moves
long member. These are generally used for measurement of
through an angle 2a. In optimeters the mirror is titled by
considerably greater dimension.
a measuring plunger movement and the movement of the
Angular Measurements reflected light is recorded as an image on a screen. The
shadow of the object is projected on to a curved graduated
Various instruments used for angular measurements are, scale to indicate in comparison measurement. Optical com-
bevel protractors, angle gauges, sinebars, clinometers, auto parators which make use of the enlarged image principle
collimators etc. are commonly known as optical projectors or optical pro-
Angle Gauges jection comparators. Optical projector is used for checking
the shape or profile of relatively small engineering com-
Angle gauges are taper pieces of hardened and stabilized
ponents with an accurate standard or drawing. It enables a
steel. The measuring pieces are lapped and polished to a
magnified image of part of a component to be projected on
high degree of accuracy and flatness. These pieces can be
to a screen where it is compared with an enlarged profile
wrung together just like slip gauges. The pieces are 75 mm
drawing. The degree of magnification may range from five
long and 16 mm wide. Angle gauges are generally available
to hundred.
in 2 sets of 12 and 13 numbers and a square block. In the 13
piece set the following angles are available – Tool Maker’s Microscope
1°, 3°, 9°, 27°, 41° This is used for measurement of small and delicate parts.
1′, 3′, 9′, 27′ It is used for complex measurement of profile of exter-
3″, 6″, 18″ , and 30″ nal threads as well as tools, templates and gauges, centre
In a 12 piece set all the above except 3″ is available. to center distance of holes in a plane etc. and for accurate
Selecting and placing the gauge pieces one over the other angular measurements.
different angles can be set for the measurement
Interferometry
Sine Bars
The advantage and peculiar property of monochromatic
Sine bar is a precision instrument used along with slip light is that its wave length has precise value until the pri-
gauges for the measurement of angles. Their general uses are mary colours which have ill defined wavelengths and the
1. To measure angles very accurately monochromatic light such as from mercury 198 or krypton
2. To locate a work to a given angle within very close 86 are exactly reproducible.
limits
3.868  |  Part III  •  Unit 7  •  Manufacturing Technology

For interference to occur, two conditions are necessary


Optical flat
1. Light rays are obtained by division from a single
source
2. Rays before being combined at the eye must travel h
paths whose lengths differ by an odd number of half
wavelengths. C G
L
For interference of light waves, the light should be from
coherent sources. i.e., the two sources should continuously x mm
have light rays of same wavelength and equal intensity
and maintain same phase differences. Further two sources The component C and the standard gauge block G are
should be very narrow and close to each other. placed on a reference surface at a known distance x. The
optical surface is placed over them as shown. The optical
Optical Flat flat will be inclined at a small angle producing an air wedge.
Optical flat is a circular piece of optical glass or fused quartz If N number of dark fringes are observed over the width L
having its two plain faces flat and parallel and the surfaces mm of the block G,
are finished to an optical degree of flatness. The difference in height
Optical flats are used to test flatness of lapped surfaces
such gauge blocks, gauges, micrometer anvils etc. When an λ xN
optical piece placed on a work piece surface, it will not be per- h= ⋅
2 L
fectly parallel to the surface, but will be at slight inclination
to the surface, forming an air wedge between the surfaces. Where l = wavelength
When the optical flat is illuminated by monochromatic If x = 50 mm
source of light interference fringes will be obtained. These L = 25 mm
are produced by the interference of light rays reflected from N=5
bottom face of optical flat and the top piece of the work λ
piece. If they are in phase, it will result brightness. If the rays = 0.000333909 mm
2
are out of phase, it will cause dark fringes. Each adjacent
fringe represent a change in elevation by half wavelength. h = 0.00333909 mm
There are four possible cases, when the contact between Interferometers are optical instruments used for measuring
optical flat and work surface is at one point only flatness and determining the length of slip gauges. These are
1. If the surfaces are perfectly wrung together no air gap based on interference principle and employs wavelengths of
exist and no fringe pattern will be observed. light as measuring units. Disadvantages of a optical flat are
2. If angle is increased, fringes are brought closer overcome in this by providing refined arrangement
together
3. If angle is decreased, fringes spacing increases. Screw Thread Measurement
4. When angle is made too large, then fringes will be Measurement of effective diameter.
closely spaced, as to be undistinguishable and no Effective diameter is the most important of all thread ele-
observable pattern will be visible. ments. It plays a big role in perfect fitting of the mating
threads.
• Eye
Effective diameter or pitch diameter is the diameter of an
imaginary co-axial cylinder which intersects the flanks of
the threads, such that the width of the threads (metal) and
Optical
flat widths of the spaces between the threads are equal, each
being-half the pitch.
Air wedge Effective diameter can be measured by the following
Surface to be methods.
tested 1. Thread micrometer method
2. One wire, two wire or three wire methods
Method of Checking Height of
a Component with the Help of Three Wire Method
Optical Flat In this method, three wires of equal and precise diameter
are placed in the groves of the threads, two on one side and
By using an optical flat the height of a given component can
one on the other side and the outer distance M is measured.
be checked against a known standard.
Chapter 5  •  Metrology and Inspection Computer Integrated Manufacturing  |  3.869

Surface Finish Measurement


The irregularity on the surface which is in the form of
hills and valleys varying in height and spacing are usually
termed as surface roughness, surface finish, surface texture
M or surface quality. If the respective or random deviations
from the nominal surface which forms pattern on the sur-
face. Surface texture, includes roughness, waviness, lays
and flaws.
Surface irregularities of small wavelength are called
primary texture or roughness. These are caused by direct
Effective diameter = E action of the cutting elements.
E=M–Q The surface irregularities of considerable wavelength of a
Where M = Micrometer reading across the top of wires. periodic character are called secondary texture or waviness.
Q = a constant depending up on wire diameter and flank Lay is the predominant direction or pattern of the surface
angle. texture.
P
= W (1 + cosecq) − cot q Flaws are surface irregularities or imperfections which
2
occur at infrequent and at random intervals. Cracks,
Where W = wire diameter
scratches, inclusions and similar defects are included in
P = pitch
this.
Best Wire Size
Evaluation of Surface Finish
Best size of wire is the size of wire of such diameter that
Surface finish is evaluated by the following methods.
it makes contact with the flanks of the thread on the pitch
line. Depending up on the pitch the best wire size varies. 1. Peak to valley height method
Measured with any other wire size will include error in the 2. The average roughness
measurement. 3. Form factor or bearing curves

B
Peak to Valley Height Method
(Rt Measurement)
r O
Pitch line It is the maximum peak to valley height within the assess-
Q A P ment length. The draw back of the method is that it may give
p same Rt value for two largely different texture.
4

Effective x Average Roughness


diameter
1. CLA method
p
2. RMS method
2
3. Ten point height method
Line OP is perpendicular to the flank position of the thread.
Let x = half the included angle. CLA (Centre Line Average) Method
In triangle OAP In this average values of ordinates from the mean line,
AP regardless of the arithmetic signs of the ordinates (average
Sin POA =
OP deviations from the nominal surface) are measured
AP CLA value (Ra)
i.e., sin(90 – x) =
OP h1 + h2 + h3 +  hn
=
AP AP n
\  OP = = = APsecx
sin (90 − x ) cos x A1 + A2 + A3 +  An
=
But OP = r, the radius of wire L
\ Best wire diameter =
∑ A
p L
db = 2APsecx = 2 sec x
4 1 L
 = ∫ h dx over 2 to 20 consecutive sampling
p L 0
i.e., db = 2 sec x lengths
3.870  |  Part III  •  Unit 7  •  Manufacturing Technology

RMS (Root Mean Square) Method In a CIM system a group of NC machines are connected
In this method also, the roughness is measured as the aver- together by an automated material handling system and
age deviations from the nominal surface. operating under computer control.
A computer integrated manufacturing system incorpo-
h12 + h22 +  hn2 rate many of the individual CAD/CAM technologies and
RMS value = concepts such as
n
Computer numerical control or CNC
L
1 2 Direct Numerical control or DNC
L ∫0
= h dx Computer aided process planning CAPP
Computer integrated production management industrial
Ten Point Height Method (Rz) roots.
Ten point height of irregularities is defined as the average
difference between the five highest peaks and five deepest Numerical Control
valleys within the sampling length measured from a line, Numerical control (NC) refers to the operations of machine
parallel to the mean line and not crossing the profile. tools from numerical data. Data for operations may be
stored on paper tape, magnetic tape, magnetic discs etc. As
2 1 numerical information is used, it is called numerical con-
trol. Machine tools and other machines are operated by a
3
series of loaded instructions. If the machine tool works with
4 a built in computer controlling, the system is known com-
5
puter numerical control (CNC)
The basic components of a NC system are
10 9
8 6 7 1. A program. i.e., a set of instructions
R2 R1 2. A machine control unit (MCU)
R6
3. The machine tool
L The MCU is further divided into two elements: The data
processing unit (DPU) and control loops unit (CLU). Data
1 process unit processes the coded data read from storage
Rz = [(R1 + R2 + R3 + R4 + R5) – (R6 + R7 + R8 + R9 + R10)] ×
5 devices and passes information such as position of each
axis, required direction of motion, speed, feed etc and aux-
This method is relatively simple, but does not account for iliary function control signals to CLU.
the frequency of the irregularities and profile shape. It is A typical program may contain an instruction like x +
used when cost is to be controlled and for checking rough 100, y + 50, s + 90
machining. This instruction is interpreted as to move a distance
100  mm in the x positive direction, 50 mm in the y +
Computer Integrated ­direction and rotate the spindle at 90 rpm clockwise.
The information pieces are decoded by the DPU and sent
Manufacturing System (CIMS) to CLU. The CLU operates the drive mechanisms as per
CIMS is a production system consisting of a group of the instructions, then receive feed back signals regarding
Numerical controlled (NC) machines connected together by the actual positions velocity etc. When one instruction is
an automated material handling system and operating under executed another is read.
computer control. Many of the individual CAD/CAM tech-
nologies are incorporated in CIM. Other concepts incor- Advantages of NC Systems
porated in CIM are Computer Numerical control (CNC), 1. High machine utilization
Direct Numerical Control (DNC), Computer Aided Process 2. Need for special tooling is mostly eliminated
planning (CAPP), Computer integrated production man- 3. High quality products can be manufactured
agement and Industrial robots. 4. Consistency in quality
CIMS depend upon the production requirements. Each 5. Quality is not dependent on operator skill
system vary according to the requirements. 6. Cost of production is less
Computer integrated manufacturing (CIM) is a recent 7. Minimum scrap
technology. It comprises of a combination of software and 8. In process inventory is less
hardware for product design, production planning, produc- 9. Higher productivity
tion control, production equipment and production process. 10. Reduced set up time
Chapter 5  •  Metrology and Inspection Computer Integrated Manufacturing  |  3.871

Disadvantages of NC System Advantages of CNC System over


1. Initial investment is very high for the specialized Conventional NC System
equipment 1. As the computer can be easily and readily
2. Redundancy of labour reprogrammed the system is very flexible. The
3. Down time is highly expensive machine can manufacture a part followed by other
4. Special skill is required for programming and parts of different designs.
operating the equipment 2. Editing and debugging programs, reprogramming
and plotting and printing part shapes etc are simpler.
CNC Retrofitting 3. Manufacturing programme for a component can be
easily called by computer.
Retrofitting means adding accessories to a given object to
This saves time and eliminates errors due to tape reading
improve its performance. High initial investment of the
4. Greater accuracy
CNC machines make it not affordable to small scale indus-
5. Ease of operation
tries which are back bone of our economy. It is possible
6. Trouble shooting is easier as the microprocessors
to modernize the existing conventional machines to CNC
have self diagnostic features.
machines by adding accessories and making slight design
modification. This is known as CNC retrofitting. Even
though the retrofitted machines are not as good as CNC Adaptive Control Systems (ACS)
machines, their performance is far better than the original ACS is a logical extension of CNC system in the sense that
machines. So when there is a budgetary constraint retrofit- in CNC system operating conditions are specified by the
ting can be resorted to. user in the form of a program. But in ACS calculation and
setting up of operating conditions like speed, feed, depth
Direct Numerical Control (DNC) of cut etc are done during machining by the control system
itself.
DNC is a manufacturing system in which a number of
For example in drilling the torque on the drill is measured
machines are inter connected using a computer through
and feed and speed or both are adjusted within programmed
direct connection in real time. In the case of NC or CNC
limits . Adaptive control is still in its infancy, since the effect
systems one computer is used to control one machine tool.
of various process variables on the finished part is still rela-
But in the case of DNC system one computer can be used to
tively unknown.
control more than 100 machines. One computer is designed
to provide instructions to each machine tool on demand.
The components of a DNC system are Basic Concepts of CAD and CAM
1. Central computer CAD/CAM means computer aided design and computer
2. Bulk memory which stores the CNC programs aided manufacturing. It is the technology concerned with
3. Tele communication lines the use of digital computers in design and production
4. Machine tolls Therefore CAD is the use of computer systems to assist
In DNC, the computer calls the program instruction from in the creation, modification, analysis or optimization of a
bulk storage and sends to machines as the need arises. It design.
also receives feed back from the machines. CAM is the use of computer systems to plan, manage
DNC system has the main draw back that if the central and control the operations of a manufacturing unit through
computer goes down, all the machines become in operative. direct or indirect computer interface with the plants pro-
This draw back is overcome by using Distributed Numerical duction resources.
control system in place of direct numerical control. In this A digital computer is the essential ingredient of a CAD/
type even if there is a central computer, the individual NC CAM.
machines are not directly controlled by this central com- A digital computer can be used for image processing,
puter. Each NC machine has its own dedicated on – board real time process control and for solving complex problem
microcomputer just like a CNC system. in a few seconds.
With the development of dedicated min- computers the A digital computer consists of the following three
benefits of DNC system can be realized in CNC system also. components
More over with the availability of small computers 1. Central processing unit (CPU)
with large memory, microprocessors and program edit- 2. Memory
ing capabilities CNC machines are widely used at pre- 3. Input/output section
sent. Also the availability of low cost programmable logic A modern CAD system can perform graphics and non-
controlled has helped in the successful implementation of graphics is function. It is based on interactive computer
CNC system. graphics (ICG)
3.872  |  Part III  •  Unit 7  •  Manufacturing Technology

Computer Aided Process Planning (CAPP) 1. Process rationalization


Process planning means the determination of the sequence 2. Increased productivity of process planners
of individual manufacturing operations required to produce 3. Reduced turn around time
a given part or product. In CAPP production planning is 4. Improved legibility
done with the use of computers. The operation sequence is 5. Incorporation of other application programs.
documented on a route sheet. The advantages of CAPP are

Exercises
Practice Problems 1
+0.003 +0.000
1. A 60 mm diameter shaft is made to rotate in a bush. 25.000 −0.000 mm (B)
(A) 25.000 +0.003 mm
Tolerances for both bush and shaft are 0.050 mm. The +0.003 +0.000
dimensions of the bush to give a minimum clearance of 25.030 −0.000 mm (D)
(C) 25.030 −0.003 mm
0.075 in the shaft basis system is
−0.075 +0.125 +0.05
60 −0.125 (B)
(A) 60 +0.075 7. A hole of size 30 −0.03 mm is to be checked by workshop
+0.125 −0.125
GO and NO GO plug gauges. Assuming wear allow-
(C) 59+0.025 (D) 60 −0.075 ance and gauge allowance as 10% of work tolerance,
size of the NO GO gauge will be
2. The following is the hole and shaft dimensions of an
+0.014 +0.014
assembly 30 −0.022 mm (B)
(A) 30 +.0.014 mm
+0.04
Hole: 30 +0.00 mm −0.014 −0.014

+0.06
30 +.0.022 mm (D)
(C) 30 −.0.022 mm
Shaft: 30 +0.04 mm 8. Determine the size of the general type NO GO plug
The type of fit is gauge for checking hole of a 30 H7/f8.
(A) Transition (B) Clearance
Given: i = 0.453 D + 0.001D microns (D in mm)
(C) Interference (D) Running
3. In a hole and shaft assembly of 30 mm nominal size Upper deviation of shaft = –5.5 D0.41
with following dimensions. Maximum and minimum Diameter step for 30 mm = 18 – 30 mm
(Least) metal limits of the shaft are −0.0231 +0.0210
+0.02 30 −0.0231 (B)
(A) 30 +0.0189
Hole: 30 −0.00 mm +0.0231 +0.0210
−0.040 30 +0.0210 (D)
(C) 30 −0.0189
Shaft: 30 −0.070 mm 9. Cold drawn shafts upto accuracy ±0.01 mm are avail-
(A) 29.960, 29.930 mm (B) 30.02, 30 mm able. An interference fit is to be designed for a 50 mm
(C) 30.07, 30.04 mm (D) 29.950, 29.930 mm basic size hole. Maximum and minimum interferences
4. A hole and mating shaft have nominal size of 50 mm. are 0.02 mm and 0.01 mm respectively. Tolerance for
Maximum clearance is 0.15 mm and minimum clear- hole will be
ance is 0.05 mm. Hole tolerance is 1.5 times the shaft (A) 0.04 mm (B) 0.06 mm
tolerance. Limits for hole in a shaft basis system is (C) 0.05 mm (D) 0.03 mm
(A) 49.02, 49.08 mm (B) 51.04, 51.10 mm 10.
For a 90 H8 e9 hole shaft assembly the GO gauge for
(C) 49.05, 49.11 mm (D) 50.05, 50.11 mm shaft will be
5. The interference between two mating parts of basic size (given:
100 mm is to be from 0.05 mm to 0.12 mm. Tolerance IT8: 25i
of shaft and hole are same. The hole size in a shaft basis T9: 40i
system is
Fundamental deviation
(A) 100.035, 100.000 mm
(B) 100.020, 100.085 mm For ‘e’ type shaft
(C) 100.00, 99.965 mm = –11D0.41
(D) 99.915, 99.88 mm (Assume hole basis system)
6. A 25 mm H8 hole is to be checked using a plug gauge. 0.0693 +0.0693
The hole high limit is 25.030 mm. Taking gauge mak- 90 −0.677
(A) (B) 90 −0.0777
er’s tolerance as 10% of work tolerance dimensions of +0.0693 −0.0693
the GO plug gauge will be 90 +0.677 (D)
(C) 90 −0.777
Chapter 5  •  Metrology and Inspection Computer Integrated Manufacturing  |  3.873

11. For the above problem the size of NO gauge for shaft +0.040
15. In an interchangeable assembly shafts of size 30 −0.010
will be +0.030
+0.1524 −0.1524 mm mates with holes of size 30 +0.020 The maximum
90 −0.1608 (B)
(A) 90 −0.1608
interference in microns in the assembly is
+0.0693 +0.1524
90 −0.677 (D) (A) 16 microns
(C) 90 −0.1608
(B) 18 microns
Direction for questions 12 and 13: (C) 22 microns
0.000 (D) 20 microns
40 −0.009 16. Two slip gauges of 10 mm width measuring 1.000
Q Datum
mm and 1.015 mm are kept side by side in contact
with each other lengthwise. An optical flat is kept
40 f7 resting on them and inspected using monochromatic
+1.00 light of wavelength 0.0058928 mm. The total number
75 0.000 of straight fringes that can be observed on both slip
gauges will be
(A) 4 (B) 6
(C) 3 (D) 8
17. In surface roughness measurement for a sampling
0.010 0.005 length of 0.8 mm, the graph is drawn to a vertical mag-
0.000 0.000 nification of 15000 and horizontal magnification of
φ32 φ27
100 and areas above and below datum line are 160, 90,
Refer the figure given above 170, 150 mm2 respectively. CLA value for this surface
Given IT7 = 16i diameter steps for 40 mm = 30 and 50 mm. is
Fundamental deviation (A) 0.6 mm (B) 0.9 mm
for f shaft = –5.5 D0.41 micron (C) 0.8 mm (D) 0.5 mm
12. Size 40 f 7 will be 18.
+0.0245 +0.0140
40 +0.0494
(A) 40 +0.0480
(B) Milled
−0.0245 −0.0140 2.0
40 −0.0494
(C) 40 −0.0480
(D)
13. Dimension of length Q will be
+0.009 +0.009 5.0
35−0.100
34 −0.100 (B)
(A)
+0.100 0.100 0.5
(C) 34 −0.009 (D)
35+0.009 mm
14. In a drawing the machined surface was represented as
shown above. The machining allowance of the surface is
(A) 0.5 mm (B) 5.0 mm
(C) 2.0 mm (D) Not shown
19. Match the following:

P.  Limits 1. Algebraic difference between the


P Q R actual size and the corresponding
basic size
S Q.  Fits 2.  Permissible variation in size

The part shown in figure is machined to the sizes given below: R.  Tolerance 3. Prescribed difference between
S = 35.00 ± 0.08 mm the dimensions of mating parts to
perform specific function
P = 12.00 ± 0.02 mm
Q = 9.99 ± 0.03 mm S.  Allowance 4. The ranges of permissible variation
in dimensions of a part.
Dimension R will have specifications
+0.04 +0.04 5. Degree of lightness and looseness
(A) 12 −0.02
13−0.02 (B) between the mating parts
+0.04 +0.02
13−0.01 (D)
(C) 12 −0.04
3.874  |  Part III  •  Unit 7  •  Manufacturing Technology

( A) P–4, Q–5, R–2, S–3 (B) P–2, Q–3, R–4, S–1 4 slip gauges totalling a height of 54.464 mm was
(C) P–3, Q–4, R–2, S–5 (D) P–1, Q–2, R–3, S–4 inserted below the top roller. The angle set up was
20. A sine bar was used to set up an angle with the help of (A) 30° (B) 25°
slip gauges. Distance between roller centres is 100 mm. (C) 33° (D) 15°

Practice Problems 2 (A) Transition fit (B) Interference fit


(C) Clearance fit (D) Shrink fit
1. Allowance and maximum clearance of a 30 H7/h8 fit
will be 8.
(A) 0.0054, 0.001 mm (B) 0,0.0054 mm 118.3 +0.08
−0.09
(C) 0.0054, 0.005 mm (D) 0,0.054 mm
2. It is possible to drill a 25 mm nominal size hole to an
+0.02 T
accuracy of 25− 0.02 mmusing a standard drill. A shaft
is to be machined to obtain a clearance fit in the above
hole such that allowance should be 0.01 mm and maxi-
mum clearance should not be more than 0.08 mm. The
tolerance on the shaft will be
(A) 0.04 mm (B) 0.05 mm 32.7 ± 0.02
(C) 0.03 mm (D) 0.06 mm
3. A hole and shaft assembly has the following dimensions 10.0+0.02 25.2+0.01
50H8/C8. Multiplier for grade IT8 is 25. Fundamental −0.01 −0.02
deviation for shaft C for D > 40 mm is (–95 + 0.8D) 30.4±0.01
and diameter step for 50 mm is 50 to 80 mm. Maximum
In the above sketch the dimension T will be
size of the shaft will be
+0.02 +0.03
(A) 49.8544 mm (B) 51.9072 mm 19 −0.01 mm (B)
(A) 20 −0.02 mm
(C) 50.9623 mm (D) 48.8233 mm +0.02
+0.02

4. For the shaft assembly 90 H8 e9 the minimum size of 19 −0.03 mm (D)


(C) 20 −0.03 mm
the shaft is 9. In an interchangeable assembly shaft of size
(Given: value of tolerance for IT8 = 25i 20 −−00..040 +0.020
0100 mm mate with holes of size 20 −0.000 . The
IT9 = 40i minimum clearance in the assembly will be
Value of fundamental deviation for ‘e’ type shaft = (A) 10 microns (B) 15 microns
–11D0.41 (C) 8 microns (D) 12 microns
(A) 84.7476 mm (B) 86.7475 mm 10. GO and NO GO plug gauges are to be designed for a
(C) 92.3456 mm (D) 89.8476 mm hole of 20 ++00..050
010 mm . Gauge tolerance can be taken as
5. For the above problem GO plug gauge size will be 10% of the hole tolerance. Following ISO system of
+0.0005 +0.0057 gauge design, sizes of GO and NO GO gauge will be
90 − 0.0057 (B)
(A) 90 +0.0005 respectively
+0.0057 +0.0005
(A) 20.134 mm, 20.164 mm
(C) 89 + 0.0005 (D) 89 − 0.0025
(B) 20.014 mm, 20.054 mm
6. For the above problem NO GO plug gauge size will be (C) 21.123 mm, 21.136 mm
+0.0576 +0.0082 (D) 21.227 mm, 21.732 mm
90 − 0.0524 (B)
(A) 90 + 0.0076
+0.0576 +0.0576 11. An unknown specimen and a set of slip gauges are
90 +0.0524 (D)
(C) 90 − 0.0524 placed over flat surface at a distance x. Using cadmium
light source of wave lengths 0.509 mm, and an opti-
H −7
7. For a hole shaft assembly 60, the type of fit will be cal flat about 4.75 fringes were observed over distance
m−6 between the slip gauges and the specimen. Difference
(Diameter step for 60 mm = 50 to 80 mm. in height between them will be
Fundamental deviation for an shaft (A) 1.3 mm (B) 1.1 mm
= + (IT7 – IT6) (C) 1.2 mm (D) 1.4 mm
IT7 = 16i 12. A threaded nut having 2 mm pitch with a pitch diam-
eter of 14.7 mm is to be checked for its pitch diameter
IT6 = 10i
using wires. Angle of threads is 60°. The diameter of
For i use the standard formula) wire used should be
Chapter 5  •  Metrology and Inspection Computer Integrated Manufacturing  |  3.875

(A) 1.155 mm (B) 2.255 mm Direction for questions 19 and 20:  In the measurement
(C) 1.055 mm (D) 2.055 mm of surface roughness, heights of 20 successive peaks and
1 3. Match the following: troughs were measured from a datum and these were
35, 25, 40, 20, 35
Features to be inspected Instrument 18, 42, 25, 35, 22
P. Pitches and angle 1.  Auto collimator 36, 18, 42, 22, 32
errors of screw thread 21, 37, 18, 35, 20 microns
Q.  latness error of surface 2.  Optical interferometer These measurements were obtained over a length of 18 mm
R. Alignment error of 3. Deviding head and dial 19. Approximate CLA or Ra value will be
machine slide way gauge (A) 27 micron (B) 29 micron
(C) 31 micron (D) 26 micron
S.  Profile of a cam 4.  Spint level
20. Approximate RMS value will be
5.  Sine bar
(A) 29.32 micron (B) 32.73 micron
6.  Toolmaker’s microscope (C) 31.18 micron (D) 28.87 micron
21. In the measurement of surface roughness the height of
(A) P–2, Q–3, R–4, S–1 (B) P–3, Q–4, R–5, S–2 10 successive peaks and valleys over datum line over a
(C) P–4, Q–5, R–2, S–3 (D) P–5, Q–2, R–1, S–6 specified sampling length were found to be
−0.008
14. A shaft has a dimension, f 40 −0.025 . The respective val-
Peaks 45 42 40 35 35 mm
ues of fundamental deviation and tolerance are.
(A) –0.008, –0.025 Valleys 30 25 25 24 18 mm
(B) –0.008, –0.017
(C) –0.017, 0.008 The Rz value of the surface will be
(D) –0.017, +0.025 (A) 15 mm (B) 20 mm
+0.018
15. A hole is of dimension f 90 +0.000 mm and the corre- (C) 12 mm (D) 18 mm
+0.012 22. Figure given below shows the dimension obtained on a
sponding shaft is of dimension 9+0.001 mm . When they component by a certain instrument
are assembled they will form a The instrument is
(A) Interference fit (B) Clearance fit (A) Precise but not accurate
(C) Transition fit (D) Running fit (B) Accurate but not precise
16. A 30 h7 shaft has the dimensional limits (C) Neither precise nor accurate
(A) 30.000, 29.979 (D) Sensitive
(B) 30.000, 30.021
(C) 30.000, 30.007
(D) 30.000, 29.993 Average
× × × × ×
17. Essential condition for an interference fit is that the × × × value
lower limit of the shaft should be
(A) Lesser than upper limit of the hole
(B) Greater than the lower limit of the hole True value
(C) Lesser than the lower limit of the hole
(D) Greater than the upper limit of the hole
18. In measuring the surface roughness of an object, a
graph was drawn to a vertical magnification of 10000 23. The reflector combined with auto collimater can be
and a horizontal magnification of 100 and the areas used for checking
above and below the datum lines were (A) parallelism (B) Circularity
(C) Surface finish (D) Alignment
Above 150 80 170 40 mm2 24. According to Taylor’s principle, NO GO gauge checks
Below 80 60 150 120 mm2 (A) Only important dimensions at a time
(B) All the dimensions at a time
The sampling length was 0.8 mm. (C) Only one feature at a time
Ra value (CLA) of the surface is (D) Only related dimensions at a time
(A) 0.08 mm (B) 1.53 mm 25. Expressing a dimension as 42.5/42.3 mm is the case of
(C) 0.85 mm (D) 1.06 mm (A) Unilateral tolerance (B) Bilateral tolerance
(C) Limiting dimensions (D) None of the above
3.876  |  Part III  •  Unit 7  •  Manufacturing Technology

2 6. Most accurate instrument is 3 blocks B1, B2 and B3 are to be inserted in a channel


(A) Vernier caliper of width S maintaining a minimum gap of width T =
(B) Screw gauge 0.125 mm, as shown in figure. For P = 18.75 ± 0.08, Q
(C) Slip gauge = 15.00 ± 0.12, R = 18.125 + 0.1 and S = 52.35 + x the
(D) Optical projector tolerance x is
27. Figure shows 3 wire method of inspecting screw threads. (A) –0.05 mm
The screw thread is ISO metric M 16 with pitch 2 mm (B) +0.05 mm
and effective diameter = 14.701 mm. Diameter of roller (C) –0.35 mm
used for measurement is 1.155 mm and corresponds to (D) +0.35 mm
best wire diameter (i.e., touches at points of effective 29. For measuring taper of a ring gauge, two balls of diam-
diameter) eter 30 mm and 15 mm were used. During inspection
the ball of 30 mm diameter was protruding by 2.5 mm
above top surface of the ring. This surface was located
at a height of 50 mm from the top of the 15 mm diam-
eter ball. The taper of the angle is
S (A) 25° (B) 20°
(C) 15° (D) 18°
30. A fits is specified as 25 H8/e8. The tolerance value
Over write measurement S will be for a basic diameter 25 mm in IT8 is 33 microns and
(a) 17.6 mm (B) 18.8 mm fundamental deviation for the shaft is –40 microns. The
(C) 18.2 mm (D) 19.4 mm maximum clearance of the fit in microns is
(A) 66 microns
28.
S (B) 73 microns
(C) 33 microns
R Q P
T (D) 106 microns
B3 B2 B1

Previous Years’ Questions


1. In an interchangeable assembly, shafts of size (A) 20.010 mm and 20.050 mm
+0.040
(B) 20.014 mm and 20.046 mm
25.000 −0.0100 mm mate with holes of size
+0.020 (C) 20.006 mm and 20.054 mm
25.000 −0.000 mm . the maximum possible clearance in (D) 20.014 mm and 20.054 mm
the assembly will be [2004] 4. In a 2-D CAD package, clockwise circular arc of
(A) 10 microns (B) 20 microns radius 5, specified from P1(15,10) to P2(10, 15) will
(C) 30 microns (D) 60 microns have its center at [2004]
2. During the execution of a CNC part program block (A) (10, 10) (B) (15, 10)
NO20 GO2 X45.0 Y25.0 R5.0 the type of tool motion (C) (15, 15) (D) (10, 15)
will be [2004] 5. Match the following: [2004]
(A) circular interpolation – clockwise
Feature to be Inspected Instrument
(B) circular interpolation – counter clockwise
(C) linear interpolation P. Pitch and Angle errors of 1.  Auto Collimator
screw thread
(D) rapid feed
Q. Flatness error of a 2.  Optical Interferometer
3. GO and No-Go plug gages are to be designed for Surface plate
+0.050

a hole 20.000 +0.010 mm. Gauge tolerances can be R. Alignment error of a 3. Dividing Head and
machine slideway Dial gauge
taken as 10% of the hole tolerance. Following ISO S.  Profile of cam 4.  Spirit Level
5.  Sine bar
system of gauge design, sizes of GO and NO-GO
6. Tool makes’s
gauge will be respectively [2004] microscope
Chapter 5  •  Metrology and Inspection Computer Integrated Manufacturing  |  3.877

(A) P-6 Q-2 R-4 S- 6 (B) P-6 Q-4 R-1 S-3 (A) AW, LC and M
(C) P-5 Q-2 R-1 S-6 (D) P-1 Q-4 R-4 S-2 (B) AW, D, LC and M
6. In order to have interference fit, it is essential that the (C) D, LC, P and SW
lower limit of the shaft should be [2005] (D) D, LC and SW
(A) greater than the upper limit of the hole 9. NC contouring is an example of  [2006]
(B) lesser than the upper limit of the hole (A) continuous path positioning
(C) greater than the lower limit of the hole (B) point-to-point positioning
(D) lesser than the lower limit of the hole (C) absolute positioning
7. The tool of an NC machining has tomove along a cir- (D) incremental positioning
cular arc from (5, 5) to(10, 10) while performing an 10. A ring gauge is used to measure [2006]
operation. The center of the arc is at (10, 5). Which (A) outside diameter but not roundness
one of the following NC tool path commands per- (B) roundness but not outside diameter
forms the above mentioned operation? [2005] (C) both outside diameter and roundness
(A) N010 G02 X10 Y10 X5 Y5 R5 (D) only external threads
(B) N010 G03 X10 Y10 X5 Y5 R5 11. Which type of motor is NOT used in axis or spindle
(C) N010 G01 X5 Y5 X10 Y10 R5 drives of CNC machine tools?[2007]
(D) N010 G02 X5 Y5 X10 Y10 R5 (A) induction motor (B) dc servo motor
8. Which among the NC operations given below are (C) stepper motor (D) linear servo motor
0.050
continuous path operations?[2005]
12. A hole is specified as 400.00 mm. The mating shaft
Arc welding (AW) Milling (M) has a clearance fit with minimum clearance of 0.01
Drilling (D) Punching in sheet mm. The tolerance on the shaft is 0.04 mm. The maxi-
metal (P) mum clearance in mm between the hole and the shaft
Laser cutting of Spot welding (SW) is  [2007]
(A) 0.04 (B) 0.05
Sheet Metal(LC)
(C) 0.10 (D) 0.11

Direction for questions 13 and 14:

f Table
Pulse Stepper Gear box
generator motor
U Nut

Lead screw

In the feed drive of a Point–to–Point open CNC drive, 14. A customer insists on a modification to change the
a stepper motor rotating at 200 steps/rev drives a table BLU of the CNC drive to 10 microns without chang-
through a gear box and lead screw-nut mechanism ing the table speed. The modification can be accom-
(pitch = 4 mm, number of starts = 1). The gear ratio = plished by [2008]
⎛ Output rotational speed ⎞ 1 1 f
=⎜ ⎟ is given by U = . The (A) Changing U to and reducing f to
⎝ Input rotational speed ⎠ 4 2 2
stepper motor (driven by voltage pulses from a pulse gen- 1
(B) Changing U to and increasing f to 2f
erator) executes 1 step/pulse of the pulse generator. The 8
frequency of the pulse train from the pulse generator is f = 1
10,000 pulses per minute. (C) Changing U to and keeping f unchanged
2
13. The Basic Length Unit (BLU), i.e., the table move- (D) Keeping U unchanged and increasing f to 2f
ment corresponding to 1 pulse of the pulse generator, 15. Which of the following is the correct data structure
is [2008] for solid models? [2009]
(A) 0.5 microns (A) solid part → faces → edges → vertices
(B) 5 microns (B) solid part → edges → faces → vertices
(C) 50 microns (C) vertices → edges → faces → solid parts
(D) 500 microns (D) vertices → faces → edges → solid parts
3.878  |  Part III  •  Unit 7  •  Manufacturing Technology

16. Match the following: [2009]

NC Code Definition
Z = 40
P  M05 1.  Absolute coordinate system
Q G01 2.  Dwell
Z = 20
R G04 3.  Spindle stop
S  G90 4.  Linear interpolation
Z=0

(A) P-2, Q-3, R-4, S-1


(B) P-3, Q-4, R-1, S-2
(C) P-3, Q-4, R-2, S-1 (A) 13.334 (B) 15.334
(D) P-4, Q-3, R-2, S-1 (C) 15.442 (D) 15.542
+0.015
1 7. What are the upper and lower limits of the shaft rep- 21. A hole is of dimension ϕ 9+0 mm. . The correspond-
resented by 60 f8? +0.010
Use the following data: ing shaft is of dimension ϕ 9+0.001 mm.
The resulting
Diameter 60 lies in the diameter step of 50–80 mm assembly has [2011]
Fundamental tolerance unit, i, in mm = 0.45D1/3 + (A) loose running fit (B) close running fit
0.001D, where D is the representative size in mm; (C) transition fit (D) interference fit
Tolerance value for IT8 = 25i. Fundamental deviation 22.
In an interchangeable assembly, shafts of
for ‘f’ shaft = –5.5D0.41 [2009] +0.040
(A) Lower limit = 59.924 mm, Upper Limit = 59.970 size 25.000 −0.020 mm mate with holes of size
+0.030
mm
25.000 −0.020 mm. The maximum interference (in
(B) Lower limit = 59.954 mm, Upper Limit = 60.000
microns) in the assembly is [2012]
mm
(A) 40 (B) 30 (C) 20 (D) 10
(C) Lower limit = 59.970 mm, Upper Limit = 60.016
mm 23. A metric thread of pitch 2 mm and thread angle 60° is
(D) Lower limit = 60.000 mm, Upper Limit = 60.046 inspected for its pitch diameter using 3-wire method.
mm The diameter of the best size wire in mm is[2013]
−0.009 (A) 0.866 (B) 1.000
18. A shaft has a dimension, ϕ 35−0.025 . The respective (C) 1.154 (D) 2.000
values of fundamental deviation and tolerance are +0.020

[2010] 24. Cylindrical pins of 25+0.010 mm diameter are elec-


(A) −0.025, ±0.008 troplated in a shop. Thickness of the plating is 30 ±2.0
micron. Neglecting gauge tolerances, the size of the
(B) −0.025, 0.016
GO gauge in mm to inspect the plated components is
(C) −0.009, ±0.008
 [2013]
(D) −0.009, 0.016 (A) 25.042 (B) 25.052
19. In a CNC program block, N002 G02 G91 X40 Z40…, (C) 25.074 (D) 25.084
G02 AND G91 refer to[2010] 25. In a CAD package, mirror image of a 2D point P(5,10)
(A) circular interpolation in counterclockwise direc- is to be obtained about a line which passes through
tion and incremental dimension the origin and make an angle of 45° counterclockwise
(B) circular interpolation in counterclockwise direc- with the x-axis. The coordinates of the transformed
tion and absolute dimension point will be [2013]
(C) circular interpolation in clockwise direction and (A) (7.5, 5) (B) (10, 5)
incremental dimension (C) (7.5, –5) (D) (10, –5)
(D) circular interpolation in clockwise direction and
absolute dimension 26. For machining a rectangular island represented by
coordinates P(0, 0), Q(100, 0), R(100, 50) and S(0, 50)
20. A taper hole is inspected using a CMM, with a probe on a casting using CNC milling machine, an end mill
of 2 mm diameter. At a height, Z = 10 mm from the with a diameter of 16 mm is used. The trajectory of the
bottom, 5 points are touched and a diameter of cir- cutter centre to machine the island PQRS is[2014]
cle (not compensated for probe size) is obtained as (A) (–8, –8), (108, –8), (108, 58), (–8, 58), (–8, –8)
20 mm. Similarly, a 40 mm diameter is obtained at a (B) (8, 8), (94, 8), (94, 44), (8, 44), (8, 8)
height Z = 40 mm. The smaller diameter (in mm) of (C) (–8, 8), (94, 0), (94, 44), (8, 44), (–8, 8)
hole at Z = 0 is [2010] (D) (0, 0), (100, 0), (100, 50), (50, 0), (0, 0)
Chapter 5  •  Metrology and Inspection Computer Integrated Manufacturing  |  3.879

27. Which one of the following instruments is widely H1


used to check and calibrate geometric features of H2 d1 Diameter
machine tools during their assembly? [2014] C
(A) Ultrasonic probe
H
(B) Coordinate Measuring Machine (CMM) B
(C) Laser interferometer A
R
(D) Vernier callipers
Recessed Ring
28. For the given assembly : 25 H7/g8, match Group A
with Group B [2014]
d2 Diameter D
Group A Group B
(P) H (I) Shaft Type The distance H2 = 35.55 mm and H1 = 20.55 mm. The
diameter (D, in mm) of the ring gauge is ____
(Q) IT8 (II) Hole Type
33. A GO–No GO plug gauge is to be designed for meas-
(R) IT7 (III) Hole Tolerance Grade
uring a hole of nominal diameter 25 mm with a hole
(S) g (IV) Shaft Tolerance Grade tolerance of ±0.015 mm. Considering 10% of work
tolerance to be the gauge tolerance and no wear con-
(A) P-I, Q-III, R-IV, S-II dition, the dimension (in mm) of the GO plug gauge
(B) P-I, Q-IV, R-III, S-II as per the unilateral tolerance system is [2014]
(C) P-II, Q-III, R-IV, S-I +0.003 +0.000
(D) P-II, Q-IV, R-III, S-I 24.985−0.003 (B)
(A) 25.015−0.006
+0.03 +0.003
2 9. The flatness of a machine bed can be measured using 24.985−0.000
24.985−0.03 (D)
(C)
 [2014]
(A) Vernier callipers 34. Holes of diameter 25.0 ++00..040
020 mm are assembled inter-
(B) Auto collimator changeably with the pins of diameter 25.0 +−00..005
008 mm.
(C) Height gauge The minimum clearance in the assembly will be:
(D) Tool maker’s microscope  [2015]
30. A robot arm PQ with end coordinates P(0, 0) and (A) 0.048 mm (B) 0.015 mm
Q(2, 5) rotates counter clockwise about P in the XY (C) 0.005 mm (D) 0.008 mm
plane by 90°. The new coordinate pair of the end point 35. The function of interpolator in a CNC machine con-
Q is  [2014] troller is to [2015]
(A) (–2, 5) (B) (–5, 2) (A) control spindle speed
(C) (–5, –2) (D) (2, –5) (B) coordinate feed rates of axes
31. For the CNC part programming, match Group A with (C) control tool rapid approach speed
Group B : [2014] (D)  perform Miscellaneous (M) functions (tool
change, coolant control etc.)
Group A Group B 36. In the assembly shown below, the part dimensions
circular interpolation, counter
(P) (I) G02 are: [2015]
clockwise
(Q) dwell (II) G03 L1 = 22.0±0.01 mm,
circular interpolation, L2 = L3 = 10.0±0.005 mm.
(R) (III) G04
clockwise
(S) point to point countering (IV) G00 Assuming the normal distribution of part dimensions, the
dimension L4 in mm for assembly condition would be:
(A) P-II, Q-III, R-I, S-IV
(B) P-I, Q-III, R-II, S-IV
(C) P-I, Q-IV, R-II, S-III
(D) P-II, Q-I, R-III, S-IV
32. The diameter of a recessed ring was measured by
using two spherical balls of diameter d2 = 60 mm and L4
L2 L3
d1= 40 mm as shown in the figure. [2014] L1

(A) 2.0±0.008 (B) 2.0±0.012


(C) 2.0±0.016 (D) 2.0±0.020
3.880  |  Part III  •  Unit 7  •  Manufacturing Technology

37. A triangular facet in a CAD model has vertices: 42. Match the following: [2016]
P1(0,0,0); P2(1,1,0) and P3(1,1,1). The area of the
facet is: [2015]   P.  Feeler gauge    I.  Radius of an object
(A) 0.500 (B) 0.707 Q.  Fillet gauge II.  Diameter within limits by
(C) 1.414 (D) 1.732 comparison
  R.  Snap gauge III. Clearance or gap
38. Which one of the following statements is TRUE?
between components
 [2015]
 S.  Cylindrical plug gauge   IV. Inside diameter of
(A) The ‘GO’ gage controls the upper limit of a hole.
straight hole
(B) The ‘NO GO’ gage controls the lower limit of a
shaft. (A) P-III, Q-I, R-II, S-IV
(C) The ‘GO’ gage controls the lower limit of a hole. (B) P-III, Q-II, R-I, S-IV
(D) The ‘NO GO’ gage controls the lower limit of a (C) P-IV, Q-II, R-I, S-III
hole. (D) P-IV, Q-I, R-II, S-III
39. A project consists of 7 activities. The network along 4 3. The figure below represents a triangle PQR with
with the time durations (in days) for various activities initial coordinates of the vertices as P(1, 3), Q(4,
is shown in the figure. 5) and R(5, 3, 5). The triangle is rotated in the
X-Y plane about the vertex P by angle q in clock-
12 11 10 wise direction. If sin q = 0.6 and cos q = 0.8,
1 3 5 6
the new coordinates of the vertex Q are: [2016]
14 12
9 Y
Q(4, 5)
7
2 4

The minimum time (in days) for completion of the R(5, 3, 5)


project is _______. [2015] P(1, 3)
40. A drill is positioned at point P and it has to proceed to
point Q. The coordinates of point Q in the incremental
O X
system of defining position of a point in CNC part
program will be [2015]
(A) (4.6, 2.8) (B) (3.2, 406)
Y
(C) (7.9, 5.5) (D) (5.5, 7.9)
4 4. For the situation shown in the figure below the expres-
Q sion for H in terms of r, R and D is : [2016]
H = D + r 2 + R2
(A)
P
12 H = (R + r) + (D + r)
(B)
5
D
X
3 4

(A) (3, 12) (B) (5, 7) r


(C) (7, 12) (D) (4, 7)
4 1. In a CNC milling operation, the tool has to machine
H
the circular arc from point (20, 20) to (10, 10) at a R
sequence number 5 of the CNC part program. If the
center of the arc is at (20, 10) and the machine has
incremental mode of defining position coordinates,
the correct tool path command is: [2015]
(A) N 05 G90 G01 X-10 Y-10 R10
(B) N 05 G91 G03 X-10 Y-10 R10
(C) N 05 G90 G03 X20 Y20 R10 H = (R + r) + D 2 − R 2
(C)
(D) N 05 G91 G02 X20 Y20 R10
H = (R + r) + 2 D ( R + r ) − D 2
(D)
Chapter 5  •  Metrology and Inspection Computer Integrated Manufacturing  |  3.881

45. Match the following part programming codes with Monochromatic light
their respective functions: [2016]
Part Programming
Functions
Codes
  P. G01    I.   Spindle stop
Q. G03 II. Spindle rotation,
clockwise
  R. M03 III. C
 ircular interpolation, θ
anticlockwise
 S. M05   IV. Linear interpolation
If the wavelength of light used to get a fringe spacing
(A) P–II, Q–I, R–IV, S–III of 1 mm is 450 nm, the wavelength of light (in mm) to
(B) P–IV, Q–II, R–III, S–I get a fringe spacing of 1.5 mm is ________. [2016]
(C) P–IV, Q–III, R–II, S–I
(D) P–III, Q–IV, R–II, S–I 47. A point P (1, 3, –5) is translated by 2iˆ + 3 ˆj − 4 kˆ
and then rotated counter clockwise by 90o about the
46. Two optically flat plates of glass are kept at a small z-axis. The new position of the point is: : [2016]
angle θ as shown in the figure. Monochromatic light is
incident vertically. (A) (–6, 3, –9) (B) (–6, –3, –9)
(C) (6, 3, –9) (D) (6, 3, 9)

Answer Keys
Exercises
Practice Problems 1
1. B 2. C 3. A 4. D 5. D 6. B 7. D 8. C 9. A 10. D
11. B 12. C 13. D 14. A 15. D 16. B 17. C 18. A 19. A 20. C

Practice Problems 2
1. B 2. C 3. A 4. D 5. B 6. C 7. A 8. D 9. A 10. B
11. C 12. A 13. D 14. B 15. C 16. A 17. D 18. D 19. B 20. A
21. A 22. A 23. D 24. C 25. C 26. D 27. C 28. A 29. B 30. D

Previous Years’ Questions


1. D 2. A 3. D 4. C 5. C 6. A 7. A 8. B 9. A 10. A
11. C 12. C 13. B 14. D 15. C 16. C 17. A 18. D 19. C 20. A
21. C 22. C 23. C 24. D 25. B 26. A 27. C 28. D 29. B 30. B
31. A 32.  91 to 94 33. D 34. B 35. B 36. D 37. B 38. C
39.  39 to 40 40. D 41. B 42. A 43. A 44. D 45. C 46.  675 47. A
3.882  |  Part III  •  Unit 7  •  Manufacturing Technology

Test

Manufacturing Technology Time: 60 Minutes

Direction for questions 1 to 25: Select the correct a­ lternative 8. A rolling mill has rollers of 400 mm diameter. The
from the given choices coefficient of friction is 0.15. In order to reduce the
1. The condition for interference fit is that the lower limit thickness from 150 mm to 10 mm for a strip, the num-
of the shaft ber of passes required are
(A) Should be greater than the upper limit of the hole (A) 4 (B) 5
(B) Should be greater than the lower limit of the hole (C) 6 (D) 7
(C) Should be less than the upper limit of the hole 9. Consider tungsten, aluminium, copper and titanium.
(D) Should be less than the lower limit of the hole If they are arranged in the decreasing order of magni-
2. In an assembly of shaft and hole, shaft size is specified tude of forgeability
as (A) Copper, tungsten, aluminium, titanium
(B) Aluminium, titanium, tungsten, copper
+0.010 -0.000
20 -0.040 mm and hole size as 20 +0.020 mm. (C) Aluminium, titanium, copper, tungsten
(D) Aluminium, copper, titanium, tungsten
The maximum clearance possible in the assembly
10. The pattern allowance for a cylindrical casting of diam-
(A) 20 microns (B) 30 microns
eter 100 mm and length 150 mm is specified as follows:
(C) 40 microns (D) 10 microns
shrinkage allowance is 2 in 50 and machining allow-
3. Cold working of steel means: ance is 2 mm/side. The pattern size is
(A) Mechanical working of steel below the lower criti- (A) d = 110.16 mm, d = 170.16 mm
cal temperature (B) d = 715.32 mm, d = 178.56 mm
(B) Mechanical working of steel below the recrystal- (C) d = 108.16 mm, d = 160.16 mm
lisation temperature (D) d = 112.5 mm, d = 163.52 mm
(C)  Mechanical working below 2/3 of the melting
11. Which of the following gating ratio indicates
­temperature
a pressurised system
(D) Mechanical working of steel below the upper criti-
(A) 4:8:3 (B) 1:3:3
cal temperature
(C) 1:2:4 (D) 3:3:4
4. In punching operation the clearance is given:
12. If ‘a’ is the rate rake angle of the tool and ‘f’ the shear
(A) On the die
angle, then shear strain ‘e’ is given by
(B) On the punch
(A) e = cot (f - a) + tan f
(C) In the die or punch
(B) e = cos (f - a) + tan f
(D) In the die and the punch
(C) e = cos (f - a) + cot f
5. From a sheet metal of thickness 1 mm a cup of (D) e = tan (f - a) + cot f
diameter 30 mm and height 150 mm is to be drawn.
If limiting draw ratio is 1.8, the number of draws 13. When chip thickness ratio is 1 and tool rake angle is
required are 12°, shear angle is equal to
(A) 2 (B) 3 (C) 4 (D) 5 (A) 33° (B) 51°
(C) 57° (D) 62°
6. A lead plate is mechanically worked at room tempera-
ture. It is 14. 18–4–1 High speed tool steel has the following
(A) A cold working process composition
(B) A hot working process (A) 18% molybdenum, 4% tungsten, 1% vanadium
(C) Neither hot working nor cold working (B) 18% tungsten, 4% chromium, 1% vanadium
(D) It is not defined (C) 18% molybdenum, 4% chromium, 1% cobalt
(D) 18% tungsten, 4% molybdenum, 1% cobalt
7. A metal having recrystallisation temperature TA is cold
worked. The recrystallisation temperature of this cold 15. Ceramic tools generally have
worked item is TA′. Then (A) Positive rake angle
(A) TA = TA′ (B) T A > TA′ (B) Negative rake angle
(C) Zero rake angle
(C) TA < TA′ (D) Cannot say from this data
(D) Zero or positive rake angle
Test  |  3.883

16. Aspiration effect in gating system is 21. Two grades of tools (A and B) are used to machine a
(A) Due to pressure difference, air flows from the gat- steel piece. The cutting speed per minute of tool life is
ing system to outside 80 for the first tool (A) and cutting speed per minute of
(B) It is the intake of air from outside atmosphere to tool life for second tool (B) is 100. If Tailors index for
the gate due to pressure difference first tool (A) is 0.2 and that for second tool (B) is 0.25,
(C) It is the oxidation of molten metal during pouring the tool which is giving maximum tool life is
(D) It is the purposeful admission of air into the mould (A) Tool A (B) Tool B
17. Non-consumable electrodes are used in (C) Both A & B (D) Data is insufficient
(A) TIG welding 22. During a machining process with 12° rake tool, the
(B) MIG welding chip thickness ratio is found to be 0.4. The shear angle
(C) Submerged arc welding is
(D) Resistance projection welding (A) 23.1° (B) 31.2°
18. Two cutting tools are designated as (C) 34.5° (D) 36.2°
A: 5, 6, 8 – 3 - 7 - 25 -0.2 mm 23. In an operation the ratio between thrust force and cut-
B: 5 - 6 - 8 - 3 - 5 - 30 - 0.2 mm ting force is found to be 2.8. Then the operation is
For the same speed and feed, which of the cutting tool (A) Drilling (B) Turning
gives better surface finish (C) Grinding (D) Milling
(A) B has better surface finish
Direction for questions 24 and 25:
(B) A produces better surface finish
A cylindrical rod of 120 mm diameter is forged from 60
(C) Both A & B give the same surface finish
mm height to 40 mm height at 900°C. The flow stress of the
(D) Data is insufficient
material is 75 MPa
19. Out of the following cutting tools, the one which is
24. The work of deformation will be
hardest next to diamond
(A) 51 kNm (B) 61 kNm
(A) Cemented carbide (B) HSS
(C) 71 kNm (D) 81 kNm
(C) Cubic boron nitride (D) Ceramics
25. If a drop hammer weighing 16 kN is used for forging to
20. In orthogonal cutting operation the cutting force and
be done in one blow, the height of fall for the hammer
thrust force are 950 N and 450 N respectively. The rake
is
angle is zero. The coefficient of friction is
(A) 2.5 m (B) 3.2 m
(A) 0.21 (B) 0.32
(C) 4.8 m (D) 5.6 m
(C) 0.4 (D) 0.47

Answer Keys
1. A 2. B 3. B 4. A 5. B 6. B 7. B 8. C 9. D 10. C
11. A 12. D 13. B 14. B 15. B 16. B 17. A 18. A 19. C 20. D
21. A 22. A 23. C 24. A 25. B

You might also like